উত্তর

 

 

 

 

 

পুরানো প্রশ্ন ও উত্তর:        

***************************************************************

১৭ এপ্রিল ২০২৩

অনেকদিন এটি চালিয়েছি! যতই দিন যাচ্ছে ততই মনে হচ্ছে মানুষকে আসলে সামনে  থেকে দেখতে হয়, মুখের দিকে তাকিয়ে তাদের সাথে কথা বলতে হয়। বিশেষ করে তোমাদের মত ছোট ছেলেমেয়েদের সাথে!

এরকম একটা স্ক্রিনের ভেতর দিয়ে মানুষের সাথে কথা বলে কোনো আনন্দ নেই।

তাই আপাতত এখান থেকে বিদায়। সবাই ভালো থেকো।

****************************************************************

২৮ আগস্ট ২০২২

প্রশ্ন:  স্যার, নবম দশম শ্রেণির পদার্থবিজ্ঞান বইয়ের বল অধ্যায় এর ৮১ পৃষ্টায় কপিকল এর যে সূত্র আর উদাহরণ আছে, সেইটা ভুল। একটু চেক করবেন স্যার।

উত্তর: তুমি ঠিকই বলেছ। আমি বহু বছর আগে NCTB কে এটি শুদ্ধ করে পাঠিয়েছি। জানতাম না তারা বইটি শুদ্ধ করেনি। পাঠ্য বইয়ে ভুল থাকা খুব খারাপ অথচ এটি ঘটে যাচ্ছে। আমি আরও যে পরিবর্তনগুলো পাঠিয়েছিলাম সেগুলোর কী অবস্থা কে জানে। এখানে দিয়ে দিচ্ছি।

পৃষ্ঠা ১৪ ১৫   পৃষ্ঠা ৮১

১২ জুলাই, ২০২২

খুবই তাড়াহুড়া করে লেখা হয়েছে, কাজেই যদি ভুল ত্রুটি থাকে তাহলে নিজেরা নিজেরা ঠিক করে নিও, প্লিজ।

যাদের এখনই উত্তর দরকার তারা m.z.iqbal@mzi.rocks তে একটা ই-মেইল পাঠাতে পার।

প্রশ্ন: :’) আমরা আপনার কেউ না!
উত্তর: কে বলেছে তোমরা আমার কেউ না। তুমি কি জান, আমার কাছে তোমরাই আমার সবকিছু?

প্রশ্ন: স্যার আপনাকে চিঠি পাঠানোর ঠিকানা কি? আয়েশা চুয়াডাঙ্গা
উত্তর:  m.z.iqbal@mzi.rocks একটা ই -মেইল পাঠিয়ে সেখানে আবার জিজ্ঞেস কর।

প্রশ্ন: আমার প্রশ্নের উত্তর দেওয়া হচ্ছে না কেন? ভুতি,সিলেট।
উত্তর: এই যে দিলাম, খুশি?

প্রশ্ন: ৪ মাস ধরে এই সাইটে কোনো উত্তর দেওয়া হয়না কেন? ভুতি,সিলেট।
উত্তর: কারন এই সাইটটা একজন খুবই অকাজের মানুষ ম্যানেজ করে!

প্রশ্ন: দুনিয়ায় যাই হোক না কেন তার দোষ আমার কেন হবে? আমি মেয়ে হয়ে জন্মেছি,এটা কি আমার দোষ? শ্রুতি,সিলেট।
উত্তর: কে বলেছে সব দোষ তোমার? এরকম কথাবার্তা এক কান দিয়ে ঢুকিয়ে অন্য কান দিয়ে বের করে দিও। আর মেয়ে হয়ে জন্মানো মোটেও দোষের নয় বরং বলা যায় এটা অনেক সৌভাগ্যের ব্যাপার। তার একটা কারণ ছেলেদের ওয়াই ক্রোমোজোম এই একটুখানি, ছোট্ট। আর মেয়েদের এক্স ক্রোমোজোম বিশাল! কাজেই বিজ্ঞানীরা বলেন একসময় পৃথিবীতে শুধু মেয়েরাই থাকবে, ছেলেরা থাকবে না!

প্রশ্ন: আসসালামু আলাইকুম স্যার।আপনার কাছে আমার অনুরোধ আমাদের জন্য ‘টুনটুনি ও ছোটাচ্চু’ এর আরো কিছু বই লিখুন। নাম:ইরফান শ্রেণী:৩য় শহর:ঢাকা
উত্তর: ঠিক আছে লিখব।

প্রশ্ন: স্যার, নবম শ্রেণীর পদার্থবিজ্ঞান বইয়ে বলা আছে যে‌, কোনো পাথরকে উপরে তোলার জন্য তার ওজনের সমান বল প্রয়োগ করতে হবে। কিন্তু পাথরটির উপরে এবং নিচে সমান বল প্রয়োগ হওয়ায় এটির তো সাম্যবস্থায় থাকার কথা! তাহলে পাথরটিকে উপরে কিভাবে তোলা যাবে?? শাইরা, নবম শ্রেণী, বরিশাল।
উত্তর: তুমি ঠিকই বলেছ। বলা উচিত ওজন থেকে একটুখানি বেশি বল প্রয়োগ করতে হবে । তবে ওজন থেকে বেশি বল প্রয়োগ করলে পাথরটার ত্বরণ হবে সেজন্য ঠিক পাথরটার সমান ওজনের বল প্রয়োগ করার কথা বলা হয়।

প্রশ্ন: স্যার, আমি গত একদিনে আমি আপনার ৩টা কিশোর উপন্যাস শেষ করেছি। আমার আউটবই পড়াটা কেউ-ই ততোটা পছন্দ করে না। তবে আমি থেমে নেই! আমার তো বই কেনার সামর্থ্য নেই, তাই পিডিএফ পড়ি, আমার পড়া বইয়ের সংখ্যা ৩০০ মাত্র। যেদিন আমার টাকা হবে, আমি একটা লাইব্রেরি বানাব। আমি একাদশ শ্রেণিতে পড়ি। যদিও এখন প্রচুর পাঠ্যবই পড়ার প্রয়োজন, তবুও আমার মনেহয়, আপনার কিশোর উপন্যাস পড়লে অনুপ্রেরণা পাই! আমি কবিতা লিখি, আর গল্পও। বড়ো হয়ে আমি আপনার মতো একজন ভালোমনের লেখক হতে চাই! অবশ্য স্যার, আমি কিছুটা বড়ো হয়েই গিয়েছি! সবাই তো প্রশ্ন করে, তারপর উত্তর পায়। আমি আপনার কাছ থেকে প্রশ্ন না করেই উত্তরের প্রত্যাশা করছি। বিষয়টা কিন্তু মজার লাগছে আমার কাছে। অবিরাম ভালোবাসা রইল স্যার।Moli Rani Barmon (Mo Li Ra Ni B Ar Mo N) Mymensingh
উত্তর: আমি শুনে খুবই খুশি হলাম যে তুমি প্রচুর বই পড়ো। পাঠ্যবই তো সবাইকেই পড়তে হয় তার মানে এই নয় যে একজন পাঠ্যবইয়ের বাইরে অন্য কোন বই পড়বে না! বই পড়ার অভ্যাসটা যেন থাকে, সারা জীবন বই পড়বে, দেখবে তোমার জীবনটা কত সুন্দর হবে। আমি পৃথিবীর মানুষকে দুই ভাগে ভাগ করি একভাগ, যারা বই পড়ে আরেক ভাগ যারা বই পড়ে না। যারা বই পড়ে তারাই কিন্তু পৃথিবীটাকে চালায়। এখন তোমরা ঠিক করো, তোমরা কি পৃথিবীকে চালাবে, নাকি অন্যরা পৃথিবী চালাবে আর তোমরা দূর থেকে সেটা দেখবে।

প্রশ্ন: আসসালামু আলাইকুম।আশা করি ভালো আছেন।আমার দুইটি খুব ছোট প্রশ্ন ছিলো ১. আপনার প্রিয় শিল্পী কে? ২.আপনার প্রিয় গান কোনটি? আশা করি উত্তর দেবেন।ভালো থাকবেন।আপনার জন্য অনেক অনেক ভলোবাসা।💜 কেয়া পাশা, ৭ম শ্রেণি,VNSC
উত্তর: আমার প্রিয় শিল্পী তো একজন নয়, অনেক। কতজনের নাম বলবো বলো। ঠিক সেরকম আমার প্রিয় গানও তো একটি নয়, অনেক। কয়টার কথা বলব বল। কিন্তু তুমি যেহেতু জিজ্ঞেস করেছ একটা উত্তর তো দেওয়া উচিত। ও আমার দেশের মাটি, এই গানটি তোমার কেমন লাগে?

প্রশ্ন: Sir, how are you? Hope you are fit as a fiddle! … … … … Sir, I have a serious question- do you people always look for genius kids out there? I will leave the answer to you, but I can assure you that though I got kicked from the Math Olympiad 2 times, I am good at Physics, and I will try my best to go to the International Physics Olympiads and earn my motherland a thing to remember! … … … … (Ok, I will tell you my name now if you haven’t guessed it- O……kho!) Another hint to recognize me- I proposed the idea to publish a “Neuron e Onuronon” based on Physics 😀 Sayonara Sensei. Onegai Shimasu! (Goodbye again, Sir. Keep well and best of luck ahead!).
উত্তর: অন্যদের কথা জানি না, কিন্তু আমি কখনো জিনিয়াস খুঁজে বেড়াই না, কারণ আমি জানি জিনিয়াস খুঁজে বেড়াতে হয় না যারা পরিশ্রম করতে রাজি আছে তারাই জিনিয়াস! আমি আমার জীবনেএরকম অনেক জিনিয়াস পেয়েছি। আমি শুনে খুবই খুশি হলাম তুমি বাংলাদেশের জন্য সম্মান বয়ে আনার জন্য প্রস্তুতি নিচ্ছ। তোমার স্বপ্ন একদিন নিশ্চয়ই সত্যি হবে।

প্রশ্ন: জনাব আমার সালাম নিন(আস সালামু আলাইকুম)।আমার কাছে প্রযুক্তি নিভরতা তেমন একটা ভালো লাগেনা।কারণ বাহ্যিক দিক থেকে প্রযুক্তির ব্যবহার ভালো দেখালেও এর ক্ষতিকর দিকই বেশি।আমরা কি ধীরে ধীরে ধ্বংসের দিকে এগোচ্ছি না? সিমান্ত জয়(দশম শ্রেণী),হাতিখলা উচ্চ বিদ্যালয়,গফরগাঁও,ময়মনসিংহ।
উত্তর: এত আতঙ্কিত হওয়ার কিছু নেই. পৃথিবীতে অনেক সময়ে নতুন প্রযুক্তি এসেছে। অতীতে দেখা গেছে যখনই নতুন প্রযুক্তি এসেছে তখন কেউ না কেউ বলেছে এটা খুবই বিপদজনক । শেষ পর্যন্ত দেখা গেছে সেগুলো এমন কিছু বিপদজনক নয়। তবে একথা সত্যি যে প্রযুক্তির মাঝে ভালো প্রযুক্তি যে রকম আছে সে রকম খারাপ এবং অপ্রয়োজনীয় প্রযুক্তিও আছে। মানুষ নিজের বুদ্ধি বিবেচনা ব্যবহার করে ভালো প্রযুক্তি বেছে নেবে সেটা আমরা তো আশা করতেই পারি।

প্রশ্ন: প্রিয় স্যার, আপনি কি জানেন আপনি আমার প্রিয় লেখক?আমার খুউউউব আপনার সাথে দেখা করার ইচ্ছা। আমার গল্পের বই পড়তে ভীষণ ভালো লাগে।আর এই ভালোলাগার শুরুটা আপনার বই টুনটুনি ও ছোটাচ্চু থেকেই হয়েছে।২০১৮ সালে যখন আমি ক্লাস থ্রি তে, তখন ক্লাস চলাকালীন আমার এক বন্ধু ডেস্কে রেখে ওই বইটা পড়ছিল(কাজটা যদিও ঠিক না তবে তার মা প্রাইভেট-কোচিং-পড়াশোনা ছাড়া কিছু বোঝে না।কাজেই বেচারির মনে হয় দোষ নেই।)।আমি দেখতে পেয়ে বইটার খানিকটা পড়ি, আমার বইটা ভীষণ ভালো লাগে তাই লাইব্রেরি থেকে কিনে আনি।তারপর থেকে প্রতি মাসে একটা করে নতুন বই না হলে আমার চলে না।
যাই হোক, এটা বলার জন্য আমি আপনাকে লিখতে বসি নি।আমার আপনার কাছে একটা প্রশ্ন আছে।আমার বয়স এখন ১৩,আগস্টের ১২ তারিখে ১৪ হবে।আমি মাঝে মাঝেই উদ্ভট আচরণ করি,রেগে যাই।আবার হঠাৎই মন খারাপ হয়ে যায়।অল্প মন খারাপ না ভয়ংকর মন খারাপ।আমার ধারণা এটা হচ্ছে কারণ আমার বয়ঃসন্ধিকাল চলছে।আমার যখন এরকম হয় তখন আমি খুব খারাপ আচরণ করি (রেগে যাই,কান্নাকাটি করি)। তখন আমার মনে হয় নিজের ওপর নিয়ন্ত্রণ থাকে না।রাগব না ঠিক করেও ভয়ংকর রেগে চিৎকার করে উঠি।তারপর নিজেই হতবুদ্ধি হয়ে যাই যে আমি এটা কি করলাম।আমি একথাগুলো আম্মুকে বলার চেষ্টা করেছিলাম।আম্মু তেমন গুরুত্ব দেয় নি।উল্টো পরে আমাকে এগুলো নিয়ে বকাবকি করেছে। তার মতে আমি ‘ঢং’ করছি। আমার প্রশ্ন হলো অভিভাবকেরা বয়ঃসন্ধিকালের শারিরীক পরিবর্তনকে খানিকটা গুরুত্ব দিলেও মানসিক পরিবর্তনকে তেমন গুরুত্ব দেয় না কেন ???? (নাকি শুধু আমার ক্ষেত্রেই ব্যাপারটা এমন?)আমার মনে হয় এসময় তাদের সন্তানদেরকে একটু বোঝার চেষ্টা করা উচিত।
বিশাল বড় রচনা লিখে ফেললাম !! জানিনা উত্তর পাব কিনা।আপনাকে আমি শ্রদ্ধা করি।আপনার বই পড়ে প্রথম আমার মনে হয়েছে আমার নিজের দেশের জন্য কিছু করা উচিত।আমি আপনার লেখা পড়ে প্রথম মুক্তিযোদ্ধাদের প্রতি শ্রদ্ধা ও ভালোবাসা অনুভব করি।কাজেই আমার মনে হয় আপনার কাছে খানিকটা কৃতজ্ঞ হওয়া উচিত।সবশেষে আমার ভালোবাসা নিবেন। —আদিবা রাইসা চৌধুরী সপ্তম শেণি দিনাজপুর।
উত্তর: আহারে! আমার খুব কষ্ট হয় যখন আমি শুনি একটা ছোট বাচ্চাকে তার বাবা-মা গল্প বই পড়তে দিচ্ছে না শুধু যে বাচ্চাকে আনন্দ পেতে দিচ্ছে না তা নয়, সেই বাচ্চাটা যেন স্বাভাবিকভাবে বড় হতে পারে সেখানেও বাধা দিচ্ছে। হ্যাঁ, তুমি ঠিকই অনুমান করেছো এখন তোমার যে হঠাৎ হঠাৎ করে মন খারাপ হয়ে যায়, তুমি যে রেগে যাও, তার কারণ তুমি এখন বয়ঃসন্ধির ভেতর দিয়ে যাচ্ছে, তোমার শরীরে এমন হরমোনের প্রবাহ হচ্ছে যেটা আগে তোমার শরীরে ছিল না। সবাইকেই এভাবে যেতে হয়। তোমার ঘাবড়ানোর কোনো কারণ নেই, কারণ একসময় এই সময়টা পার হয়ে যাবে তখন আবার আবার আগের মত স্বাভাবিক জীবনে ফিরে যাবে। দেশের জন্য কিছু একটা করতে চাইছ জেনে আমি খুব খুশি হলাম। দোয়া করি তুমি যেন দেশের জন্য অনেক কিছু করতে পারো!

প্রশ্ন: স্যার আপনি কোথায়? আর কত অপেক্ষা করব? 🙂 ফিহা,নরসিংদী।
উত্তর: ধর ধর ধৈর্য ধর।

প্রশ্ন: আমাদের শোনার এবং দেখার ক্ষমতা সীমিত।২০ থেকে ২০ হাজার Hz কম্পাঙ্ক এর শব্দ এবং ৪০০ থেকে ৭০০ ন্যানো মিটার তরঙ্গ দৈর্ঘের আলো ছাড়া আমরা দেখতে পাই না,আর জ্বীন তোহ তৈরি করা হয়েছে আলো দিয়েই,তাহলে কি জ্বীন বিজ্ঞান দিয়ে প্রমান করা যাবে?(কোনো শক্তিশালী উচ্চ কিংবা নিম্ন তরঙ্গ দৈর্ঘ্য আলো রিসিপ্টর দিয়ে) সিয়াম মাহমুদ শাহজাদপুর,সিরাজগঞ্জ
উত্তর: ধর্ম দিয়ে বিজ্ঞান কিংবা বিজ্ঞান দিয়ে ধর্ম প্রমাণ করার চেষ্টা করো না এটা কখনো কাজ করবে না। কারণ বিজ্ঞান হচ্ছে পরীক্ষা নিরীক্ষা যুক্তি তর্কের বিষয় আর ধর্ম হচ্ছে বিশ্বাসের বিষয়।

প্রশ্ন: প্রিয় স্যার, আসসালামু আলাইকুম আমি আয়েশা। তুমি আমাকে নাম ধরে ডাকতে পার৷ আর আমি তোমাকে তুমি করে বলছি কারণ আমি তো তোমার থেকে মাত্র দুই দিনের ছোট। আমার জন্মদিন ২৫ শে ডিসেম্বর। তুমি আমার প্রিয় লেখক। তোমার অনেক বই পড়ে ফেলেছি। এর আগেও আমি এখানে লিখেছি, তোমার কোন উত্তর পাইনি। আমি তোমাকে প্রায় এক বছর আগে হাতে একটা চিঠি লিখেছিলাম, কিন্তু আমার কাছে তোমাকে চিঠি পাঠানোর ঠিকানা নেই, তাই পাঠাতে পারিনি। তোমাকে চিঠি পাঠানোর ঠিকানা দেবে প্লিজ। ও তোমাকে বলা হয়নি, আমি ভেন্টিকুয়েলিজম করতে পছন্দ করি। ইতি আয়েশা ইশতিয়াক (তিতলি বুড়ি), বয়স ৮ বছর ৬ মাস ২য় শ্রেণী, চুয়াডাঙ্গা
উত্তর: m.z.iqbal@mzi.rocks এখানে একটা ই -মেইল পাঠিয়ে জিজ্ঞেস কর, দিয়ে দেব।

প্রশ্ন: Assalamu alaikum. সৃজনশীল প্রশ্ন কিভাবে পড়ানো উচিত ক্লাসে? এই বিষয়ে কথা বলতে চাই।
উত্তর: বল। নিজের নাম ঠিকানা দিও।

প্রশ্ন: আপনি এবং কায়কোবাদ স্যার class 9-10 এর physics বইটাকে নষ্ট করেছেন। আগের বইটাই ভালো ছিল। আপনারা এই বইয় একটা সংঙ্গাও ঠিকমতো দেননি, সূত্রের প্রমাণ করেছেন গল্প বলে। এই ধরনের কাজ করার কোনো অধিকার কেউ আপনারদেরকে দেয়নি। আপনারা জানেন আপনাদের এই কাজের জন্য অনেক শিক্ষার্থী physics কে ভয় পাচ্ছে, কোচিং সেন্টারের ও গাইড বইয়ের উপর নির্ভরশীল হয়ে যাচ্ছে। আপনাদের মনে রাখা উচিত ছিল যে physics বই কোনো উপন্যাসের বই নয়। ভবিষ্যতে এ ধরনের কাজ করা থেকে বিরত থাকবেন। আপনার কাছে যদি কোনো সদুত্তর থাকে তাহলে আপনি আমার লেখাটা পুরোটা প্রকাশ করে উত্তর দিবেন। উত্তর না পেলে বুঝব আপনার কাছে এর কোনো সদুত্তর নেই। -জাকিয়া নূর চৌধুরী ১০ম শ্রেণি  Dhaka-1219
উত্তর: তোমার লেখাটা পুরোটা প্রকাশ করে দিচ্ছি। সমস্যা হচ্ছে, যারা পরীক্ষা নিয়ে মাথা না ঘামিয়ে পদার্থ বিজ্ঞান শিখতে চায় তারা কিন্তু এই বইটা খুব আগ্রহ নিয়ে পড়ে! কিছুদিন আগে ক্লাস সেভেনের একটা বাচ্চা আমাকে জানিয়েছে সে এই বইটা পড়ে পদার্থবিজ্ঞান শিখে ফেলেছে! তুমি কি পরীক্ষার কথা ভুলে গিয়ে পদার্থ বিজ্ঞান শেখার জন্য বইটা একটু পড়ে দেখবে?

প্রশ্ন: জিয়াদ হোসেন,খুলনা থেকে] আশা করি ভালো আছেন,স্যার।তরশু ssc পরীক্ষা দিতে যাচ্ছি।যতটা না পরীক্ষার চিন্তা তার থেকে পরিবারকে এ প্লাস এনে দেবার চিন্তা করতে হচ্ছে বেশী।হা হা হা।এনাদের মনোভাব তো পরিবর্তন করতে পারলামনা তাই নিজের আনন্দটা নিজেই খুঁজে নিয়ে উপভোগ করার পাশাাপাশি তাদের জন্য একটু দুঃচেষ্টা করেই না হয় যায়,কী বলেন।অন্যথায় বলবে,মাা বাবার অবাধ্য।তবু সবার নিষেধ অমান্য করে এখন আমি প্রতি মাসে কিশোর আলো,বিজ্ঞান চিন্তা ম্যাগাজিন পড়ি।এমনকি ইন্টারনেেটে বই পড়ার ও বই ধার করার ওয়েবসাইট পর্যন্ত খুঁজে নিয়েছি নিজে।এই লেখা কবে আপনি পড়বেন জানিনা,যখনই পড়বেন আমার জন্য দোয়া করবেন।যাইহোক,একটা প্রশ্ন আছে,যে সবুজ কালার ব্লাইন্ড সে গাছের পাতা কী রঙের দেখবে,ধূসর দেখবে,নাকি অন্য কোনো রঙের দেখবে,নাকি গাছের পাতাই দেখতে পাবেনা।ইমেইল করতে সাহসে কুলায়নি তাই এখানে লিখলাম।ভালো থাকবেন।
পুনশ্চ,আমার সায়েন্টিস মামা,আমার ডেঞ্জারাস মামী,সফদর আলীর মহা মহা আবিষ্কার এবং বুবুনের বাবা,বইগুলো কিছুদিন আগে পড়েছি। বইগুলো চমৎকার এবং একইসাথে ফাটাফাটি।
উত্তর: আমি খুব আশা করে আছি একদিন এইদেশে ছেলেমেয়েদের পরীক্ষা নিয়ে মাথা ঘামাতে হবে না। অবশ্যই আমি তোমার জন্য দোয়া করি করছি। ইন্টারনেটে তোমার এই প্রশ্নটার উত্তর খুব সুন্দর ভাবে দেওয়া আছে তুমি দেখে নেবে প্লিজ!

প্রশ্ন: স্যার আসসালামু আলাইকুম। আপনাকে সবসময় দেখি আমাদের মত আণ্ডাবাচ্চাদের creative কাজে সাপোর্ট করেন,অনেক সময় আমাদের বইয়ের মধ্য দিয়ে মেন্টালি সাপোর্টও দিয়ে থাকেন। তার জন্য আমি আপনার কাছে অনেক কৃতজ্ঞ🥺। আপনার বইয়ের অনেক জায়গায় আপনি আমাদের জন্য অনেক টিপস,facts এসব গুলায় রাখেন,সেগুলা আমরা কিন্তু ঠিকই বুঝতে পারি😌। আপনার লেখলেখির এত অসাধারণ ট্যালেন্ট,আপনি কি আপনার আণ্ডাবাচ্চা দের মাঝে দেখতে চাননা?🥺। অবশ্যই চান😊।এখন-আপনি-যদি-আমাদের-বা বিশেষ করে আমাকে,যে একটা fiction novel লেখার কাজ হাতে নিয়েছি (অলরেডি ১৬ পৃষ্ঠা লিখে ফেলেছি😁),আপনার কিছু writing tips দিবেন🥺? মানে কেমন করে আপনি এত সুন্দর plot twist দেন,সুন্দর বর্ণনা দেন এইসব? প্লিইইইইইইইইজ😭😭😭 ইতি, মাইশা, মিরপুর,ঢাকা।
উত্তর: তুমি ঠিকই বলেছ, আমি আন্ডা বাচ্চাদের ভেতরে নতুন নতুন লেখক চাই। তুমি লিখতে শুরু করেছো জেনে আমি খুব খুশি হলাম। দোয়া করি তুমি খুব সুন্দর এবং ফাটাফাটি একটা উপন্যাস যেন লিখতে পার। তুমি টিপস চেয়েছ, আসলে লেখালেখির সেরকম টিপস নেই। যদি থাকতো তাহলে সেই টিপস দেখেই শত শত লেখক বের হয়ে যেত! যেটা বলতে পারি সেটা হচ্ছে, অনেক অনেক অনেক বই পড়ো। দেখবে তুমি তাহলে লিখতে শিখে যাবে। আমি এখনও যখন খুব ভালো একটা বই পড়ি তখন কিছু একটা লিখতে ইচ্ছা করে। তুমি চেষ্টা করে দেখো।

প্রশ্ন: নামঃমারিয়াম ইসলাম ঠিকানাঃহাসনাবাদ,কেরানিগঞ্জ, ঢাকা স্যার,একজন জন্মান্ধ ব্যক্তি কি আমাদের মতই স্বাভাবিক সপ্ন দেখে?
উত্তর: আমি এই বিষয়ে এক্সপার্ট না। কিন্তু শুনেছি যারা জন্মান্ধ তারাও নাকি এক ধরনের ভিজুয়াল স্বপ্ন দেখতে পারে, তবে অবশ্যই সেগুলো অন্যদের মত স্বাভাবিক স্বপ্ন নয়।

প্রশ্ন: স্যার,একজন জন্মান্ধ ব্যক্তি কি আমাদের মতই সাভাবিকভাবে সপ্ন দেখতে পায়? স্যার আপনি বিজ্ঞানের বই কেন জানি খুবই কম লিখেছেন। আপনার কাছে বিজ্ঞানের প্রচুর বই এর জন্য অনুরোধ করছি। আসিফ, ঢাকা।
উত্তর: কি আশ্চর্য! পর পর দুইজন একই প্রশ্ন করেছ! কে বলেছে আমি বিজ্ঞানের বই বেশি লিখি নাই এখন পর্যন্ত পনেরোটা বিজ্ঞানের বই লিখেফেলেছি। এটা কি কম হলো? তুমি কি জানো আমি প্রত্যেক বছর কমপক্ষে একটা বিজ্ঞান কিংবা গণিতের বই লিখি।

প্রশ্ন: Sir, amar question ta hochhe jara chosma chara sob kichu japhsha dekhe tara ki sopno tao japsha dekhe naki clear dekhe ? Nafisa Hasnabad,Dhaka .
উত্তর: কি হলো, হঠাৎ করে সবার স্বপ্ন নিয়ে এত প্রশ্ন কেন? আমি চশমা ছাড়া ঝাপসা দেখি এবং আমি চশমা পরে ঘুমাই না কিন্তু তার পরে যথেষ্ট পরিষ্কার স্বপ্ন দেখি!

প্রশ্ন: বিটিএস ফ্যান হওয়া কি খারাপ? প্লিজ উত্তর দিবেন। ওমর ফারুক, মালিবাগ, ঢাকা
উত্তর: বিটিএস বিষয়টা কি? (সরি, ইন্টারনেট ঘেটে বিটিএস কি সেটা বের করার ধৈর্য নাই!)

প্রশ্ন: “চমৎকার! উত্তরটা জানলে দেওয়ার চেষ্টা করা হবে!” প্রশ্ন সেকশনে যখন আবোলতাবোল কিছু লিখে নিচের বাটনে ক্লিক করি, এই লাইন দুটি দেখে কেমন যেন অস্বস্তি লাগে! মনে হয়, যে না আমার প্রশ্ন, তার আবার উত্তর…! স্যার কী না কী মনে করেন প্রশ্নের ছিড়ি দেখে! পূর্ণতা।
উত্তর: না, আমি প্রশ্নের ছিরি দেখে কিছুই মনে করি না। সব প্রশ্নই আমার কাছে সমান। তবে যারা প্রশ্ন করার নামে আমাকে খুব খারাপভাবে গালি দেয় সেটা অন্য ব্যাপার।

প্রশ্ন: আপনি চলে গেলেন, স্যার! সত্যি সত্যি চলে গেলেন? আর আসবেন না? আসবেন না আমাদের কাছে? কতদিন আপনার কথাগুলি আর পড়ি না।বই তো একটা বানানো কাহিনী। ওতে কি আর আপনার কোনো কথা ওইভাবে পাওয়া যায়? আহা,অনেক যে দূরে চলে গেলেন! অনেক অনেক দূরে! পূর্ণতা।
উত্তর: যাই নাই! যাই নাই! যাই নাই! আমি আছি, বেশি দূরে যাই নাই! আমার সময় নিয়ে কী যে টানাটানি সেটা যদি একবার আমি তোমাদের বুঝাতে পারতাম!

প্রশ্ন: প্রিয় স্যার, আমি রিফাত মুন্সীগঞ্জ থেকে বলছি।।। আমার মহাকাশ নিয়ে খুব আগ্রহ ।।আমার ইচ্ছা বড় হয়ে আমি মহাকাশ বিজ্ঞানী হব।। আমার জন্য দোয়া করবেন…আমি যেন মহাকাশ বিজ্ঞানী হতে পারি।।
উত্তর: আমি দোয়া করছি তুমি যেন বড় হয়ে মহাকাশ বিজ্ঞানী হতে পারো!

প্রশ্ন: স্যার, আপনি আমাকে অটোগ্রাফ দেননি। Please Sir, give me your autograph…Tareq Al Mamun- From Jaintapur,SylhetClass 10
উত্তর: এইবার শুধু প্রশ্নের উত্তর। আরেকটু গুছিয়ে নেই তারপর অটোগ্রাফ দেব, ঠিক আছে?

প্রশ্ন: স্যার আমি আপনার সাথে মেইলে যোগাযোগ করতে চাই মেইল আইডি টা কি পেতে পারি? ইসমাম জাহান মিরপুর গার্লস আইডিয়াল ল্যাবরেটরি ইন্ট্রিটিউট একাদ্বশ শ্রেণী
উত্তর: এইযে আমার ইমেইল অ্যাড্রেস: m.zafar.iqbal@mzi.rocks

প্রশ্ন: আমার নাম মুশফিকুর রহমান। কাজিরপাগলা এ টি ইন্সটিটিউশন এর ৮ম শ্রেনীর একজন ছাত্র । আমি মুন্সীগঞ্জ এ থাকি । আমি “নিউরনে অনুরণন” বই এর ১০১ নং সমস্যার সমাধান পাঠাতে চাই।।।
উত্তর: পাঠাও! এটা খুবই বিখ্যাত সমস্যা!

প্রশ্ন: আসসালামু আলাইকুম। কেমন আছেন স্যার? সাবা, ঢাকা থেকে
উত্তর: ওয়ালাইকুম সালাম। আমি ভালো আছি, থ্যাঙ্কু। তুমি কেমন আছ?

প্রশ্ন: বিজ্ঞানী এলিট সিলভারের চাঞ্চল্যকর দাবিঃ মানুষ পৃথিবীর জীব নয় … … … স্যার বিবর্তন এর চেয়ে এই থিওরি কি বেশি উপযুক্ত নয় মানুষের জন্য…???? ড. এলিট সিলভা র HUMAN ARE NOT FROM EARTH. বইটি পড়তে হবে। একটি ভিডিও লিংক দিলাম। https://youtu.be/V-oLpOhzOvk ফয়সাল আহমেদ
উত্তর: এরকম বইয়ের কোন অভাব নেই। তুমি কয়টা চাও? এজন্য বিজ্ঞান সম্পর্কে জানতে হলে বিজ্ঞানের বই পড়তে হয়, এরকম বই পড়ে বিজ্ঞান জানা যায় না! আমি সেই ছোটবেলা থেকে এরকম বই পড়ে আসছি। এরকম বই পৃথিবীর কোন কাজে আসে কিনা আমি জানিনা তবে যারা লিখে তারা অনেক টাকা কামাই করে সেটা আমি জানি। তুমি কি জানো একজন খাঁটি বিজ্ঞানী কিন্তু কখনোই তার তত্ত্ব দিয়ে কোনো টাকা উপার্জন করতে পারে না।

প্রশ্ন: স্যার আসালামু আলাইকুম। স্যার আপনাকে অনেক অনেক অনেক বেশি ভালোবাসি। দিন দিন এর পরিমাণ বেড়েই যাচ্ছে। আপনার কাছে একটাই অনুরোধ আমাদের জন্য কখন লেখা বন্ধ করবেন না তাহলে আমরা সব ক্ষুদে পাঠক অক্সিজেনের অভাবে মারা যাব। আর খবরদার টুনটুনি সিরিজ লেখা বন্ধ করবেন না, প্রতি বছর বইমেলায় গিয়ে সবার আগে টুনটুনি সিরিজের বই খুঁজতে থাকি। আমাদের ভালবাসাটা বুঝতে পারছেন? তাহমিনা সুলতানা( ঐ যে উপলব্ধি পরিবারের সদস্য) অষ্টম শ্রেনি, চট্টগ্রাম।
উত্তর: না, দুশ্চিন্তা করো না। তোমাদের জন্য আমার লেখালেখি বন্ধ করার কোন পরিকল্পনা নাই, তবে ভালো লিখতে হলে সময় দরকার, আমার কপাল খারাপ সেরকম কোন সময় নাই! এই সমস্যা কেমন করে মেটাবো? তোমাদের অক্সিজেন শুধু আমাকে কেন সাপ্লাই দিতে হচ্ছে, আরও কত বড় বড় অক্সিজেনের সাপ্লায়ার আছে, তারা কি দোষ করল?

প্রশ্ন: আসসালামু আলাইকুম স্যার! আশা করি ভালো আছেন। আমি বই নিয়ে বসে আছি কিন্তু পড়তে ইচ্ছা করছে না। আবার না পড়েও থাকতে পারছি না কারণ কালকে পরীক্ষা। কী জ্বালা!
উত্তর: ঠিকই বলেছ, কি জ্বালা! যাইহোক এতদিনে নিশ্চয়ই পরীক্ষা শেষ হয়ে গেছে, জ্বালা আর নেই!

প্রশ্ন: আপনি টুনটুনি ও ছোটাচ্চু বই এ শান্ত র নাম শান্ত দিয়েছেন কেন? ওর নাম রাখা উচিত ছিল দুরন্ত। আমি পাঠ্যবই এ এই নাম পড়েছি। দুরন্ত নাম তো হয়। রুফাইদা সাঈদ, সোনার গাঁ, নারায়ণগঞ্জ।
উত্তর: তুমি ঠিকই বলেছ আমি একেবারেই ভালো নাম দিতে পারি না। আমার সব বইয়ের নাম সব সময় উল্টা-পাল্টা হয়!

প্রশ্ন: স্যার, আদাব। আপনার শরীর ভালো তো? আমি আত্রলিতা, যশোর থেকে। স্যার, বিটিএস নিয়ে আমাদের দেশের মাতামাতি আপনি সাপোর্ট করেন কি? অজান্তে কোনো বেয়াদবি হয়ে থাকলে আমি সে জন্যে ক্ষমাপ্রার্থী। স্যার, ভালো থাকবেন। নিজের শরীরের যত্ন নিবেন।
উত্তর: সমস্যা হচ্ছে, আমি বিটিএস কি জানিনা! আমি যে বিটিএস (BTS: Base Transceiver Station) জানি তুমি নিশ্চয়ই সেই বিটিএসের কথা বলছো না! ইন্টারনেট ঘেঁটে বিটিএস কি সেটা বের করার ইচ্ছাও করছে না।

প্রশ্ন: Assalamualaikum. Sir, can we prove the four colour theorem by saying that 5 2d objects cannot join with one another (Which is actually true). When any 2d object can join with another 3 on a 2d surface, why would we need more than 4 colours to differentiate them? Rayet 3079 Mirzapur Cadet College Mirzapur, Tangail, Bangladesh
উত্তর: আমার একটু চিন্তা করে উত্তর দিতে হবে, কারণ পাঁচটি দ্বিমাত্রিক প্লেন যোগ দেওয়া আর four colour theorem কী একই ব্যাপার না? পাঁচটি দ্বিমাত্রিক প্লেন সবার সাথে সবাই যুক্ত হতে পারে না তার প্রমানটি কীভাবে করা হয়েছে জানা নেই। তবে তুমি যে এইধরনের একটা সমস্যা নিয়ে মাথা ঘামাচ্ছ ট্যাট্যাই আমি মহা খুশি।

প্রশ্ন: I am Shoiboliny Bhattacharjee. Currently I am 14.5 years old. I live in Chattogram, Bangladesh. So, I read in class 9 at St. Scholastica’s Girls School and College. I am in Science group. We had our evaluation test about 1 month ago. The result was not so pleasing but I am satisfied by that. Atleast now I know that I am good enough. About my future goal, I want to have GPA 5 in SSC, then join a college out of Chattogram, then after finishing HSC, I will join BMA or Medical College. But if you ask me what means to me more, I will say BMA. It’s Army training center. I want to be an Army from bottom of my heart. Not BTS army, those who protects country. I really want to fight for my country. As for my friends, I don’t have any. I mean I do have many to joke with but is that called friendship? NOPE. So I am alone but not sad. Because I know enjoying your own company is best for being happy. … … … … It has been tough because I am a teen, and in our age we need our friends help most. In this place I don’t even have friends leave about the help. Sir what should I do now?
উত্তর: আমার দেখে খুবই ভালো লাগলো যে তুমি ভবিষ্যতে কি করবে সেটা নিয়ে তোমার একটা সুনির্দিষ্ট পরিকল্পনা আছে। আমি খুব বেশি মানুষের ভেতরে এটা দেখিনি। দোয়া করি তুমি যেন তোমার স্বপ্নের মতো তোমার জীবনটাকে গড়ে তুলতে পারো। দেশের জন্য কাজ করার এতো তীব্র ইচ্ছা দেখে আমি খুব খুশি হয়েছি। আর বন্ধুত্বের ব্যাপারটা নিয়ে তোমার ব্যাখ্যাটা আমি ঠিক বুঝতে পারিনি। একজনের মাত্র একজনই বন্ধু থাকে সেটা কি আসলেই সত্যি? আমার ধারণা আসল ব্যাপারটি যতটুকু জটিল তুমি এটাকে তার থেকে বেশি জটিল ভাবে দেখছো। আমি তো ধরেই নিয়েছি তোমাদের বয়সে একসঙ্গে অনেক ছেলেমেয়ে থাকে এবং তারা সবাই সবার বন্ধু। একজনের শুধুমাত্র একজন বন্ধু থাকবে সেটা তো আসলে আমি কখনো সেভাবে দেখিনি। কারো সাথে একটু বেশি হতে পারে আবার কারো সাথে একটু কম কিন্তু সবাই তো সবার বন্ধু। তাই না? জীবনটাকে এতো সিরিয়াসলি না নিয়ে আরেকটু সহজ ভাবে নিলে কেমন হয়?
তবে অন্য একটা বিষয় স্বীকার করতেই হবে। তুমি তোমার বক্তব্যটা খুবই সুন্দর করে লিখেছ। এত গুছিয়ে যে লেখা যায় তাও আবার নিজের ভাষায় নয়, ইংরেজিতে, সেটা দেখে আমি মুগ্ধ হয়েছি!

প্রশ্ন: আসসলামু আলাইকুম স্যার। আশা করি আপনি ভালো আছেন। আমিও ভালো আছি। আপনার কাছে আমার কিছু প্রশ্ন আছে। ১. টুনটুনি ও ছোটাচ্চুর বইয়ে টুনির আসল নাম কী? ২. টুনিদের বাসায় মোট কতজন সদস্য? ৩. এক ডজন এক জন বইয়ে খা বক্কিলারে খা – এই শব্দটি আপনি কোথা থেকে পেলেন? (শব্দটা খুবই মজার) আজ আর প্রশ্ন করবো না, কিন্তু আমার মাথার কাজ করবে না কেননা আমার মাথায় অনেক প্রশ্ন জমে আছে। বাকি প্রশ্নগুলোও করে ফেলব। ভালো থাকবেন। আমার জন্য দোয়া করবেন। আর হ্যাঁ আমার জন্য একটা অটোগ্রাফ দেবেন প্লিজ? ‘ মেধু ‘(আমার আদরের নাম যেটা আমার মা-বাবা দিয়েছে) নামটা লিখে… নাম- অদিতি মেধা শ্রেণি- পঞ্চম ঢাকা
উত্তর: আমি আসলে টুনির ভালো নাম এখনো দেই নাই, এখন পর্যন্ত দরকারও ড়ে নাই। যদি কখনো দরকার পড়ে তখন দেখা যাবে। তবে মনে হয়না দরকার পড়বে, সে তার ডাকনাম দিয়েই জীবন কাটিয়ে দিতে পারবে। নিজের বাসায় কতজন আছে সেটা কেউ জানে না, আমিও জানিনা। যখন লেখালেখি শুরু করেছি তখন ভাবি নাই এত দিন লিখতে হবে তাহলে আমি এই বিষয় গুলো আরও গুছিয়ে ঠিক করে নিতাম। যখন ছোট ছিলাম তখন দেখতাম সাপুড়েরা বলতো, খা খা খা বক্কিলারে খা, তার মানে যারা কৃপণ, যারা টাকা দিতে চায়না, সাপ যেন তাদেরকে কামড় দেয়! এবার আমি শুধু প্রশ্নের উত্তর দিচ্ছি অটোগ্রাফ দেওয়ার জন্য আমার সিস্টেম টা একটু রেডি করতে হবে। এখনো রেডি হয় নাই একটু ধৈর্য ধরো প্লিজ।

প্রশ্ন: শুভ অপরাহ্ন স্যার, শ্রদ্ধা ও ভালোবাসা নেবেন। আপনি সবসময় সুস্থ থাকুন। ভালো থাকুন। যে আনন্দ আপনি আমাদের শৈশব, কৈশোরে দিয়েছেন, সৃষ্টিকর্তা সহস্রগুণে তা আপনাকে ফিরিয়ে দিক। আপনার পুরো জীবন ভয়াবহ আনন্দে কাটুক! আমরা এখনো আপনার লেখা পড়ে আমাদের শৈশবে একটা উঁকি দিয়ে আসি। পুনশ্চঃ আপনার প্রতি ভালোবাসা, ভালোবাসা আর ভালোবাসা। অনেক অনেক কৃতজ্ঞতা। সাথে প্রচুর শুভকামনা! [রিমঝিম। কাজীহাটা, রাজশাহী]
উত্তর: তুমি তোমার শৈশব এবং কৈশোর এর কথা বলেছো, তাহলে তুমি কি এগুলি পার হয়ে এসেছ? তারপরও আমার কথা তোমার মনে আছে সেজন্য তোমাকে অনেক ধন্যবাদ। আমার চিন্তা করে খুবই ভালো লাগে যে তোমাদের মত কারো কারো শৈশবে আমি একটু আনন্দ দিতে পেরেছি। ভালো থেকো। তোমার নামটা খুব সুন্দর, রিমঝিম!

প্রশ্ন: (জিয়াদ হোসেন,রূপসা, খুলনা)স্যার,কেমন আছেন?ঈদ-উল-ফিতরের শুভেচ্ছা:”ঈদ মোবারক”।
উত্তর: ঈদুল আযহাতে তোমার ঈদুল ফিতরের শুভেচ্ছার উত্তর দিচ্ছি! তোমাকে ঈদের শুভেচ্ছা!

প্রশ্ন: স্যাাাাাাাাাাাাাাাাাাাাাাারররররররর…..আপনি কোথায় গেলেন? আর কী আসবেন না এই সাইটে?😐…কবে আসবেন? ফিহা মনোহরদী,নরসিংদী।
উত্তর: এই যে এসেছি! খুশি?

প্রশ্ন: আমার কয়েকটা প্রশ্ন ১। আহা টুনটুনি উহু ছোটাচ্চুর পর নতুন বইয়ের নাম কি হবে ? ২। টুনটুনি ও ছোটাচ্চু কি কোনো আসল চরিত্র থেকে অনুপ্রাণিত নাকি পুরোটাই কাল্পনিক । ৩। যেসব বইয়ে চরিত্রের সংখ্যা বেশি (টুনটুনি ও ছোটাচ্চু ) সেসব বইয়ের দ্বিতীয় পার্ট লেখার সময় কি আপনার সব চরিত্রের নাম ও বৈশিষ্ট্য মনে থাকে নাকি একবার আগের বইটা পড়ে নিতে হয়? * আপনার সাথে দেখা করার আমার খুব ইচ্ছা করে । আপনার সাথে দেখা করার জন্য গণিত অলিম্পিয়াড , ফিজিক্স অলিম্পিয়াড এর মত অনেক প্রতিযোগিতায় অংশগ্রহণ করি কিন্তু পরের লেভেলগুলোতে টিকতে পারি না আফসোস । যদি একটা গল্প পড়া বা লেখা প্রতিযোগিতা হয় এবং সেটির আয়োজন যদি আপনি করেন তাহলে আমি অবশ্যই প্রতিযোগিতায় ভালো পজিশন করার চেষ্টা করব। তাহলে পুরষ্কার নেওয়ার সময় তো আপনার সাথে দেখা করতে পারব* সাদিয়া মাহজাবীন রাজশাহী
উত্তর: এখনো ঠিক করি নাই, আগে আগে বইটা লেখা হোক তারপরে দেখা যাবে। বইয়ের সমস্ত চরিত্র কাল্পনিক, সবই বানিয়ে বানিয়ে লেখা! নতুন একটা বই লেখার সময় আসলে পুরনো বই গুলি মনে হয় দেখা দরকার কিন্তু দেখা হয়না। সেজন্য আমার বইয়ের চরিত্রের নাম পাল্টে যায়, উল্টা পাল্টা হয়ে যায়! নিশ্চয়ই তোমার সাথে একদিন দেখা হবে। এখন যেহেতু করোনা কমে আসছে, ভয়ও কমে আসছে তাই আমি বিভিন্ন জায়গায় যাচ্ছি। কোনো একটা অনুষ্ঠানে তোমার সাথে দেখা হয়ে যাবে একদিন।

প্রশ্ন: Sir, এভাবে অল্প কয়েকজনের উত্তর দিয়ে উধাও হয়ে গেলেন! এটা কিন্তু ঠিক না…খুব রাগ হয়েছে আপনার উপর! আরি! আরি! আরি! -সরোজ সিংহ (কুমিল্লা)
উত্তর: আমি আসলে উধাও হয়ে গেছি কথাটা পুরোপুরি ঠিক না। আমি আসলে সময় করতে পারছিলাম না, বুঝতেই পারছ এরকম প্রশ্নের উত্তর দিতে আমার অনেক সময় লেগে যায় এতো সময় পাচ্ছিলাম না।

প্রশ্ন: আমি গল্প লিখতে ও গল্পের বই পড়তে পছন্দ করি বিশেষ করে “ভ্রমণ কাহিনী “। ইনশাআল্লাহ কোনোদিন বড় লেখিকা হতে পারলে আপনার সাথে দেখা করতে পারব । আমার ভ্রমণ কাহিনী একবার প্রথম আলোতে প্রকাশিত হয়েছিল । সাদিয়া মাহজাবীন, রাজশাহী ।
উত্তর: শুনে খুবই খুশি হলাম তোমার লেখা প্রথম আলোতে ছাপা হয়েছে। লিখতে থাকো, দেখবে লিখতে লিখতে একদিন বড় লেখক হয়ে যাবে। ভ্রমন কাহিনী লেখার জন্য ভ্রমণ করতে হয়, দোয়া করি তুমি যেন পৃথিবীর অনেক জায়গা ভ্রমন করতে পার। আমার ভ্রমণ কাহিনী লিখতে কোন ভ্রমণ করতে হয় না আমি মহাকাশে কিংবা ভিনগ্রহে না গিয়েই মহাকাশ ভ্রমণের কাহিনী লিখে ফেলি!

প্রশ্ন: শ্রদ্ধেয় স্যার,আপনি কেমন আছেন ? আমি একদমই ভালো নেই।কেন ভালো নেই সেই কারণ টাই নিচে লেখা আছে।দয়া করে সবটুকু লেখা পড়বেন।এই ট্যাবে বাংলা টাইপ করা অনেক কষ্টের কাজ।তাই,সবটা লেখা মনযোগ দিয়ে পড়বেন।আর ভালো থাকবেন।
আচ্ছা বলেন তো,বাংলা ও সমাজের সৃজনশীল প্রশ্নের উত্তর আর বিজ্ঞানের সৃজনশীল প্রশ্নের উত্তর কি এক ধরণের হবে ? বাংলা আর সমাজে আমি চাইলে বানিয়ে বানিয়ে অনেক লাইন যোগ করতে পারব।কিন্তু বিজ্ঞানে কি আমি তা পাবর ? পারব না।বিজ্ঞানে লেখার জন্য আমার সঠিক তথ্য জানতে হবে।বইয়ে যে তথ্য আছে আমি তো তাই লেখব পরীক্ষার খাতায়।আর বইয়ে এতো বেশি তথ্য দেওয়াও থাকে না।তাহলে এবার বলেন যে,আমি হুবুহু বইয়ের মতো না হলেও প্রায় একই রকম লিখে দিয়ে আসছি।তাহলে আমাকে ৪ এর মধ্যে ৩ আর ২ এর মধ্যে ১ দিলো কেন ? এই জন্য আমার মোট পরীক্ষার নাম্বার অনেক কমে গিয়েছে।আর আমি অন্যর প্রশ্নগুলাতেও একটু খারাপ করেছি।তাই এই ২ নাম্বার দিয়ে দিতো তাহলে মোটামোটি একটা নাম্বার হয়ে যেতো।এ জন্যে আমার মন অনেক খারাপ।
রিনি,সিলেট।(আমি বুঝতে পারছিলাম না যে এই দুঃখ কার সাথে শেয়ার করব।তাই এখানে লিখে দিয়েছি।🙂) Bye…..
উত্তর: দুঃখের কাহিনী ঘটার পর অনেকদিন কেটে গেছে তাই আশা করছি তোমার দুঃখটা এতদিনে কমে এসেছে। তুমি যেহেতু আমাকে বলেছ সে কারণেও দুঃখটা কমে আসার কথা। তবে লেখাপড়া নিয়ে দুঃখটা নিয়ে বেশি মাথা ঘামিও না। এর পরেরবার যখন ভালো পরীক্ষা দেবে তখন এমনিতেই মনটা ভাল হয়ে যাবে।

প্রশ্ন: একটা জায়গা খুঁজে দিবেন প্লিজ যেখানে আমি শান্তিমতো একটা অন্যায় কাজ করতে পারব। … …
রাস্তায় বের হলে আগে অভ্যাস ছিল ললুপ দৃষ্টি আর এখন সাথে যোগ হয়েছে স্পর্শ। অবিশ্বাস্য লাগে আমার কাছে যে আমি অনেক কষ্টেও গুনতে পারি না সর্বমোট আমি রাস্তায় কতবার এমন পরিস্থিতিতে পরেছি। অপ্রাসঙ্গিক সব কথা মিলিয়ে জগাখিচুরি বানিয়েছি!! আচ্ছা মোবাইল রাখি। বকা খাব নাহয়। না না আর কয়টা কথা আছে। যেজন্য এত কিছু লিখলাম।
আমার দুনিয়াটা যে দিন দিন টুকরো টুকরো হয়ে ভেংগে পরছে এর দায় কে নিবে? আপনি নিবেন? আপনার প্রজন্ম নিবে? হম্বিতম্বি করা এদেশের রাজনৈতিক দল গুলো নিবে? আমি digital বাংলাদেশ নিয়ে কি করব স্যার পদ্মা সেতু দিয়ে কি হবে আমার যেখানে আমার সম্মানই নাই! … … … … এখন বলতে পারেন মেয়েদের শিক্ষার হার বেড়েছে। তারা অনেক ক্ষেত্রে ছেলেদের সাথে পাল্লা দিচ্ছে। আরে এত ক্যারিয়ার দিয়ে হবে কি!!!! আমি অশিক্ষিত ভাবে ঘরসংসার করতে রাজি আছি যদি আমার প্রাপ্য সম্মান টুকু আমাকে দেওয়া হয়। বাইরের জগতে বড় বড় লেকচার দিয়ে ঘরে এসে বাথরুমের দরজা আটকে চোখের পানি ফেলার মতো জীবন আমি চাই না! আপনারা এই ”স্বাধীন” দেশকে নিয়ে ৫০ বছর experiment করে অনেক উন্নতি ঘটিয়েছেন কিন্তু সেগুলো দিয়ে আমার মত মেয়েদের কি হবে বলতে পারবেন? আমরা শুধু রাস্তায় না, আমাদের স্যার দের মাধ্যমেও মানসিকভাবে নির্যাতিত হই (কি বোঝাতে চেয়েছি বুঝেছেন নিশ্চয়ই) এগুলো কে ঠিক করবে? আমরা? আর আমরা ততদিনে যতটুকু সম্মান হারাব তা কে ফিরিয়ে দিবে স্যার????
নাম দিব না। ছেলে হলে বুক ফুলিয়ে দিতাম। আমার আপনার উপর রাগ নেই স্যার। আপনাকে অনেক শ্রদ্ধা করি। বেয়াদবির জন্য সরি।
উত্তর: তোমার লেখাটা পড়তে গিয়ে আমার বুকটা ভেঙ্গে গেছে। আমি অনেকদিন কারো ভেতর এমন তীব্র ক্ষোভ আমি দেখিনি। তুমি গুছিয়ে খোলাখুলি সবকিছু লিখেছ, আমি কেটেছেটে অল্প একটু এখানে রেখেছি। অন্য অনেকের নিশ্চয়ই একই ধরনের অভিজ্ঞতা হয়েছে কিংবা হচ্ছে, কিন্তু তারা এত স্পষ্ট ভাবে এভাবে গুছিয়ে লিখেনি। আমি জানিনা আমি তোমাকে কিভাবে সান্তনা দেব, কিংবা সান্ত্বনা দেওয়া সম্ভব কিনা। এটা কি দেশের সমস্যা নাকি সমাজের সমস্যা নাকি পৃথিবীর সমস্যা নাকি পুরুষ মানুষের সমস্যা সেটাও আমি ভালো করে জানিনা। প্রতিদিন খবরের কাগজে এই ধরনের অসংখ্য খবর দেখি, খবরের কাগজে আসে না এরকম খবর নিশ্চয়ই আরো হাজার গুণ বেশি। দোয়া করি তোমরা যখন বড় হয়ে এই দেশের দায়িত্ব নেবে তখন এই সমস্যাগুলির সমাধান করবে। তুমি ঠিকই বলেছ আমরা কিংবা আমাদের প্রজন্ম আসলে এটা করতে পারে নাই, পরের প্রজন্ম হয়তো পারবে। আমার পরিচিত যেসব মেয়েরা আছে তারা সবাই আমাকে এই কথাগুলো বলেছে, আমি জানি এটা আমাদের দেশের একটা অনেক বড় সমস্যা। পৃথিবীর খুব অল্প কয়টি দেশ আছে যারা তাদের মানুষকে বোঝাতে পেরেছে যে মেয়েদের সম্মান করতে হয়। আমি খুবই দুঃখিত।

প্রশ্ন: Sir, I am a big big fan of your science fiction stories. I will be very much greatfull if I could meet you. I read in class 6. Aparajita Bornomala7C, 11/A, Daulat Lotus Tower, 3rd Lane, Gopibagh, Dhaka.
উত্তর: এখন যেহেতু করোনা কমে আসছে কিংবা মানুষজন করোনাকে কম ভয় পাচ্ছে, তাই আবার আমরা সবাই স্বাভাবিকভাবে হতে শুরু করছি। আমিও অনেক জায়গায় যাচ্ছি, আমি নিশ্চিত সেরকম কোন একটা জায়গায় তোমার সাথে আমার একদিন দেখা হয়ে যাবে।

প্রশ্ন: স‍্যার ভালো আছেন? আসসালামু আলাইকুম। আপনাকে একটা সত্যি কথা বলব? স‍্যার আপনি কি জানেন আপনি কত ভাগ‍্যবান!!! অনেকেই অন‍্যদের সম্মান পায় ( কখনো কখনো প্রকৃত কিংবা জোর খাটিয়ে!!!*বিষয়টা আমি লক্ষ করেছি আমাদের একজন স‍্যারের মাঝে। স‍্যারকে দেখলে সবাই দাড়াবে,সালাম দেবে এটাই স্বাভাবিক। কিন্তু অন‍্য কাজে ব‍্যস্ত থাকলে যদি সালাম দিতে বা দাড়াতে একটু দেরি হয় তাহলে বকাবকি রাগারাগি করা তো জোর করে সম্মান আদায় করাই!) কিন্তু বিশুদ্ধ ভালোবাসা খুব কম মানুষই পায়! এজন‍্য সত্যি বলতে আপনাকে এদিক দিয়ে আমার হিংসেই হয়। আমি বড় হয়ে এমন কিছু করতে চাই যাতে সবাই আমাকে ভালোবাসে, যাতে সম্মান জোর করেআদায় করে নিতে না হয়! ভালোবাসা থেকেই যেন সবাই সম্মান করে! আমি আসলেই এটা ভয় পাই যে সবাই বাইরে আমাকে সম্মান করবে আর ভেতরে ভেতরে গালি দিবে!!!! কি ভয়ংকর !!!! কি হবো জানি না কিন্তু সবার নির্মল ভালোবাসা পেতে চাই। আমার ভালোবাসার মতন। আমি যেভাবে কিছু মানুষকে ভালোবাসি ঠিক সেইভাবে ( যেমন আপনাকে…..) বাঁধন
উত্তর: হ্যাঁ আমি জানি, আমি কত ভাগ্যবান। আমি এটাও জানি একই সাথে অনেক মানুষ আছে যারা আমাকে এত অপছন্দ করে যে আমাকে জানে মেরে ফেলতে পারলেই তাদের আনন্দ। তার সাথে সাথে আমি এটাও জানি এই দেশের অনেক বাচ্চাকাচ্চারা শিশু-কিশোর-কিশোরীরা আমাকে ভালবাসে, সেজন্য আমি সবসময় ভাবি কেমন করে তাদের ভালোবাসাটা তাদেরকে ফিরিয়ে দেব। তারা আমাকে কেন এত ভালোবাস সেটাও সবসময় বুঝতে পারিনা। আমি কয়টা বই লেখা ছাড়া আরো বেশি কিছু করিনি, মনে হয় আরো কিছু করা দরকার ছিল।

প্রশ্ন: স্যার, আপনি প্লিজ টুনটুনি ও ছোটাচ্চু এর আরও সিরিজ লিখেন। আফিয়া উত্তরা, ঢাকা
উত্তর: হ্যাঁ আমি আরও বই লিখব। এখন মনে হচ্ছে সারা জীবন ধরেই টুনটুনির বই লিখতে হবে। কোন একদিন টুনটুনির বই এর চরিত্র চরিত্র গুলি বই থেকে বের হয়ে আমার উপরে না হামলা করে, সেটা নিয়ে দুশ্চিন্তায় আছি।

প্রশ্ন: স‌্যার আসসালামু আলাইকুম । আমার নাম রাইহা । ‌আমি আপনার ‌অনেক বড় ভক্ত । আপনার টুনটু‌নি ও ছােটাচ্চু সি‌রিজ আমার এত্ত প্রিয় যে প্রতি বছর আ‌মি চিন্তায় থা‌কি এই বু‌ঝি আপ‌নি টুনটু‌নি ও ছোটাচ্চু লিখা বন্ধ ক‌রে দি‌লেন । প্রতি বছর বই‌মেলায় গি‌য়ে আশায় থা‌কি আপনার সা‌থে দেখা হ‌বে । কিন্তু দেখা হয়‌নি । আমার খুব শখ আপনার একটা অ‌টোগ্রা‌ফ নিব । প্লি‌জ , আমা‌কে একটা ‌অটোগ্রাফ দে‌বেন । আর আরও বে‌শি বে‌শি ক‌রে বই লিখ‌বেন ।Raiha Tasmia, Narayanganj
উত্তর: আমার এই প্রশ্নের উত্তর গুলো বেশিরভাগই দিতে হচ্ছে টুনটুনি আর ছোটাচ্চুকে নিয়ে। তোমাদের মতন বাচ্চারা কেন এটা পছন্দ করে সেটাও আমি বুঝতে পারি না। যাই হোক যেহেতু আটকা পড়ে গেছি মনে হচ্ছে আমাকে সারাজীবন ধরেই টুনটুনি আর ছোটাচ্চুকে নিয়ে লিখতে হবে। একটা বিষয় লক্ষ্য করেছো এত দিন হয়ে গেল তারপরেও কিন্তু কোন চরিত্রের বয়স বাড়ে নাই। মনে হয় বাড়বেও না। অটোগ্রাফ দেওয়ার সিস্টেম টা এখনো রেডি করতে পারি নাই একটু অপেক্ষা করো প্লিজ।

প্রশ্ন: ও দাদু , দুইটা প্রশ্ন করতে চাই। করবো কি ? আচ্ছা করি ।তুমি কেমন আছো…? তোমার ছেলেমেয়েরা কত বড় হয়েছে এবং তারা এখন কি করে..? কিয়াস, নবম, নীলফামারী।
উত্তর: দুইজন ছেলে মেয়ে আমার। তারা বড় হয়ে গেছে, কেউ বিশ্ববিদ্যালয়ের পড়ায় কেউ বিশ্ববিদ্যালয়ে গবেষণা করে। এখন আমার নাতি নাতনিও আছে। হ্যাঁ, আমি খুবই ভালো আছি, থ্যাঙ্কু।

প্রশ্ন: ভগ্নাংশের যোগ ও বিয়োগ করার সময় হরের লসাগু করে সমাধান করি, গসাগু করে সমাধান করা যায় কিনা জানতে চাই, রনজিৎ রায় , জলঢাকা , নিলফামারি
উত্তর: না যায় না। কারণ লসাগু আর গসাগু একেবারেই ভিন্ন দুটো ব্যাপার।

প্রশ্ন: স্যার আমরা যখন আপনার কাছে এই ওয়েবসাইটে কিছু লেখা পাঠাই, তখন আপনার কাছে আমাদের সম্পর্কে কি কি তথ্য ( যেমন ইমেইল এড্রেস বা ফোন নাম্বার অথবা কেউ ছদ্দনামে লিখলে তার আসল পরিচয়, এইরকম) যায়????? স্যার আশা করি আপনি উত্তর দিবেন।।।।। রিদিশা,ঢাকা
উত্তর: হয়তো অনেক কিছুই যায়। কিন্তু আমি সেগুলো খুঁজে বের করার চেষ্টা করি না। সেজন্য যারা আমাকে গালাগাল দিতে চায় তারা এখানে ধুমসে গালাগাল করে!

প্রশ্ন: স্যার আমি চতু্র্থ শ্রেণীতে পড়ি ।আমার নাম চয়ন সরকার আবির ।আমার প্রশ্ন হচ্ছে আপনি যদি বাচ্চাদের জন্য না লিখতেন তাহলে আমরা কার বই পড়তাম ? কারন আমার আপনার লিখা সব বই পড়া শেষ ।এখন আমি কোন বই পড়বো ভেবে পাচ্ছি না ।বই পড়া আমার অভ্যাস এখন আপনার বইগুলোই আমি বার বার পড়ি যখন সময় পাই।
উত্তর: আমি 230 টা থেকে বেশি বই লিখেছি তুমি কি সব বই পড়ে ফেলেছ ? পৃথিবীতে আরো অনেক ভালো ভালো লেখক আছে, তাদের লেখা বই পড়া শুরু করে দাও। তুমি কি সত্যজিৎ রায়ের বই পড়েছ? Roald Dahl এর বই পড়েছ? Harry Potter পড়েছ? না পড়ে থাকলে এখুনি পড়া শুরু করে দাও।

প্রশ্ন: আপনার সাইন্টিস্ট মামা বইটা পড়ে দারুণ লেগেছে মুশফিকিন মাহমুদ চতুর্থ শ্রেণি ঠাকুর পারা কুমিল্লা।
উত্তর: থ্যাঙ্কু! থ্যাঙ্কু! থ্যাঙ্কু!

প্রশ্ন: একজন এখানে লিখেছে যে কোনো কোনো কোচিং নাকি ভালোও হয়! পাঠ্যবইয়ে যে তথ্যের ব্যখ্যা নেই, তা নাকি সুন্দর করে বুঝিয়ে দেয়! বাহ্! এই যদি হয় চিন্তার ছিরি, তাহলে দেশ তো চুলোয় যাবেই!
পাঠ্যবইয়ের মান কোনো ব্যাপার না। সবার হাতে হাতে স্মার্টফোন আছে, তথ্য খোজাও কোনো ব্যাপার না। শুধু চাই ইচ্ছা। এখন আর লাইব্রেরীতে বুদ হয়ে হাজার হাজার বই ঘাটার দিন নেই। খুব সহজে যেকোনো দরকারি তথ্য দেখে নেয়া যায়। হ্যাঁ বলতে পার সবার তো ফোন কেনার সামর্থ্য নেই, কিন্তু বাস্তবতা হল, অষ্টম শ্রেণী, কিংবা এর উর্ধ্বে যে সব শিক্ষার্থী পড়াশোনা করে তাদের বেশিরভাগই অন্তত একটা নেটওয়ার্কিং ডিভাইসের মালিক। বাকি রইল ইন্টারনেটের দাম। এ একটা অজুহাত ছাড়া তেমন কিছুই না। প্রতিদিন চব্বিশ ঘন্টা ফেসবুক চালানোর জন্য ডাটা প্যাক নিতে পারবে, আর দুই এক ঘন্টা গুগলিং করতে পারবে না? এ আবার বিশ্বাস করব? ছিঃ!
এখনকার ছেলেপিলে সারাক্ষণ ফেসবুকে অথবা tiktok এ ফালতু সব জিনিস দ্যাখে, আর উপস্থাপিত সব তথ্য ছাগলের মত বিশ্বাস করে। ওদের কেউ যদি বলে, “ওই কালকা কেয়ামত ঐব!” ওরা লোটা নিয়ে বের হয়ে যাবে। করোনা মহামারীর শুরুর দিকে বিভিন্ন রাজনৈতিক আর ধর্মীয় “বসেরা” কত ধরনের আজগুবি কথাই না বলেছে। ওদের চেহারা দেখলেই এখন আমার গা গুলোয়।
কোথাথেকে কোথায় চলে যাচ্ছি, যাহোক, পড়ালেখার মূল উদ্দেশ্য যে ডাক্তার ইঞ্জনিয়ার হয়ে বাড়ি- গাড়ি করা না এইটাই এইসব কোচিং পাগলাদের বোঝাতে হবে।
আর একটা কথা স্যার, বেআদবই নিবেন না। এটা নিতান্তই আমার ব্যাক্তিগত অভিমত, আমার প্রাতিষ্ঠানিক শিক্ষার ওপর প্রবল বিতৃষ্ণা। আমার যা ইচ্ছা আমি তো কেন পড়তে পারব না? কেন আমাকে পরীক্ষার চাপে থাকতে হবে? A+ এর ভ্যালু নাই, আপনিও জানেন, আমিও জানি, তাহলে ইউনিভার্সিটি ভর্তি পরীক্ষায় নম্বর কেন অ্যাড হয়? … … … … তাই প্রাতিষ্ঠানিকভাবে সিএসই আমার শেখা হবে না। ( যদিও কিছু English pirated বই ডাউনলোড করেছি সখ টা মেটানোর খাতিরে। পাইরেসি করেও বিন্দুমাত্র অপরাধবোধ কাজ করছেনা! যা খুশি ভাবতে পারেন।)
কিন্তু প্রাতিষ্ঠানিক শিক্ষার মত কি আর হবে? আমি কি আর হাতে কলমে কোড করতে পারব? ( মোবাইলের স্ক্রিনে কোড করা অনেক স্লো. আর অ্যান্ড্রয়েড সিস্টেম হ্যাক করলেও সব প্রোগ্রামিং ভাষা সাপোর্ট করে না।)
নাহ্ ভেক্টরের মত বার বার দিক বদলাচ্ছি। এবেলা লিখা শেষ করে দেই নাহলে আবার অন্যদিকে কিছু লিখা শুরু করে দেব 🙂
ভালো থাকবেন। ওহ্ ভালো কথা! দেখলাম আপনি লিখেছেন বাংলা টাইপ করতে নাকি কিছুটা সমস্যায় পড়েন? আমার একটা ফাটাফাটি বুদ্ধি আছে! আপনি গুগল কিবোর্ড ব্যবহার করেন না কেন? সেখানে উচ্চারণ ভিত্তিক টাইপিং, এমনকি ভয়েস টাইপিং এর ব্যবস্থাও আছে! Internet connection থাকলে ভয়েস টাইপিং অবিশ্বাস্য রকমের accurate! যদি ফোনে টাইপ না করতে চান, তাহলে পিসি তে একটা অ্যান্ড্রয়েড এমুলেটর ডাউনলোড করে নিন, আরামসে অ্যান্ড্রয়েড অ্যাপস চালাতে পারবেন। তাছাড়া উইন্ডোজ ১১ তে বিল্ট ইন এমুলেটর আছে! একটু গুগলিং করলেই বিস্তারিত পেয়ে যাবেন 👋
উত্তর: তোমার লেখাটুকু পড়ে মনে হল তুমি মোটামুটি ঠিক লাইনে আছো। আমি শুনে খুবই খুশি হয়েছি যে তুমি ফেসবুক কিংবা টিক টক জাতীয় পেপার থেকে উপরে উঠতে পেরেছে আমার নিজের হিসেবে কেউ যদি পৃথিবীতে কিছু দিতে চায় তার মানসিকতা ফেসবুক এবং টিকটক জাতীয় বিষয়গুলো থেকে বেশি হতে হবে। তবে একটু মনে রেখো প্রাতিষ্ঠানিক পড়াশোনা ভালো না লাগলেও লেগে থাকতে হয়, কারণ যদি না থাকো তাহলে পরে বড় হয়ে যদি তোমার মনে হয় সময় থাকতে প্রাতিষ্ঠানিক পড়াশোনায় আরেকটু গুরুত্ব দেওয়া উচিত ছিল, তখন নিজের কাছেই খারাপ লাগবে। কেমন করে কথা বলে লেখা যায় সেটা নিয়ে আমাকে আইডিয়া দিয়েছো, থ্যাঙ্কু। কিন্তু আমি বাসায় বসে বসে একা একা কথা বলে যাচ্ছি সেটা দেখে অন্য সবাই আমাকে পাগল ভাবতে পারে।

প্রশ্ন: আচ্ছা স্যার ছোটদের নিয়ে আপনি কাজ করেন। তারা বড় হলে তখন কি তাদের নিয়ে কাজ করা যায় না..? আপনি তো আমাদের কিছু শেখাতে চান তাই যে কিছুই বুঝার সময় পায় নি তাকে নিজের দর্শন শেখানের চেষ্টা কেন..? আমরা যারা বড় হয়েছি কিন্তু কোনো আদর্শ ধারণ করতে পারিনি আমাদের কিছু শেখান। আমরা মনে করি ইসলাম সঠিক আদর্শ তাই এটা মানতে চাই। এক্ষেত্রে আমাদের সাহায্য করেন প্লিজ……।
উত্তর: আমি যে বড়দের নিয়ে কাজ করি না সেটা পুরোপুরি সত্যি না। আমিতো ইউনিভার্সিটিতে পড়াতাম, সেখানে যে সমস্ত ছেলে মেয়েদের পড়িয়েছি তারা তো মোটামুটি বড়ই হয়ে গেছে! এখনো অনেক ধরনের কাজ করি। তোমাকে সাহায্য করতে বলেছ, ঠিক কী ধরনের সাহায্য বুঝতে পারিনি। ইচ্ছা করলে আমার ইমেইলে লিখে ব্যাখ্যা করতে পার।

প্রশ্ন: স্যার বিবর্তন তত্ব অনুযায়ী তো রুপান্তরের মাধ্যমেই সকল প্রাণী সৃষ্টি হয়েছে..! তাহলে সকল প্রাণীয় সতন্ত্র বৈশিষ্ট্যগুলো অটোমেটিক তৈরী হয়েছে…? সকল প্রাণীর মধ্যে দুটি লিঙ্গ কি অটোমেটিক..? উচ্চ শ্রেণির কোন প্রাণীতে কি উভয়লিঙ্গের বৈশিষ্ট্য দেখা যায়..? কম্পিউটারের প্রসেসর তো সিলিকন দিয়ে তৈরী তবুও প্রকৃতিতে সিলিকন খনিতে প্রাকৃতিকভাবে প্রসেসর পাওয়া যায় না অথচ তার চেয়েও জটিল প্রাণীদেহগুলো এমনি এমনি তৈরী হয়ে গেলো..? এটা কি বৈজ্ঞানিক…? বৈজ্ঞানিক গবেষণার যে প্রক্রিয়া তার মাধ্যমে কি কোনো সামান্য প্রাণী গবেষণাগারে তৈরী করা সম্ভব হয়েছে..? তাহলে বিবর্তন কিভাবে বৈজ্ঞানিক হয়..? বিজ্ঞানে কি এমনি এমনি বিশ্বাস করার কিছু আছে..?
উত্তর: তুমি যদি প্রশ্ন করো বিবর্তন কিভাবে বৈজ্ঞানিক হয় তাহলে তোমার সাথে কথা বলার কিছু নেই! একজন মানুষ মুখের কথা বললেই সেটা বিজ্ঞান হতে পারে না বিজ্ঞানের বিষয় হতে হলে তাকে অনেক নিয়ম মানতে হয়। পৃথিবীতে অনেক মানুষ আছে যারা বিবর্তনকে বিশ্বাস করে না, তাতে বিবর্তনের কোন ক্ষতি হয়নি, বরং মানুষগুলোর একটু ক্ষতি হয়েছে তারা বিজ্ঞানের একটা চমৎকার তত্ত্ব উপভোগ করতে পারলো না! বিজ্ঞানের কোন বিষয় আসলে নির্বাচনে দাঁড়ায় না যে তাদেরকে অনেক মানুষের ভোট পেয়ে জিতে আসতে হবে!

প্রশ্ন: টুনটুনি ও ছোটাচ্চুকে নিয়ে লেখার জন্য আমি আপনাকে সাহায্য করতে চাই । আপনি রাজি কিনা বলুন? (দয়া করে আমার কথা হেসে উড়িয়ে দেবেন না )
উত্তর: তুমি যদি টুনটুনি ও ছোটাচ্চু গল্প লিখে আমার উপরে চাপ কমাতে পারো তাহলে আমার লাভ হয়। কিন্তু যদি শেষ পর্যন্ত আমাকেই লিখতে হয় তাহলে তুমি কেমন করে আমাকে সাহায্য করবে বোঝাও।

প্রশ্ন: https://www.facebook.com/1000… … … … 3055/ আমরা জানি মুক্তিযুদ্ধের সময় জামআত পাকিস্তানকে সাহায্য করেছে কিন্তু এই রেফারেন্স তার উল্টা কথা বলছে..
উত্তর: তোমার নিশ্চয়ই কাজের অভাব আছে তাই এধরনের উল্টাপাল্টা লেখা পড়ে সময় নষ্ট করছো! ফেসবুকে কিছু থাকলে সেটা পড়তে হবে কিংবা বিশ্বাস করতে হবে সেটা কেন তোমার মনে হচ্ছে

প্রশ্ন: Sir apni ki Amar dangerous Mami boita ei website e ektu dite parben? Boimela theke ashar por mone hoyese boita kena hoy I ar shomoy kore jawao hoy I tai Jodi diye diyen boita onek upokrito hotam. Sorry English e lekhar jonno Bangla ei letter khuje Pete onek shomoy Lage . Amr Abar dhorjo ektu kom…. Raina .Dhaka
উত্তর: আমি খুবই দুঃখিত যে আমি নির্দিষ্ট কোন বই এই ওয়েবসাইটে দিতে পারি না! প্রথমে সেটাকে স্ক্যান করতে হবে, তারপর সেটাকে পিডিএফ করতে হবে, এত সময় কোথায় বল? তবে আমি যতটুকু জানি আমার অনেক বইই কেউ না কেউ কোন না কোন ওয়েবসাইটে দিয়ে রাখে। তুমি কষ্ট করে সেখান থেকে নিয়ে পড়ে ফেলো প্লিজ।

প্রশ্ন: স্যার আমি লক্ষ্য করলাম আমাদের ক্লাস ৬ এর ICT বই‌টা মুহাম্মদ জাফর ইকবাল নামের কেউ সম্পাদনা করেছেন। আপনি কি আমাদের বইটার সম্পাদক? রায়না,ঢাকা
উত্তর: হ্যাঁ। আমি তোমাদের বইয়ের সম্পাদক। তবে তুমি জেনে খুশি হবে তোমাদের জন্য সবাই মিলে আর চমৎকার বই লিখছে!

প্রশ্ন: Lal salam, Zafar Iqbal Dadu. Hope you’re doing good. If I ever write any story In bangla or english and intend to send that to you. Would you like to read it and comment on it? —- Syeda Kashfi from Savar, Dhaka.
উত্তর: তোমাকে শুধু লাল নয়, লাল-সবুজ সালাম! তোমার লেখা পড়ার সময় পাবো কিনা তার উপর নির্ভর নির্ভর করে তবে তুমি অবশ্যই পাঠাতে পারো

প্রশ্ন: Sir, you wrote , ”Appendix is an unnecessary organ. ” But Arif Azad said ”It is very important for our bodys safety. Please reply in Bangla.
উত্তর: তুমি যদি ঘোড়া হও তাহলে অবশ্যই অ্যাপেন্ডিক্স তোমার জন্য খুবই জরুরী একটা জিনিস! ঘোড়ার অ্যাপেন্ডিক্স কত বড় তুমি জানো ?

প্রশ্ন: tuntuni ar cotassu first golpo ta koi.my name is yousra.my address hoilo akbarsah railway housing society.
উত্তর: টুনটুনি ও ছোটাচ্চু বইগুলোর প্রথম বইটাতে প্রথম গল্পটা আছে!

প্রশ্ন: sir, আপনার প্রিয় রং কি? এশা — ঈশ্বরদী, পাবনা
উত্তর: আকাশের জন্য নীল, গাছের পাতার জন্য সবুজ, ইলিশ মাছের জন্য রুপালি, মনে হয় চুলের জন্য সাদা, গোঁফের জন্যও সাদা … … (তোমার সাথে একটু ঠাট্টা করলাম, সব রঙই আমার প্রিয়।)

প্রশ্ন: স্যার আপনার জন্য অনেক চিন্তা হয়। আমাদের উত্তর দেওয়ার সময় না হলে সমস্যা নেই। তবে মাসে অন্তত একবার হলেও জানাবেন যে আপনি স্বপরিবারে ভালো আছেন, প্লিজ! সাবা////ঢাকা (আমার পক্ষ থেকে এত্তগুলো ভালোবাসা রইল… ভালো থাকবেন স্যার আর আমাদের জন্য সুন্দর সুন্দর বই লিখতে থাকবেন! ♥♥♥)
উত্তর: আমার বয়স ৭০  হয় হয় করছে কাজেই এখন চিন্তার কিছু নেই! আমার জীবনের টার্গেট ছিল ৫০ , আমি এর মাঝে ২০ বছর এক্সট্রা বেঁচে আছি।

প্রশ্ন: স্যার আমি আপনার সঙ্গে দেখা করতে চাইইইইইই……. যারা ঢাকায় থাকে না তাদের কি আপনার সাথে দেখা করার কোনো অধিকার নেই??? ঠিক যেমন তারা একুশে বইমেলায় যেতে পারে না। ইফ্ফাত, গাইবান্ধা
উত্তর: আছে নিশ্চয়ই আছে। আমি যখন গাইবান্ধা যাব তখন নিশ্চয়ই তোমার সাথে দেখা হবে।

প্রশ্ন: শ্রদ্ধেয় জাফর ইকবাল স্যার, আসসালামুআলাইকুম স্যার😊। আমি আপনার অন্নেক বড় ভক্ত মুনতাহা মনসুর ঊর্বী। স্যার আমি আজকে আপনার কাছে একটি অনুরোধ জানাতে চাই।এক বছর হলো আপনাকে এটি জানাতে চাচ্ছিলাম কিন্তু সাহস পাচ্ছিলাম না স্যার। স্যার প্লীজ আপনি কিন্তু আমার অনুরোধটি রাখবেন। (প্লীজ টু দি পাওয়ার ইনফিনিটি স্যার) স্যার আপনার অনেক গুলো বই থেকেই তো মুভি বানানো হয়েছে।তাহলে স্যার আরেকটা বই দিয়ে কি আরেকটা মুভি বানানো যায়না? প্লীজ স্যার প্লীজ আপনার রাশা বইটি দিয়ে কি একটা মুভি বানানো যায়না স্যার?
স্যার আপনি জিজ্ঞেস করবেন কেনো আমি আপনাকে হটাত করে এই অনুরোধ করছি।স্যার অনেকগুলো কারণের মধ্যে সবচেয়ে গুরুতর দুটো কারণ হলো স্যার -২.আমি মনে করি এই মুভি টা আমাদের প্রজন্মের জন্য সত্যি দরকার স্যার।১.স্যার মুভি টি বানানো হলে আমি এই মুভিটিতে রাশা হিসেবে অভিনয় করতে চাই স্যার। তাই স্যার প্লীজ স্যার প্লীজ আপনি কিন্তু আমার অনুরোধ টা রাখবেন স্যার। ইতি, মুনতাহা মনসুর ঊর্বী
উত্তর: শোনো, আমি তোমাকে বলি, আমাকে প্রায় প্রতি সপ্তাহেই কোন না কোন চিত্র পরিচালক যোগাযোগ করে আমার লেখালেখি থেকে ছবি বানানোর আগ্রহ দেখায়। আমি কখনো তাদের প্রস্তাবে রাজি হই না, তার কারণ আমি যখন একটা বই লিখি আমি চাই তোমাদের বয়সের ছেলেমেয়েরা সেই বইয়ের সব কিছু নিজে নিজে কল্পনা করুক। যাদের কল্পনা করার ক্ষমতা বেশি তারা অনেক সুন্দর ভাবে আমার বইটা নিজের মত করে কল্পনা করবে। কিন্তু যদি একজন চিত্র পরিচালক আমার একটা বই থেকে একটা সিনেমা বানায় তাহলে সে যেভাবে বিষয়টা দেখাবে সবাইকে সেভাবেই দেখতে হবে। কেউ আর কোন কিছু নিজের মতো করে কল্পনা করতে পারবে না। আমি সেটা চাইনা আমি কখনোই কাউকে আমার লেখা থেকে সিনেমা বানাতে দেই না। তোমার যেহেতু ছবিতে অভিনয় করার শখ আমি দোয়া করি রাশা বইটিতে না হলেও তুমি যেন অন্য কোন বইয়ে অভিনয় করার সুযোগ পাও।

প্রশ্ন: Sir Salam. Hope that you are in good health. I have called you, may be you are busy. I am sorry for disturbing you . We want to invite you in program on 7th April 2022. It would be my honor if we could talk regarding your presence. my number 01711505436 Mahbub
উত্তর: হায় হায় এপ্রিল মাসে দাওয়াত দিতে চেয়েছিলে, কিন্তু এখন জুলাই মাস! তবে বিষয়টা হলো আমি আসলে কখনোই টেলিফোন ধরি না, তাই কেউই আমাকে টেলিফোন করে পায় না!

প্রশ্ন: স্যার,গতকাল আমাদের প্রস্তুতিমুলক পরীক্ষা শেষ হলো।এই পরীক্ষায় একটা বিষয় লক্ষ করলাম যে, যারা স্কুলে নিয়মিত আসে না তাদের অনেকেই, সামনে, পিছনে বা পাশে যে বসেছে তাদের খাতা দেখে লিখেছে।আমার সামনের জন তো আমাকে প্রায় পাগল করে ফেলে এমন অবস্থা।আমি নিজেই খুব ধীরে লিখি,লিখে শেষ করতে পারব কিনা সেই চিন্তায় থাকি,তার উপর কেউ যদি আমার খাতা দেখতে চায়, কেমন লাগে বলুন।শুধু তাই না একবার দেখে নিয়ে লিখতে পারেনা,একটা শব্দ দেখে একটা শব্দ লেখে।আপনি কি ভাবছেন আমি আমার বন্ধুদের দেখাই না?না,তা ঠিক না।এখন কেউ যদি probability tree আঁকতে না পারে,বিজ্ঞান বিভাগে পড়ে ‘বিজ্ঞান’বানান লিখতে না পারে কিংবা একটা সূত্র বলে তাতে মান বসাতে বললে তা করতে না পারে তখন কি করা উচিত আপনিই বলেন।তবে একটা বিষয় মাঝে মাঝে ভাবি,এরকম কত শিক্ষার্থী আছে আমাদের দেশে।তারা কেন নিজেরা পরীক্ষা হলে লিখতে পারেনা?কেন তারা স্কুলে নিয়মিত না?এটা কি শুধু আমাদের শিক্ষাব্যবস্থার দোষ নাকি মোবাইল ফোনের প্রতি আসক্তি।আমারতো মনে হয় দোষটা আমাদের শিক্ষাব্যাবস্থার নয়,দোষটা আমাদেরই।আমরাই শিক্ষাব্যবস্থাটিকে ভুলভাবে ব্যবহার করছি!(করোনাকালে এসাইনমেন্টের নামে এখন শতকরা ৯০ জনের হাতেই স্মার্টফোন শোভা পাচ্ছে।শুধু ক্লাসরুমে না,পরীক্ষাহলেও ছাত্রছাত্রীরা মোবাইল নিয়ে ঢুকছে,কেউ ধরতেও পারছেনা।কি বিচ্ছিরি একটা ব্যাপার!)।তবে এসব মানুষেরা সহজেই অন্যের সাথে মিশতে পারে,বন্ধু হতে পারে।মাঝে মাঝে তো পরীক্ষাহলে স্যারদেরও হাত করে ফেলে(যেমনটা আমাদের jsc পরীক্ষায় ঘটেছিল)।মাঝে মাঝে এসব দেখেও না দেখার ভান করি।কারণ,এগুলো না পারি সহ্য করতে,না পারি কিছু বলতে।আজ আপনাকে বললাম,বলে কোনো লাভ না হলেও বললাম,যদি আপনি কোনো সান্ত্বনার বাণী শোনান সেজন্যে।(জিয়াদ হোসেন,রূপসা,খুলনা)
উত্তর: তোমার বিশাল লেখাটা পুরোটাই দিয়ে দিলাম! পড়ে আমি খুব মজা পেয়েছি অন্যরাও নিশ্চয়ই অনেক মজা পাবে। তোমার চারপাশে যারা আছে তুমি তাদের নিয়ে খুব বেশি দুশ্চিন্তা করোনা, কারণ আমরা যখন বড় হয়েছি আমাদের আশে পাশে যারা ছিল তারাও এ রকমই ছিল। না, আমি তোমাকে কোন সান্তনার কথা শুনাতে পারবো না। তবে এই খুবই প্যাথেটিক সিস্টেমের ভেতর দিয়েও তোমাদের মতন কিছু খাটি ছেলেমেয়ে বের হয়ে আসবে। তারাই ভরসা! আগেও এরকম হয়েছে ভবিষ্যতেও হবে।

প্রশ্ন: স্যার, আপনি আয়েশা দাদী কিংবা নানীর লেখা বইটা সম্পাদনা করসেন? বইটা পড়ে অলমোস্ট লাগতেসিলো যেন আপনি লিখসেন, এত কাটছাট করা ঠিক হয় নাই, কিন্তু অনেকদিন পর কোনো বই পড়ে আমার খুব কান্না আসছে। সম্ভবত সেই জায়গাগুলাই আয়েশা দাদী কিংবা নানী সম্পুর্ন নিজে লিখসেন, কেউ কাটাছেড়া করে নাই। উনাকে দেখতে ইচ্ছা করে। আফসোস আমি দেখার জন্য যতদিনে ঢাকা যাব, ততদিনে আমার লিস্ট অর্ধেক ফাকা হয়ে যাবে। আমার নাম হৃদিকা। আমি চট্টগ্রাম থেকে, কলেজিয়েটে পড়ার ইচ্ছা ছিল। মেয়েদের তারা নেয় না, নটরডেমেও পড়ার ইচ্ছা ছিলো, সেখানেও মেয়েদের নেয় না। লেখাপড়া ছেড়ে দিব ভাবতাম স্যার। ভাগ্য ভালো কিছু জায়গায় আগেরকালের কিছু মেয়ে কামড়াকামড়ি করে জায়গা করে রেখেছিলো। তাই এখনো পড়ি।
উত্তর: আমার আমার মা পুরো বইটা নিজেই লিখেছেন আমি শুধু জায়গায় জায়গায় ঠিক করে দিয়েছি। আমার মা ২০১৪ সালে মারা গিয়েছেন, তাই তাকে আর দেখতে পাবে না! তুমি কোথায় পড়ছো সেটা নিয়ে বেশি মাথা ঘামিও না, পড়াশোনাটা নিজের উপরে, স্কুল কলেজ বিশ্ববিদ্যালয়ের উপরে না। তুমি খুব সুন্দর করে প্রকাশ করেছে যে আগের কালের কিছু মেয়ে কামড়াকামড়ি করে তোমাদের জন্য ব্যবস্থা করে রেখেছিল, কথাটা পুরোপুরি সত্যি! কে জানে, হয়তো এখনও এই কথাটি সত্যি। তোমাদের হয়তো ভবিষ্যতের মেয়েদের জন্য কিছু কামড়া কামড়ি করে যেতে হবে।

প্রশ্ন: স্যার কেমন আছ? আশা করি তুমি ভালো আছ। স্যার আমি মনে প্রাণে একজন ফিজিক্স প্রেমী। তুমি আমাকে কয়েক টা ফিসিক্স বই এর নাম বল প্লিজ। আনন্দ সিদ্দিকী। ক্লাস ১০
উত্তর: তুমি যদি সত্যি কারের ফিজিকস প্রেমী হয়ে থাকো তাহলে তোমাকে ফিজিক্সের প্রবলেম সলভ করতে হবে তুমি কি সেটা করছ? তুমি যদি আমাদের ন্যাশনাল কারিকুলাম এর ছাত্র হয়ে থাকো তাহলে ও লেভেল এবং এ লেভেলে যে পদার্থবিজ্ঞান বই পড়ানো হয় সেগুলি করে দেখতে পারো। আর যদি জনপ্রিয় ধারার বই পড়তে চাও তাহলে মিচিও কাকুর বইগুলি পড়ে দেখতে পারো

প্রশ্ন: স্যার,,,, আপনার বাবার তো জ্যোতিষশাস্ত্র নিয়ে অনেক আগ্রহ ছিলো,, এর সুবাদে আপনাদেরও এ নিয়ে অনেক বই পড়া হয়েছে,,আমারও এই নিয়ে আগ্রহ আছে।আপনি কি আমাকে কিছু বই সাজেস্ট করবেন,,প্লিজ,,,,,( ইলা,, নরসিংদী)
উত্তর: না, করবো না। এ ধরনের ভুয়া বিষয় আমি কিভাবে তোমাকে সাজেস্ট করি বল!

প্রশ্ন: “প্রশ্ন: তুর্যয় পাল, ফেনী। স্যার, আমি এবার শিক্ষার মাধ্যমিক পর্যায় পেরিয়ে উচ্চ মাধ্যমিক পর্যায়ে অর্থাৎ একাদশ শ্রেণিতে উঠেছি। কিন্তু আমার কেমন জানি পড়ালেখাসহ আরও বিভিন্ন বিষয় উলট – পালট মনে হয়! আপনি যদি কোনো উপদেশ দেন তাহলে উপকৃত হবো মনে হয়। উত্তর: উপদেশ দিয়ে যদি কোনো কাজ হত তাহলে দিতাম। উপদেশ দিয়ে কোনো কাজ হয় না, নিজের দায়িত্ববোধ দিয়ে নিজের কাজ করতে হয়! উৎসাহ দিলে মাঝে মাঝে কাজ হয়। দিব উৎসাহ?” কপি – পেস্ট করলাম ! উৎসাহ দিয়েন, স্যার ।
উত্তর: আমি জানি লেখাপড়া করতে কারোই ভালো লাগে না, লাগার কথাও না। তারপরও করতে হয়, অনেকটা শাক-সবজি খাওয়ার মতন, শাক-সবজির খাওয়া থেকে ফ্রায়েড চিকেন খেতে মজা বেশি লাগে কিন্তু তার পরেও আমাদের শাক-সবজি খেতে হয়! তুমি যেহেতু এখন ইন্টারমিডিয়েট পড়ো, কাজেই যদি দাঁতে দাঁত কামড়ে আর এক দুইটা বছর কাটাতে পারো তাহলে তোমাকে আর এই বোরিং লেখাপড়া করতে হবে না! তুমি তখন বিশ্ববিদ্যালয় পর্যায়ে নিজের শখের মতন নিজের ইচ্ছা মতন আরো মজার লেখাপড়া করতে পারবে। তোমার বিভিন্ন বিষয়ে উলটপালট মনে হচ্ছে সেজন্য দুশ্চিন্তার কিছু নাই আসলেই আমাদের চারপাশে যে বিষয়গুলো থাকে তার বেশিরভাগই উলটপালট, আমাদের সেটা মেনে নিতে হয় সেটাতে অভ্যস্ত হতে হয় তুমিও অভ্যস্ত হয়ে যাও

প্রশ্ন: স্যার গণিতের উচ্চতর বিষয়ে কোনো বাংলা বই নেই । একটা লিখবেন please please please please please please please. আপনি আপনার বেশির ভাগ বইয়েই (physics) গাণিতিক সমীকরণ বেশি লিখেন না । যদিও বইগুলো খুব ভালো । তবে অনেক অনেক গাণিতিক সমীকরণ নিয়ে একটা বই লিখবেন please please please please please please please please please please please please please. হতে পারে ১০০০ পৃষ্ঠার পদার্থবিজ্ঞানের দ্বিতীয় পাঠ । বা একটু খানি পদার্থ বিজ্ঞান । বা সহজ পদার্থ বিজ্ঞান । একটা লিখবেন কিন্তু এ দাবি থাকল । উত্তরে কিন্তু বলে দিবেন ‘কবে লিখবেন’ ।সাইমুন কুষ্টিয়া ।
উত্তর: তুমি গণিতের উচ্চতর বিষয়ে বাংলা বই লিখতে বলেছ। আমাদের দেশে একেবারে টানা ১২ বছর সবাইকে ইংরেজি শেখানো হয়, তা দিয়েই কাজ চালানো মতন ইংরেজি সবারই জানার কথা। পৃথিবীতে গণিত এবং বিজ্ঞানের উপরে এত চমৎকার ইংরেজি বই আছে তখন সেই বইগুলো যে কেউ পড়তে পারবে। কাজেই এখন বসে বসে নতুন করে বাংলায় পদার্থ বিজ্ঞান কিংবা গণিতের বই লেখার কোন মানে হয় না। তারপরেও আমি হয়তো এধরনের কাজ করতাম যদি আমার অন্য কোন কাজ না থাকতো। কিন্তু আমাকে আর একশ রকম ভিন্ন ভিন্ন কাজ করতে হয়, নতুন একটা ডিউটি হয়েছে, প্রতি বছর হিসাব করে টুনটুনির বই লিখতে হয় কাজেই তুমি বলো আমি কেমন করে এখন উচ্চতর গণিতের উপর বই লিখবো?

প্রশ্ন: আমার জানা নেই সময় পরিভ্রমণের ওপর কোনো সূত্র আবিষ্কৃত হয়েছে কিনা, তবে সেটি যদি না হয়ে থাকে, আমার ইচ্ছা আমি এর ওপর গবেষণা করব। বিষয়টা মুহাম্মাদ জাফর ইকবাল স্যার কে জানালে মন্দ হয়না। হয়ত চশমার কোণা দিয়ে আমার দিকে তাকিয়ে তাচ্ছিল্যের সাথে একটি মুচকি হাসি দিবেন। যেমনটা সাধারণত আমি করে থাকি। কে জানে!
আমি সারা জীবন ‘রুবি’ নামক প্রোগ্রামিং ভাষায় প্রোগ্রাম করে এসেছি। কিন্তু আমার খুব খারাপ লাগে যে ভাষাটি এখন কতটা অবহেলিত। একমাত্র ‘রুবি অন রেইলস’ ফ্রেমোয়ার্ক টি ছাড়া তেমন কোনো রিচ মডিউল এটির জন্য করে রাখা নেই। তাই আমাকে বাধ্য হয়ে এখন পাইথন শিখতে হচ্ছে।
বর্তমানে আমি ক্লাস এইটে পড়ি। আমার ক্লাসে সবাই বড় হতে চায়, একমাত্র আমি ছাড়া। শৈশবের কথা আমার বড় মনে পড়ে। বাচ্চাদের সামান্য কিছু নিয়ে উত্তেজিত হতে দেখলে আমার ভারি ইচ্ছা করে শৈশব টা ফিরে পেতে।
… … … যাই হোক, আপনার মেইল আ্যাড্রেস যদি দেন মন্দ হয়না। আপনার সাথে কিছু বিজ্ঞান বিষয়ক প্যানপ্যানানি করা যাবে। কাজী মুহাম্মদ উমার চট্টগ্রাম
উত্তর: সময় পরিভ্রমণের উপর কাজ করতে চাও জেনে খুশি হলাম। প্রোগ্রামিং ল্যাঙ্গুয়েজটা মোটেও গুরুত্বপূর্ণ নয়, তুমি যদি যে কোনো একটি প্রোগ্রামিং ল্যাঙ্গুয়েজে প্রোগ্রামিং জানো তাহলে অন্য যেকোন ল্যাঙ্গুয়েজ শিখে নেওয়া কোন মোটেও কঠিন কোনো ব্যাপার নয়। প্রোগ্রামিং করার লজিক যদি তুমি জানো, কিছু চমৎকার অ্যালগরিদম জানো, গণিত যদি জানো তাহলে তোমার জন্য প্রোগ্রামিং করা মোটেও কঠিন কোন ব্যাপার নয়। তুমি শৈশব পার করে বড় হতে চাচ্ছ না বলে লিখেছ, ক্লাস এইটের একজন ছেলে যদি মনে করে সে তার শৈশব পার হয়ে এসেছে তাহলে আমি একটু মুচকি হাসি! মানতে চাও আর নাই চাও তুমি এখনো তোমার শৈশবেই আছো। আমার ইমেইল এড্রেস হচ্ছে m.z.iqbal@mzi.rocks এখন চারপাশে ফেসবুক টিকটক এ ধরণের অসংখ্য হালকা বিনোদনের বিষয় আছে, কেউ যদি সেগুলোতে আসক্ত হয়ে সেখানে ডুবে যায় তাহলে আমি খুব কষ্ট পাই। আমি চাই ছেলে মেয়েরা আনন্দ করুক, বিনোদন করুক কিন্তু তারপরও তারা যেন তাদের মস্তিষ্ক ব্যবহার করে কোন একটা গুরুত্বপূর্ণ কাজ করে। আমরা যে পৃথিবীর অন্য প্রাণী থেকে ভিন্ন তার কারণ আমাদের একটা মস্তিষ্ক আছে সেই মস্তিষ্কের অপব্যবহার করলে কেউ আমাদের কখনো ক্ষমা করবে না।

প্রশ্ন: আসসালামু আলাইকুম। স্যার, আপনি আমার প্রিয় লেখক। আপনার লেখা বই পড়ে হেসেছি, কেঁদেছি। এখন শখ হয়েছে, আপনার মতো উপন্যাস লেখার। স্যার, প্লিজ আপনার উপন্যাস লেখার মন্ত্রটা শিখিয়ে দিন। প্লিজ! আরেকটা কথা, আপনি তো বগুড়াকে ভুলেই গেছেন। বগুড়া আপনার উপর রাগ করেছে। রাগটি ভাঙতে কবে বগুড়ায় আসবেন? একটা আক্ষেপ— স্যার, আপনারা তিন ভাই ই লেখালেখি করেন। আমিও যদি আপনার ভাই হতে পারতাম তাহলে বোধহয় আমিও আপনার মতো লেখক হতে পারতাম। অন্তত আপনার ছেলে তো হতে পারতাম।
আপনার ভক্ত, মুশফিকুর রহমান, একাদশ শ্রেণি, বগুড়া।
উত্তর: হা হা হা! তুমি যদি মনে করো যে উপন্যাস লেখার একটা মন্ত্র আছে এবং সেই মন্ত্রটা জেনে গেলে যে কেউ উপন্যাস লিখতে পারবে তাহলে বুঝতে হবে যে তুমি বাস্তব জীবনটা এখনো দেখ নাই! পৃথিবীর কোন কাজে শর্টকাট নাই, সব কাজই কষ্ট করে করতে হয়। তবে তুমি যদি সে কাজটা করে আনন্দ পাও তাহলে সেই কষ্ট করতে আমাদের কোন সমস্যা হয় না। তুমি যদি সত্যি সত্যি লিখতে চাও তাহলে তোমাকে অনেক অনেক বেশি পড়তে হবে। যে যত বেশি বই পড়ে সে তত সুন্দর লিখতে পারে, তুমি যদি লিখতে চাও তাহলে বই পড়ো এটাই হচ্ছে লেখালেখির মন্ত্র!

প্রশ্ন: এক আধবার মাথা ঘুরে পড়ে গেলে সমস্যা নেই,,সব রকম অভিজ্ঞতাই থাকা উচিত।আপনি বলুন আপনার ইংরেজি লেখা কারা বাংলায় টাইপ করে দেয়,,,,,( ইলা, নরসিংদী)
উত্তর: আমার ইংরেজি লেখা নয়, হাতে লেখা বাংলা লেখা টাইপ করে দেন প্রফেসর মোহাম্মদ কায়কোবাদ। বিশ্বাস হয়?

প্রশ্ন: প্রিয় চাচ্চু, আমার একটা প্রশ্ন আছে।’এক সেট গয়না’গল্পটা কি শুধু এখানেই প্রকাশ হয়েছে নাকি টুনটুনি ও ছোটাচ্চু সিরিজের কোন বইয়েও আছে?আমার কাছে এই সিরিজের সবগুলি বই আছে কিন্তু এই গল্পটি কোনটির ভিতর নেই😢 প্লিস উত্তর দেবেন! অত্রি,ঢাকা।
উত্তর: যে যেরকম টুনটুনি সেরকম ছোটাচ্চু বইটিতে এক সেট গয়না গল্পটা আছে।

প্রশ্ন: প্রিয় জাফর ইকবাল চাচ্চু, কেমন আছ?আমি না অনেক দিন ধরেই তোমাকে একটা চিঠি লেখার কথা ভাবছিলাম।কিন্তু সেটা লিখে কোথায় পাঠাবো বুঝতে পারছিলামনা।আমার ভাগ্য ভাল;অবশেষে এই ওয়েবসাইটটা খুজে পেয়েছি!😄 কয়েকদিন আগেই আমি টুনটুনি ও ছোটাচ্চুর নতুন দুটা বই কিনেছি।আমি ভীইইইইইষন খুশি!!!! তোমার কাছে আমার একটা অনুরোধ আছে।সেটা হল, টুনটুনি ও ছোটাচ্চুর আরো একটা নতুন বই চাই!:)) সেই বইটাতে ছোটাচ্চুর সাথে ফারিহাপুর বিয়ে নিয়ে একটা গল্প থাকতে হবে।আর ছোটাচ্চু তো আলটিমেট ডিটেকটিভ এজেন্সি পেয়ে গেছে।এবার ছোটাচ্চুর আবার এজেন্সির নতুন নতুন কেস সলভ নিয়ে ব্যাক দেখতে চাই।কিন্তু খবরদার,ফারিহাপুকে যেন কেউ ছোটাচ্চি না ডাকে!ফারিহাপু শুধুই আমাদের আপু। প্লিস প্লিস এই অনুরোধ টা রাখো।প্লিইইইইইইইইস!!! না রাখলে কিন্তু খুব মন খারাপ হবে।☹আর আমিও চাপ দিতে থাকব😁 ওহ্ হ্যা,”আহা টুনটুনি উহু ছোটাচ্চু”বইয়ের ভূমিকায় শেষ লাইনে তুমি প্রশ্ন করেছ,যে এভাবে আর কতদিন এই সিরিজের বই বের হবে?উত্তরটা আমি দিয়ে দেই।—সারাজীবন!🙃 ইতি অত্রি অপরাজিতা মনসুরাবাদ…
উত্তর: আমিওতো ছোটাচ্চু আর ফারিহাপুর বিয়ে দিতে চাই, কিন্তু আজকালকার ছেলেমেয়েরা কেন জানি বিয়ে করতে চায় না! কি যে করি। সারা জীবন ছোটাচ্চু ও টুনটুনির বই লিখতে হবে? কি সর্বনাশ!!

প্রশ্ন: আপনার ভেসে উঠার সময় কি এখনো হয় নি????????? আর আপনি আমাদের না বলে বই মেলায় চলে গেয়েছিলেন কেন??? ( পরে অবশ্য আমরাও না আপনাকে না বলে চলে গেছি,,হুহ্,,,,) আপনার এ অকাজের জন্য আমরা একটা শাস্তি ঠিক করেছি,,শাস্তিটা হলো,,আপনাকে আমরা পাকিস্তান পাঠিয়ে দেব ( প্লেনে এত্তগুলো মাকড়সা সহ)। শাস্তিটা শুনে কি রাগ করলেন?? রাগ করলেও কিছু করার নেই,, আপনি যখন আমাদের না বলে বই মেলায় চলে গিয়েছিলেন তখন আমরাও অ,,,নে,,,ক রাগ করেছি,,,,তো,,,আপনার রাগ ( যদি করে থাকেন) আর আমাদের রাগ কাটাকাটি,,,,,,,( ইলা, নরসিংদী)
উত্তর: এই যে ভেসে উঠেছি। আবার যে ডুব দেবো না তার গ্যারান্টি কিন্তু দিচ্ছি না! এখন এখন যেহেতু করোনা বলতে গেলে নেই, থাকলেও সেটাকে আর কেউ ভয় পায় না তাই নিশ্চয়ই তোমাদের সাথে নানা জায়গায় নানা ভাবে দেখা হবে।

প্রশ্ন: স‍্যার আসসালামু আলাইকুম। আশা করি ভালো আছেন। আমরা সবাই আপনার উত্তরের আশায় থাকি।এটা ঠিক। তবে আমরা আপনার ব‍্যস্ততাটাও ও বুঝি। বড়দের এমনিই অনেক কাজ। আর আপনাদের তো আরও দ্বিগুন!!!! তাই প্লিজ আমাদের জন‍্য কষ্ট পাবেন না! আপনি অনেক ভালো স‍্যার!!! আমাদের জন‍্য ভাবেন!! এটা ভাবতেই ভালো লাগে!!! মাঝে মাঝে মনে হয় আপনি আমাদের স‍্যার এর থেকে বেশি বন্ধু!!! নিজের খেয়াল রাখবেন!! (*আর করোনা তো প্রায় চলেই গেছে!!! জানেন এখন আমাদের স্কুল আগের মত খুলে দিয়েছে!! তবে মাঝে মধ‍্যে ভয়ই লাগে অনেকের কথা শুনে। খালি মনে হয় বড় হয়ে যাচ্ছি!!) বাঁধন— টাঙ্গাইল। এখন আমি ক্লাশ ৯!!! আপনার ওয়েবসাইটটার সাথে আমরাও বড় হচ্ছি!!!
উত্তর: তোমার লেখা পড়ে বুঝতে পারছি তোমার চমৎকার একটা শৈশব আছে। অনেকেই বড় হতে চায় কারণ তারা শৈশবে খুব বেশি একটা আনন্দ করতে পারেনি তাই তারা ভাবে বড় হয়ে অনেক আনন্দ করবে। তুমি নিশ্চয়ই শৈশবে অনেক আনন্দ করছ, তাই বড় হতে চাও না! বয়সে বড় হলেও মনে মনে ছোট থাকা যায়, এই বিষয়টা আমি জানি।

প্রশ্ন: Sir, 2022 saler notun apnar lekha class 6 at science boita onek darun. Afia Azmaeen, Rajshahi
উত্তর: তোমাদের ভালো লেগেছে জেনে আমারও খুব ভালো লাগলো। তুমি কি জানো সব ক্লাসের সব ছেলেমেয়েদের জন্য এখন এরকম বই লেখা হবে?

১৮ মার্চ ২০২২

অনেকদিন পর ওয়েবসাইটে এসে মনে হল কয়েকটা প্রশ্নের উত্তর দেওয়ার চেষ্টা করি। যে কয়টা সামনে পেয়েছি, উত্তর দিয়েছি। কোনো নিয়ম না মেনে!

যাদের এখনই উত্তর দরকার তারা m.z.iqbal@mzi.rocks তে একটা ই-মেইল পাঠাতে পার।

প্রশ্ন: স‍্যার আসসালামু আলাইকুম। আশা করি ভালো আছেন। আমরা সবাই আপনার উত্তরের আশায় থাকি।এটা ঠিক। তবে আমরা আপনার ব‍্যস্ততাটাও
ও বুঝি। বড়দের এমনিই অনেক কাজ। আর আপনাদের তো আরও দ্বিগুন!!!! তাই প্লিজ আমাদের জন‍্য কষ্ট পাবেন না! আপনি অনেক ভালো স‍্যার!!! আমাদের জন‍্য ভাবেন!! এটা ভাবতেই ভালো লাগে!!! মাঝে মাঝে মনে হয় আপনি আমাদের স‍্যার এর থেকে বেশি বন্ধু!!! নিজের খেয়াল রাখবেন!! (*আর করোনা তো প্রায় চলেই গেছে!!! জানেন এখন আমাদের স্কুল আগের মত খুলে দিয়েছে!! তবে মাঝে মধ‍্যে ভয়ই লাগে অনেকের কথা শুনে। খালি মনে হয় বড় হয়ে যাচ্ছি!!) বাঁধন— টাঙ্গাইল। এখন আমি ক্লাশ ৯!!! আপনার ওয়েবসাইটটার সাথে আমরাও বড় হচ্ছি!!!
উত্তর: হ্যাঁ, আমিও দুশ্চিন্তায় আছি যে তোমরা বড় হয়ে যাচ্ছ! দরকার কী বড় হওয়ার? ছোট থেকে যাও! (আমার মতন!)

প্রশ্ন: Sir, 2022 saler notun apnar lekha class 6 at science boita onek darun. Afia Azmaeen, Rajshahi
উত্তর: শুনে ভালো লাগল। মুখস্ত, কোচিং, গাইড বই, পরীক্ষার চাপ সব কিছু থেকে এই দেশের বাচ্চাদের মুক্তি দেবার জন্য যে কাজ শুরু হয়েছে, এই বইটা হচ্ছে তার একটা অংশ!

প্রশ্ন: পৃথিবীতে আমি আপনাকে সবচেয়ে বেশি ভালোবাসি !!! অথচ আপনি পৃথিবীতে আমার অস্তিত্ব সম্বন্ধেই জানেন না।কেমন অদ্ভুত তাই না ??? দয়া করে কোনো উত্তর দিবেন না।কারণ আইডিটা আমার না। … … … …।
উত্তর: উত্তর দেবার জন্য মুখটা নিশপিশ করছে কিন্তু তুমি যেহেতু না করেছ তাই উত্তর দিলাম না!

প্রশ্ন: “এখানে একটা ইমেইল পাঠাতে পার, আমি চেষ্টা করব উত্তর দিতে, ইংরেজিতে, কারণ আমি যখন বাংলায় লিখি, সবসময় কাগজে বল পয়েন্ট কলম দিয়ে লিখি! কম্পিউটারে বাংলা লিখতেই পারি না।”
So, who typed all of your answers on this website for you?
উত্তর: কারা কারা আমার ইংরেজি লেখা বাংলায় টাইপ করে দেয় আমি যদি তাদের নাম বলি তাহলে তোমরা এত অবাক হয়ে যাবে যে মাথা ঘুরে পড়ে যেতে পার!

প্রশ্ন: প্রিয় জাফর দাদু, তুমি আমার প্রিয় লেখকদের মধ্যে অন্যতম। তোমার গল্প, উপন্যাসও প্রবন্ধগুলো আমি নিয়মিত আনন্দের সাথে মনোযোগ দিয়ে পড়ি। তার মধ্যে তোমার প্রবন্ধগুলো—আমার সবচেয়ে বেশি ভালো লাগে। যাই হোক, আমি যে প্রশ্নটি নিয়ে এসেছি তোমার কাছে, সেটা অনেকটা অভিযোগও বটে— দাদু, তুমি প্রায়ই বলো, কোচিং না করতে, গাইড বই না পড়তে, আর মুখস্থ না করে—বুঝে বুঝে পড়তে। আমিও তোমার সাথে একমত। তাই আমি এগুলো কিছুই করি না। সবার আগে পাঠ্যবই নিজে নিজে বুঝে বুঝে পড়ি। তবে দাদু, সমস্যা হচ্ছে যে— বইয়ে অনেককিছুরই উল্লেখ পাওয়া যায়, কিন্তু সেগুলোর কোনো ব্যাখ্যা নেই। তাই আমি বাইরের বই ঘেটে বের করে পড়ি। পাঠ্যপুস্তকে কয়েক ক্ষেত্রে উদাহরণ পাওয়া যায়, তবে ভালো কোনো ব্যাখ্যা বা সংজ্ঞা নেই। আর একটা কথা পরিষ্কার করি, উদাহরণ কিন্তু ভালো জিনিস, তবে সংজ্ঞা ও ব্যাখ্যা ছাড়া আমরা কিভাবে নিজে থেকে উদাহরণ তৈরি করবো? আমি নাহয় বাইরের বই ঘেটে দেখি। কিন্তু বাকিরা? সবার তো সুযোগ ও সামর্থ্য নেই, তারা কি করবে? এজন্য অধিকাংশ শিক্ষার্থী কোচিংমুখী হয়। (সব কোচিং খারাপ না, অনেক কোচিং ভালোও হয়। ভালোভাবে পড়া বুঝিয়ে দেয়, অবশ্য পরীক্ষার ঝাক্কি কিন্তু পোহাতে হয়—যেটাই তাদেরকে চাপে রাখে) তাই দাদু, তোমাকে বলি, পাঠ্যবইয়ের মানটা যদি ভালো হতো—সবকিছু যদি সুন্দরভাবে সহজে ব্যাখ্যা করা থাকত, তাহলে হয়তো অনেক শিক্ষার্থী কোচিং থেকে দূরে থাকত। পড়াশোনার চাপে থাকতে হতো না। তাই দাদু, আমাদেরকেও সম্পূর্ণভাবে দোষ দেওয়া যায় না। অনেক ঝামেলা পাঠ্যবই ও শিক্ষাব্যবস্থাতেও আছে। ইতি, আদৃতা বড়ুয়া, চট্টগ্রাম।
উত্তর: তোমার কথাগুলো পুরোটা দিয়ে দিলাম। কিন্ত আমি খুব ভালো করে জানি পাঠ্যবই আর শিক্ষাব্যবস্থা যত খারাপই হোক, সেটা মূল কারণ না। অস্ট্রেলিয়ার পাঠ্যবই আর শিক্ষাব্যবস্থা যথেষ্ট ভালো। কিন্তু আমাদের বাংলাদেশি আর ইন্ডিয়ানরা গিয়ে সেখানে বিশাল রমরমা কোচিং সার্ভিস খুলে বসে আছে। দেখতে দেখতে অস্ট্রেলিয়ার বারোটা বেজে যাচ্ছে। আমি শুধু আশা করে থাকি হয়তো একেবারে হাতেগোনা অল্প কিছু ছেলে মেয়ে আমাদের কথা বিশ্বাস করে কোচিং ফোচিং করবে না, তারা ফেসবুক গুগলে চাকরি করার জন্য বিদেশ চলে যাবে না, দেশটা তারাই চালাবে।

প্রশ্ন: না স্যার, আপনার উপর আমি মোটেও রাগ করিনি। মাঝে মাঝে একটু মন খারাপ হয়েছে। কিন্তু রাগ করিনি। রাগ করা, অভিমান করা- এসব ছেড়ে দিয়েছি। একদিন হটাৎ বুঝতে পারলাম এসব রাগ, অভিমান করা মানে অন্যের উপর নিজের জীবন খানিকটা ছেড়ে দেওয়া। বুঝিয়ে দেওয়া যে, “দেখ, তোমার উপর আমি অনেক ভরসা করি! আমার হাসি কান্না নির্ভর করছে তোমার কথা, কাজের উপর!” এটা আমি কেন করব? আমার জীবন আমি নিজের মতো করে চালাব। কে কী বলল, তাতে আমার কিছু যায় আসে না। কারো কাছে আমার আর কোনো প্রত্যাশা নেই বললেই চলে! মানুষের কত ধরনের জীবন আছে, তার সবটা কি আমি জানি ? জানি না। অতএব, মাথা ঘামানো বাদ দিয়েছি। জীবনে আনন্দ কত রয়ে সয়ে আসে, তাই না স্যার? তবু আপনার জীবন আনন্দে ভরে উঠুক, এটাই আমার চাওয়া। যে মানুষটার লেখা আমার শৈশবে, কৈশোরে, এমনকি এখন পর্যন্ত আনন্দ দিচ্ছে, তার জন্য এটুকু তো চাইতেই পারি! প্রকৃতি রায়, প্রথম বর্ষ, ঢাকা বিশ্ববিদ্যালয়।(ডিপার্টমেন্টের নামটা ইচ্ছে করে দিলাম না! যদিও আমার নাম দেখেই চিনে ফেলবে, যদি ওরা এখানে থেকে থাকে!)
উত্তর: মুশকিল হচ্ছে আমি বুঝতে পারছি না ঠিক কী জন্য তুমি হঠাৎ করে এতো সিরিয়াস হয়ে গিয়েছ! তোমার বড় একটা ক্ষতি হয়েও আমার জীবন আনন্দে ভরে উঠুক এরকম ইঙ্গিত দিয়েছ, কেন? আমার স্মৃতি শক্তি যে গোল্ডফিশের মত সেটা কি জান?

প্রশ্ন: তুর্যয় পাল, ফেনী। স্যার, আমি এবার শিক্ষার মাধ্যমিক পর্যায় পেরিয়ে উচ্চ মাধ্যমিক পর্যায়ে অর্থাৎ একাদশ শ্রেণিতে উঠেছি। কিন্তু আমার কেমন জানি পড়ালেখাসহ আরও বিভিন্ন বিষয় উলট – পালট মনে হয়! আপনি যদি কোনো উপদেশ দেন তাহলে উপকৃত হবো মনে হয়।
উত্তর: উপদেশ দিয়ে যদি কোনো কাজ হত তাহলে দিতাম। উপদেশ দিয়ে কোনো কাজ হয় না, নিজের দায়িত্ববোধ দিয়ে নিজের কাজ করতে হয়! উৎসাহ দিলে মাঝে মাঝে কাজ হয়। দিব উৎসাহ?

প্রশ্ন: (জিয়াদ হোসেন,রূপসা,খুলনা)আলহামদুলিল্লাহ–একটু কড়া নাড়ার জন্য।ভেবেচিলাম ভুলে গেছেন।তবুও কয়েকদিন পরপর ঢুঁ মেরে দেখেছি,আর হতাশ হয়েছি।আশা করি ব্যস্ত সময় দ্রুত শেষ হবে,আবার আমাদের মাঝে ফিরে আসবেন।আর মাঝে মাঝে একটু কড়া নাড়বেন–কী অবস্থা তা জানিয়ে।ধন্যবাদ।(ছোট্ট একটা খবর আছে , আরেকদিন বলব)।টা টা bye bye, আবার যেন কথা হয়।
উত্তর: এখন তো করোনা শেষ! কাজেই এই ভারচুয়াল জগতটা ছুড়ে ফেলে দিলে কেমন হয়? সবার সাথে সামনা সামনি দেখা হবে, কথা হবে! তাহলে আর কাউকে কড়া নড়তে হবে না।

প্রশ্ন: না না, তা হয় না। আমার মতো পাঠকরা আপনাতে শান্তিতে থাকতে দেবে, তা হয় না। আগামী বছর ‘আমার ডেঞ্জারাস মামি’ এর আরেকটা বই বের করবেন। জ্বি, ঠিকই পড়েছেন, আরেকটা। আপনাকে জ্বালাতে হবে না? রুম্মান জারা, চাঁদপুর।
উত্তর: না আ আ আ আ আ আ আ আ আ আ আ আ আ আ আ আ আ আ আ আ আ … … … …

প্রশ্ন: স্যার আমিও ভেবেছিলাম আপনি ডুবই দিয়েছেন,গতকাল আপনার লেখা পড়ে তো নিশ্চিতই হলাম।..তো যাই হোক,আমি এই সাইটের পুরনো প্রশ্ন-উত্তর গুলো পড়ছিলাম।একটা প্রশ্নে কেউ লিখেছিল যে সে পৃথিবীর প্রায় সাতশ কোটি মানুষের মাঝে আপনাকে সবচেয়ে বেশি ভালোবাসে।উত্তরে আপনি লিখেছিলেন আপনার নিজেকে অপরাধী মনে হচ্ছে, সে আপনাকে সাতশ কোটি মানুষের মধ্যে সবচেয়ে বেশি কেন ভালোবাসে, আপনি কী করেছেন আমাদের জন্য?..এই সাইটে এসে আপনাকে অনেক রকম প্রশ্ন,কথা জিজ্ঞেস করি। কিন্তু কখনো আপনাকে আপনার লেখা বই পড়ে আমার মনে যে ধরণের ইতিবাচক মনোভাব তৈরি হয়,প্রফুল্ল মনোভাব তৈরি হয়,আরো নানান উপকার হয় (আসলে আমার ভাষায় সুন্দর শব্দ ব্যবহার করে কীভাবে কথাটাকে আরো সুন্দর,মনোমুগ্ধকর করে তুলতে হয় সেই জ্ঞান নেই ,তাই “..নানান উপকার হয়..” এরকম লিখতাম না,কথাটা কেমন যেন বেমানান!)সেসব কিছুর জন্য কখনো ধন্যবাদ দেয়া হয়নি।আপনি আমাদের জন্য অনেক অনেক কিছু করেছেন স্যার।আপনাকে সকল কিছুর জন্য অনেক অনেক ধন্যবাদ আর ভালোবাসা।
উত্তর: থ্যাঙ্কু! শুনে খুশি হলাম যে আমার বইগুলো কোনো কাজে আসছে!

*************************************************************************

ফেব্রুয়ারি ২৭, ২০২২

আমি জানি তোমরা যারা আমার এই ওয়েবসাইটটাতে ঘোরাঘুরি কর তারা আমার উপর অনেক রাগ করে আছ কারণ আমি অনেকদিনের জন্য ডুব দিয়েছি! কিন্তু বিশ্বাস কর, আমি অনেকদিন থেকে নিঃশ্বাস ফেলার সময় পাই না। আমি এত ব্যস্ত যে এই বছর আমি ভালো করে লিখতেও পারি নি! এত সুন্দর একটা বই মেলা হচ্ছে কন্তু ভালো করে সেখানেও যেতে পারছি না। আমার শধু ইচ্ছা করে আমি যদি তোমাদের মত ছোট একজন মানুষ হয়ে থাকতাম আমার যেটা ইচ্ছা করে শুধু সেই কাজগুলো করতে পারতাম তাহলে কত মজা হত। কেন যে ছাই বড় হয়ে গেলাম, কেন যে এত দায়িত্ব আমি বুঝতে পারি না।

আমি কিন্তু নিয়মিতভাবে এই ওয়েবসাইটটাতে ঢুকে দেখি তোমরা কে কি লিখেছ। কে রাগ হয়েছ, কে দুঃখ পেয়েছ, কে হতাশাতে ভুগছ আমি সব জানি, শুধু তোমাদের প্রশ্নের উত্তর দেওয়ার সময় করতে পারছি না। সত্যি কথা বলতে কি তোমরা এত সুন্দর আর আন্তরিক ভাবে আমাকে লিখ যে পড়ে আমার মনে হয় আমি তোমাদের সাথে কথা বলছি, আর আমার মন ভালো হয়ে যায়। বলতে পার তোমাদের লেখাগুলো পড়া আমার আনন্দের একটা বড় উৎস। কাজেই তোমাদের মনের কথাগুলো আমাকে লিখতে থাক, আমি সেগুলো পড়ে আনন্দ পেয়ে যেতে চাই।

আর এমন যদি হয়, খুবই ইমার্জেন্সি একেবারে বাড়াবাড়ি কিছু হয়ে যাবে উত্তর না পেলে তাহলে m.z.iqbal@mzi.rocks  এখানে একটা ইমেইল পাঠাতে পার, আমি চেষ্টা করব উত্তর দিতে, ইংরেজিতে, কারণ আমি যখন বাংলায় লিখি, সবসময় কাগজে বল পয়েন্ট কলম দিয়ে লিখি! কম্পিউটারে বাংলা লিখতেই পারি না।

সবাই ভালো থেকো, পাজী বেয়াদপ ফাজিল হতভাগা করোনা দেশ থেকে চিরদিনের জন্য দূর হয়ে যাচ্ছে, আবার সবার সাথে সামনাসামনি দেখা হবে।

************************************************************************

নভেম্বর ৬ ২০২১

যারা অটোগ্রাফ চেয়েছ, প্রশ্ন-উত্তর শেষে সেগুলো দিয়ে রেখেছি, খুঁজে নাও প্লিজ।

প্রশ্ন: স্যার আপনি evolution নিয়ে একটা ভালো বড় বই লিখেন। প্লিজ। আমাদের দেশে এই evolution কে মানুষ just a theory, Missing link, Macro evolution হয় নাহ শুধু micro হয়, অনেক আজগুবি আর বাজে কথা বলে,.. … …
আপনি একটা বড় বই লিখুন আপনার বই যেটি রয়েছে সেটি খুব ছোট । Evolutionary mechanism , বিভিন্ন যুগ , প্রাণীদের বিবর্তন নিয়ে আরো ভালো বই চাই
উত্তর: Evolution নিয়ে লিখতে আমার কোন আপত্তি নেই কিন্তু তুমি যে কারণটি দেখিয়েছ সেটি নিয়ে আমার কোন মাথা ব্যাথা নেই। যারা Evolution বিশ্বাস করে না তাদেরকে তাদেরকে তাদের মত থাকতে দাও। বিজ্ঞান তো আর ফুটবল খেলা না, রাজনীতিও না যে এখানে বেশি সাপোর্টার নিয়ে দল ভারী করতে হবে। কেউ বিশ্বাস না করলেও বিজ্ঞান থাকবে, এটা তো আর ভোটাভুটি করে নির্বাচিত হয় না, পরীক্ষা নিরীক্ষা করে দাঁড়া করানো হয়! যারা বিজ্ঞান জানে না তারা পৃথিবীর সৌন্দর্যটা উপভোগ করতে পারে না, তাতে অন্যদের সমস্যা কী?

প্রশ্ন: আচ্ছা আপনার কোন প্রিয় কবিতা আপনি পুরো মুখস্থ করেছেন? আমার এখন পযর্ন্ত সবচাইতে প্রিয় কবিতা হলো কাজী নজরুল ইসলামের ” মানুষ “!!!!! আমি এটা সম্পূর্ণ মুখস্থ করেছি!!!! আর না দেখে আবৃত্তিও করতে পারি!!!!! “কবর” আমার পছন্দ হয়েছে অনেক। তাই ওটাও একটু একটু করে মুখস্থ করছি!! আপনি প্লিজ মানুষ কবিতা টা আজকে আবার শুনবেন!!! অনেক সুন্দর……. কবিতাটা!!!!!!!! তোহুরা ইসলাম বাঁধন ( class 8, টাঙ্গাইল )
উত্তর: আমার অনেক কবিতা জানা ছিল (মুখস্ত শব্দটা আমার জঘন্য লাগে) যখন বয়স কম ছিল, ব্রেন্টা অনেক তেজী ছিল। প্রিয় কবিতা কয়েকবার পড়লেই মনে থাকতো! এখনও অনেক কবিতা মনে আছে। যখন নিজে নিজে বিড়বিড় করে আবৃত্তি করি একটা দুইটা শব্দ বা বাক্য মাঝে মাঝেই ছুটে যায়! হ্যাঁ, নজরুলের ‘মানুষ’ খুব সুন্দর কবিতা।

প্রশ্ন: wasiur rahman, rajshahi. sir, apni ki amake ekta otograph dite paren? er age apni amake otograph diyechen kina, janina. ami apnar ondho vokto. amar kache apnar 65+ boi ache. ei computere bangla type korte somossa hoy to, tai rag korben na. doya kore ei prosner uttor diye ekjon 11 bochor boyosi cheler asha purno korben. accha sir, amar baba bolen apni naki islamer biruddhe lekhen? sotti naki? ami obosso bishwash kori na. sir, apnar boyosh koto? apni dari rakhen na keno? apni ki ki islamic boi likhechen? amar babake dekhate hobe to, tai. amar baba abar ru er botanyr professor, tai ja ta bolen, amake boken!
উত্তর: এই যে নাও আমার অটোগ্রাফ। খুশি? কম্পিউটারে বাংলা লিখতে পার না বলে ইংরেজী অক্ষরে লিখেছ বলে রাগ হইনি, তবে আমি চাই ইংরেজী অক্ষরে যখন লিখবে তখন ইংরেজীতে লিখবে!

প্রশ্ন: আপনি সবাইকে অটোগ্রাফ দিচ্ছেন (যারা যারা চেয়েছিল)। আমাকেও একটা দেন প্লিজ। শেখ ফারিহা তাসনিম স্নেহা, লালমনিরহাট।
উত্তর: এই যে আরেকটা অটোগ্রাফ।

প্রশ্ন: মেহেরুন নিসা ইলেন বলছি চাঁদপুর থেকে।(স্যার দয়া করে আমার নামটা মেহেরুন্নেছা পড়বেন না। সবাই আমার নাম তাই বলে কি না!) সালাম নিবেন। কবি হলে বলতে পারতাম আজকের দিনে এই পৃথীবিতে যত ফুল ফুটেছে সব গুলোর শুভেচ্ছা নিন। যাই হোক, আমি দশম শ্রেণির একজন শিক্ষার্থী।ক্লাস সেভেনে পড়ার সময় স্কুলের লাইব্রেরিয়ান ম্যাম জোর করে হাতে একটা বই ধরিয়ে দিলেন পড়ার জন্য (ভাব্বেন না যে আমি বই পড়ি না। আসলে আমি আমার ক্লাসের সবচেয়ে বেশি বই পড়ুয়া মেয়ে। কখনোই টেক্সট বুক এর কথা বলছি না)। আমার নজর আমার হাতের বইটার দিকে ছিলো না ছিলো পাশে দাঁড়িয়ে থাকা বান্ধবীর হাতের বইটার দিকে(শিবরাম চক্রবর্তী এঁর লেখা বাড়ি থেকে পালিয়ে) আপনি কি এই বইটা পড়েছিলেন? । কি আর করা মুখ বেজার করে বাসায় আসলাম বইটা হাতে নিয়ে।কে জানতো এই বই আমি তিন বার পড়ে ফেলবো! আমার হাতের বইটির নাম দীপু নাম্বার টু।লেখক আমার বর্তমান প্রিয় লেখক … … … সাথে চাই একটা অটোগ্রাফ (পুরো নাম লেখতে হবে কিন্তু!)। প্লিইইইইইইইইইইইইইইইইইইজ!!!!! ইসস যদি আপনার সাথে একবার দেখা হতো তাহলে অবশ্যই অবশ্যই একটা ছবি তুলতাম।আশা রাখি দেখা হবে।আমার জন্যে দোয়া রেখেন।উত্তরের অপেক্ষায় রইলাম। মেহেরুন নিসা ইলেন। শাহরাস্তি,চাঁদপুর।
উত্তর: তোমার বিশাল প্রিয় লেখকদের তালিকায় আমিও ঢুকে পড়তে পেরেছি জেনে ভালো লাগল। দীপু নাম্বার টু আমার প্রথম দিকে লেখা বই- বইটার বয়স প্রায় পঞ্চাশ, দীপু এখনও বাচ্চা আছে, কী মজা তাই না? এই যে তোমার অটোগ্রাফ। হ্যাঁ কোনো একদিন নিশ্চয়ই দেখা হবে, তখন ছবি তুলে নিতে পাড়বে।

প্রশ্ন: স্যার, টুনটুনি ও ছোটাচ্চুর সিরিজে সর্বশেষ প্রকাশিত হয়েছে কোনটি? এই সিরিজে আরো বেশি বেশি লিখুন। আপনার সব রচনা ই অসাধারণ। তবে এটা আরো বেশি। ( সাদিয়া আফরিন, দশম শ্রেণি)
উত্তর: এই সাইটেই বইয়ের তালিকা দেয়া আছে, দেখে নেবে প্লীজ! এতোগুলো লিখেছি আর তাদের নাম এতো কাছাকাছি যে আমার নাম মনে থাকে না!

প্রশ্ন: প্রিয় স্যার, নতুন কি কি বই বের হচ্ছে ২০২২ সালের বইমেলায়? অনন্ত আহমেদ, ঢাকা ।
উত্তর: একটা বের হয়েছে আর একটা বের হবে হবে করছে। অন্যগুলো মাথায়, দেখি কাগজে নামাতে পারি কিনা।

প্রশ্ন: শ্রদ্ধেয় স্যার, আসসালামু আলাইকুম। কেমন আছেন? আমার নাম আফিফা রহমান মুনিরা। আমি আপনার একজন ভক্ত। আমি তেমন গুছিয়ে কথা বলতে পারি না, গুছিয়ে লিখতেও পারি না। তাই আমার লেখাটা আপনার কাছে boring মনে হতে পার। কিন্তু স্যার আপনি যদি ধৈর্য্য ধরে আমার এই লেখাটা পড়েন আর কষ্ট করে উত্তর দেন তাহলে একজন ভক্ত হিসেবে আমার অনুভূতি কেমন হতে পারে বুঝতেই পারছেন।
আপনার ওয়েবসাইটে একটা প্রশ্ন করেছিলাম। আপনি তার উত্তর দিয়েছেন, তাই আপনাকে অসংখ্য ধন্যবাদ। কিন্তু উত্তরটা বুঝতে আমার কিছুটা সমস্যা হচ্ছে। … … … চলন্ত বাসের জানালা দিয়ে ভেতর থেকে বাইরে স্প্রে করা আর বাইরে থেকে ভেতরে স্প্রে করা হলে একই ঘটনা ঘটে। স্যার এই দুটো ঘটনাই কি আপেক্ষিক গতির জন্য? স্যার Please, কষ্ট করে আমার প্রশ্নের উত্তর দিবেন, Please? স্যার আমাকে কি একটা অটোগ্রাফ দিবেন, PLEASE? ফুলপুর, ময়মনসিংহ।
উত্তর: হ্যাঁ, দুটো ঘটনাই আপেক্ষিক গতির জন্য। এই নাও অটোগ্রাফ।

প্রশ্ন: স্যার, আস-সালামু আলাইকুম। জানিনা প্রকৃতপক্ষেই, মুহম্মদ জাফর ইকবাল স্যার এর কাছে লেখাটি পৌঁছাচ্ছে কিনা ? আমি কোন রাজনৈতিক দলের ব্যাক্তি কিংবা সমার্থক নই; একজন অতি সাধারন মানুষ। কিছু তথ্য আছে যেগুলো জানতে ইচ্ছা করে এমন কারো কাছ থেকে যিনি নির্ভরযোগ্য এবং বিশ্বাসযোগ্য। তথ্যগুলো হতে পারে আমাদের মুক্তিযুদ্ধ বিষয়ক, আমাদের মুক্তিযুদ্ধের পরের কিছু অংশের বিষয়ক, হতে পারে বিজ্ঞান বিষয়ক, আবার হতে পারে শুধুই কিছু সাধারন স্নেহ সম্মানীয়-স্নেহ এর কথা। আমি চাচ্ছিনা, যাকে বলতে চাচ্ছি, তিনি ছাড়া অন্য কেউ আমার কথাগুলো জানুক। এইজন্যই আমার নিশ্চিত হওয়া দরকার যে, মুহম্মদ জাফর ইকবাল স্যার এর কাছেই লেখাটি পৌঁছাচ্ছে। আমি কিভাবে নিশ্চিত হতে পারি? ধন্যবাদ। বিনীত, মোঃ আব্দুল্লাহ আল মাহবুব শিরোইল, বোয়ালিয়া, রাজশাহী-৬১০০
বিঃদ্রঃ যদিও আমার মত অভাজনকে সময় দেওয়া আপনার জন্য একান্তই অনর্থক হওয়ার কথা। সে ক্ষেত্রে আমি আশা করি আমার এই অন্যায় সাহসকে আপনি ক্ষমাসুন্দর চোখে দেখবেন ।
উত্তর: তুমি যদি এই সাইটটি দেখে বুঝতে না পার এটা সত্যি না ভুয়া সাইট তাহলে তোমাকে কেমন করে বোঝাব বল! মাঝে মাঝে কিছু জিনিস বিশ্বাস করে নিতে হয়, জীবন তাহলে অনেক সহজ হয়! (নিজেকে কেন অভাজন বলছ? পৃথিবীতে কেউ অভাজন না।)

প্রশ্ন: আপনি একজনের প্রশ্নের উত্তরে লিখেছেন বাচ্চা কাচ্চাদের ক‍্যাচ ম‍্যাচের মধ‍্যে একটু বড়দের কথা শুনতে ভালো লাগে!!!!!!!!!!!!!!!!!!!!!!!!!!! আমারা খালি ক‍্যাচ ম‍্যাচ করি!??????হুমমমমম?????? ( বাঁধন, Class 8 টাঙ্গাইল )
উত্তর: যারা ক্যাঁচম্যাচ করে তারা কখনো টের পায় না যে তারা ক্যাঁচম্যাচ করছে।

প্রশ্ন: স্যার আসসালামু আলাইকুম। কেমন আছেন? আপনার ই-মেইল ঠিকানা কোনটি? এখানে আমি দুটি ই-মেইল ঠিকানা পেয়েছি। দুটির মধ্যেই ই-মেইল পাঠিয়েছি। কিন্তু কোনো উত্তর পাইনি। প্রথমটি হচ্ছে m.z.iqbal@mzi.rocks দ্বিতীয়টি হচ্ছে m.zafar.iqbal@mzi.rocks ইরা, ময়মনসিংহ।
উত্তর: প্রথমটা ঠিক। আবার পাঠাও। উত্তর না পেলে আবার। না পেলে আবার… … (তোমারটা কেন পেলাম না জানি না। মাঝে মাঝেই তো আমি এই ঠিকানায় মেইল পাই।)

প্রশ্ন: স্যার,আশা করছি ভাল আছেন,সুস্থ আছেন৷স্যার,আপনার কাছে কয়েকটা প্রশ্ন ছিল। ক।এলিয়েনকে আমরা শুধু এভাবে কল্পনা করি কেন-মাথা মোটা,গলা চিকন,সবুজ রঙের শরীর,চোখ বড়ো বড়ো,মুখটা লম্বা প্রায় ত্রিভুজাকৃতি।এরা কী অন্য রকম আকৃতির হতে পারে না.UFO- গুলোইবা এরকম কল্পনা করা হয় কেন-একটা উপুর করা বাটির উপর আরেকটা বাটির আকৃতির মতো.UFO-গুলোতো পিরামিড আকৃতির কল্পনা করতে পারি তাই না-কিংবা অন্য কোনো রকম. খ৷ভূতকে টেস্ট টিউবে ভরে পরীক্ষা করা সম্ভব হয়নি বিধায় ,আমরা ভূত বিশ্বাস করি না৷তাই বলে কি আমরা মন বিশ্বাস করব না-মন’কে ও তো টেস্টটিউবে ভরে পরীক্ষা করা সম্ভব হয়নি৷ আজ আর নয়৷আশা করি সবগুলো প্রশ্নের উত্তর পাবো৷Please,প্রশ্নগুলোর উত্তর দিয়েন৷ধন্যবাদ৷ জিয়াদ হোসেন রূপসা,খুলনা৷
উত্তর: না, সবাই এরকম এলিয়েন কল্পনা করে না, অন্যভাবেও করে। তোমার বর্ণনায় এলিয়ান আগের যুগের সিনেমা থেকে এসেছে! আজকালকার এলিয়ানে অনেক বৈচিত্র্য আছে। সব কিছুর অস্তিত্ত্ব প্রমান করার জন্য তাকে টেস্ট টিউবে ঢুকাতে হয় মস্তিস্কের প্রক্রিয়ার একটা প্রতিফলন হচ্ছে মন। তাই তুমি ওষুধ দিয়ে বিষণ্ণ মনের পরিবর্তন করতে পার। ভূতের বেলায় সেটা পেরেছ?

প্রশ্ন: স্যার, পৃথিবীটা এরকম স্বার্থপর কেন? তাসনিম, কুমিল্লা
উত্তর: অন্যদের যত ইচ্ছা স্বার্থপর হতে দাও, তুমি নিজে কখনো হয়ো না।

প্রশ্ন: স্যার, আমি সহজে কোনো গণিত-বিজ্ঞানের বিভিন্ন বিষয়, সমীকরণ ও সূত্র বুঝতে পারি না। দেখলেই মাথায় প্যাঁচ লেগে যায়। পড়ি ক্লাস এইটে কিন্তু কোনো বুদ্ধিভিত্তিক গাণিতিক সমস্যা মাথায় ঢুকে না। গণিত অলিম্পিয়াডের বাচ্চাদের প্রশ্নও সমাধান করতে পারি না। এই বিষয়ে আপনি কি কোনো সাহায্য করতে পারেন? আর, আপনি সবসময় বলেন ছোটবেলায় আপনি ভ্যাবলা টাইপের ছিলেন। আর আপনাদের সময় রিসোর্সও কম ছিলো। সেক্ষেত্রে আপনি কোন ক্লাস বা বয়স থেকে সত্যিকারের গণিত-বিজ্ঞান বুঝতে শিখেছেন? (যদি জানি বড় হয়ে তাহলে একটা সান্ত্বনা পাবো।) নাঈম, বাসাবো, ঢাকা
উত্তর: আমি ভ্যাবলা টাইপের ছিলাম সত্যি! কিন্তু তাই বলে গণিত, বিজ্ঞান এসব বুঝতে আমার কখনো কোন সমস্যা হয়নি। আমার ধারনা কোন একটা কারনে তুমি ধরেই নিয়েছ গণিত দেখলেই তোমার প্যাঁচ লেগে যাবে, এই জন্য আগেই নার্ভাস হয় যাচ্ছ! দেখো এই কারণটা দূর করতে পার কীনা।

প্রশ্ন: স্যার, গতবার আপনাকে একজন বলেছিল ‘কিশোর আলো’তে আপনার নামে একটা কবিতা প্রকাশ পেয়েছে। তারপর আপনি বলেছেন যে ওটা আপনার না। অনেকদিন আগে শুধু ছড়ার একটা বই প্রকাশ পেয়েছিল!চ ‘কিশোর আলো’র ছড়াটা আপনাকে দিচ্ছি! ( অনেক খোঁজাখুঁজি করেও নেটে আপনার ছড়ার বইটা পাচ্ছি না। আপনি একটু বলেন যে ছড়টা কি আপনি লিখেছিলেন? মানে আমার জানতে ইচ্ছা হচ্ছে যে ‘কেউ একজন’ কি শুধু আপনার নামটা ব্যবহার করেছে নাকি ছড়াটাও! )
দুজন বাচ্চার ঝগড়া চলছে
ডাবলিউ লেখা আছে একজন বলছে… … …
পুনশ্চ: ভুলবশত বেয়াদবী হয়ে থাকলে মাফ করে দিয়েন! আর আমার নাম প্রকাশ না করাই মনেহয় ভালো। (*** ঢাকা)
উত্তর: হ্যাঁ, এটা আমার লেখা ছড়া ছিল। কিশোর আলো হঠাৎ করে কেন এটা ছেপেছে আমি জানি না!

প্রশ্ন: আমার ইউটিউব চ্যানেলে একটা কমেন্ট করে যান স্যার ধন্য হয়ে যাব আমি , আমি অনেক গুলো কবিতা আবৃত্তির ভিডিও দিয়েছি স্যার, কিন্তু দুঃখের বিষয়, আমি সাবস্ক্রাইবার পাই না। আমার কবিতা কেউ দেখে না। আমি খুবই নিরুৎসাহিত হই… … … … অনেক প্রতিযোগিতায় অংশগ্রহণ করেছি। 2 বার জাতীয় পর্যায়ে প্রথম হয়েছি। কিন্তু কবিতা আবৃত্তির কোনো প্রোগ্রাম হয় না এখন। তাই এই ইউটিউব চ্যানেল খুলেছিলাম। কিন্তু সাবস্ক্রাইবার খুব কম। কেউ দেখে না আমার আবৃত্তি । আমি খুবই মর্মাহত স্যার। এখন আপনি যদি গিয়ে একবার দেখে আসেন, খুবই আনন্দ পাবো স্যার, একটা কমেন্ট করে আসবেন স্যার, অন্য কেউ না দেখুক আপনি দেখলে এবং কমেন্ট করলে অনেক খুশি হবো।প্লিজ স্যার দেখবেন। … … … নামটা গোপন রাখবেন), অষ্টম শ্রেণি, নোয়াখালী জিলা স্কুল।
উত্তর: শোন- আমি এই বিষয়গুলোকে খুব উৎসাহ দেই না! তুমি তোমার পছন্দের কাজ করে যাবে- অন্যরা কী বলল সেটা নিয়ে একবারেই মাথা ঘামাবে না! আমার কেন জানি মনে হয় সাবস্ক্রাইবার পাওয়ার বিষয়গুলো একটা ষড়যন্ত্র। দৈত্যকার সফটওয়ার কোম্পানিগুলো তোমাদের ব্যবহার করে এভাবে পয়সা কামাই করে! বিক্রি হওয়ার জন্য নিজেকে পণ্য বানিয়ে লাভ কী?

প্রশ্ন: ২০২২ সালে আপনার কি কি বই প্রকাশ হবে? আহনাফ প্রত‍্যয়, টাঙ্গাইল
উত্তর: এখনও জানি না। লেখার চেষ্টা করছি।

প্রশ্ন: আমি জয়পুরহাটের মালিহা। চিনতে পারছেন? আমি না নিজেকে হারিয়ে ফেলছি। সারাক্ষণ মনে হচ্ছে চোরাবালির মধ্যে ডুবে যাচ্ছি। আমি না আর পারছি না। বাবা মা চাচ্ছে সব বিশ্ববিদ্যালয়ে চান্স পাই। বলুন, একজনের পক্ষে এটা একটু বাড়াবাড়ি হয়ে যায় না? আমি কি একসাথে অনেক জায়গাতে পড়তে পারব? … … … … আমার বাড়ি দেশের উত্তর-পশ্চিম প্রান্তের জয়পুরহাটে। এখান থেকে আমাকে আজ চট্টগ্রাম যেতে হচ্ছে পরীক্ষা দিতে। যেতেই দুদিন লেগে যাবে, ফিরতে ওরকম দুদিন। কেউ বুঝতে চাচ্ছে না এভাবে আমার শক্তি সময়ের অপচয় হচ্ছে। আমার যা টার্গেট, সেটার জন্য প্রস্তুত হতেও তো কিছু সময় দরকার, তাই না? মাঝে মাঝে মনে হয় যে এতকিছুর চাপে নিজের স্বপ্নটাই হারিয়ে ফেলবো। বাস্তবতাটা হল, এই দেশে স্বপ্ন টিকিয়ে রাখা আর বাস্তবায়ন করার মতো কষ্টকর আর কিছুই নেই। আপনারা বড় বড় শিক্ষক, আপনারা এই বাজে সিস্টেমটার পরিবর্তন করতে পারেন না?
উত্তর: ভর্তি পরীক্ষা দেওয়ার জন্য ছাত্রছাত্রীদের যে কী পরিমান কষ্ট হয় সেটা আমার থেকে ভালো করে কেউ জানে না। ইচ্চছা করলেই বিশ্ববিদ্যালয়ের শিক্ষকেরা এটা দূর করে দিতে পারে, কিন্তু তারা সেটা করে না। কেন করে না সেটাও আমি জানি সেই জন্য আমা দুঃখটা সবচেয়ে বেশি। সিস্টেমের সমস্যা অবশ্যই আছে। ছাত্রছাত্রীদের বিশেষ করে অভিভাবকদেরও মনে হয় বিষয়টা বোঝায় একটু সমস্যা আছে। পছন্দের বিষয় না পেলেও যে বিশ্ববিদ্যালয়ের একটা ভালো শিক্ষা নিয়ে জীবন তৈরি করা যায় সেটা কী সবাই জানে? আমি আশা করি তুমিও যেন বিশ্ববিদ্যালয়ের শিক্ষা পেয়ে নিজের জন্য সুন্দর একটা জীবন গড়ে তুলতে পার।

প্রশ্ন: আসসালামুয়ালাইকুম স‍্যার। আমার প্রশ্ন হচ্ছে যে, সময় পরিভ্রমণ করে ভবিষ্যতে যাওয়া যায়। কিন্তু এর জন্য অনেক গতিতে ভ্রমণ করতে হয়। এতো গতিতে ভ্রমণ করার জন্য কি কোনো যন্ত্র আছে? থাকলে তার নাম কি এবং কিভাবে তা কাজ করে? ধন্যবাদ স‍্যার। মোঃ মুশফিক রহমান, ৯ম শ্রেণী, নারায়ণগঞ্জ।
উত্তর: তুমি কোথায় এই তথ্য পেলে যে অনেক গতিতে গেলে সময় পরিভ্রমন করে ভবিষ্যতে যাওয়া যায়? এটা সঠিক তথ্য নয়।

প্রশ্ন: আমার নাম বন্যা বড়ুয়া। আমি চট্টগ্রামে থাকি।স্যার,,কারোর কোনো কথায় বা ব্যবহারে যদি কখন খারাপ লাগে তাহলে কি তা তাকে বলা উচিত,,তার কথায় আমি কষ্ট পেয়েছি?? কিংবা ভালো ব্যবহারে যে আমি খুশি হয়েছি তাও কি বলা উচিত??
উত্তর: ভালো ব্যবহারে খুশি হয়েছ সেটা তো বলতেই পার। খারাপ ব্যবহারে কষ্ট পেয়েছ সেটা কাকে কেমন করে বলছ এই ব্যাপারগুলো মনে হয় একটু দেখে শুনে নিতে হবে। তুমি যেভাবে বলবে সেটাও যদি পাল্টা খারাপ ব্যবহার হয়, সেটার জন্য যদি অন্যের মন খারাপ হয়, তাহলে?

প্রশ্ন: স্যার, একটা সমস্যায় পড়েছি। সমস্যাটা আপনার হয় কিনা জানিনা। আমরা যারা চশমা ব্যবহার করি, মাস্ক পরা তাদের জন্য খুব সমস্যার। চশমা ও মাস্কের কম্মিনেশন খুব বিরক্তিকর। মাস্ক পরলে চশমা ঘোলা হয়ে যায় আপনিও তো চশমা ব্যবহারকারী। আপনি কিভাবে এটা থেকে নিস্তার পান? বিভিন্ন জায়গায় যেতে মাস্ক ও চশমা দুটোই কি একসাথে পরেন? আমি তো এখন কোথাও যেতে চশমা খুলে যাই। খুব সমস্যায় পড়েছি। চশমা ছাড়া মাথাব্যথা করে। এ থেকে চশমাওয়ালা দের রক্ষার উপায় কি স্যার?( হেসে ফেলবেন না! আপনিও যেহেতু চশমা পরেন তাই আমার দুঃখ নিশ্চয়ই বুঝবেন!)এটার কোনো বিজ্ঞানসম্মত সমাধান চাই
আর সাথে একটা অটোগ্রাফ ও দেবেন স্যার, আপনার সবচেয়ে বড় ভক্ত, পদার্থবিজ্ঞান প্রেমী অরিত্র বণিক আমাকে অটোগ্রাফ দেবেন কিন্তু
উত্তর: মাস্ক পড়ার সময় লক্ষ্য রেখ নাকের কাছাকাছি অংশটাতে যেন ফাঁক না থাকে তাহলেই চশমা ঘোলা হবে না! মাস্ক যেন নাকে চেপে বসে থাকে সেজন্য আজকাল মাস্কের ভেতর ধাতব একটা পাত থাকে, সেটা দেখেছ?( আশা করছি শীগ্রই মাস্ক সম্পর্কে এই অমূল্য জ্ঞানগুলোর আর কারো কোনো প্রয়োজন থাকবে না, আমরা সবাই মাস্ক ছাড়া ঘুরে বেড়াতে পাড়ব।) দিলাম তোমার অটোগ্রাফ।

প্রশ্ন: 1905 সালে যে থিওরি দিলাম, সেটা দিয়ে মিতু তিতু টাইমমেশিন কিভাবে বানাল? তারা এত বেশি বেগে কিভাবে চলতে পারল? আমাকে এটার বিজ্ঞানসম্মত ব্যাখ্যা দাও। এসব জিনিস কি লেখ তুমি? তুমি যে লিখলে ব্লাকহোলের বাচ্চা, কিন্তু এটা তো অবৈজ্ঞানিক কথা! ব্লাকহোল কোনো মানুষ নয়, একটা মৃত নক্ষত্রের মহাকর্ষ বলের কারণে তৈরি একটা গুহা মাত্র! সেটা সবকিছুই টানে। তুমি তার বিয়ে দিলে কেমনে? তারপর তার আবার বাচ্চাও হল! কি আজিব! আশ্চর্য কথা! আচ্ছা তুমি এসব আইডিয়া কোথায় পাও? তোমাকে তো পদার্থবিজ্ঞানে (অবশ্য কাল্পনিক পদার্থবিজ্ঞান!) 10টা নোবেল দেওয়া উচিত! আমি যদি নোবেল কমিটির লোক হতাম, তোমাকে তাই দিতাম! আলবার্ট আইনস্টাইন (ভুয়া), জার্মানি( এটাও ভুয়া!)
উত্তর: ভূয়া আইনস্টাইন, তুমি কী জান, আসল আইনস্টাইন বলেছিলেন, “জ্ঞান থেকে কল্পনা বেশি গুরুত্বপূর্ণ”? আমি সেটা বিশ্বাস করি বলে আমার দোষ হয়ে গেল?

প্রশ্ন: আমার কিছু ‘বৈজ্ঞানিক’ সূত্র আছে স্যার। এগুলো আমার আবিস্কার!
১. আপনি যদি ভোর 6: 00 টায় উঠে অতিরিক্ত 5 মিনিট ঘুমানোর জন্য চোখ বন্ধ করেন, চোখ খুলে দেখবেন 7:45 বাজে! অর্থাৎ, এখানে 5মিনিট=105 মিনিট, আর এখানে কোনো প্রকার গতি ছাড়াই সময় প্রসারণ হয়ে যায়! ২. যদি ক্লাসে বসে বোরিং লেকচার শুনতে শুনতে 12:30এ 5 মিনিটের জন্য চোখ বন্ধ করেন, চোখ খুলে দেখবেন, 12: 31 বাজে। অর্থাৎ এখানে সময় সংকুচিত হয়ে যায়! ৩. পরীক্ষার হল সময়ের আপেক্ষিকতার সবচেয়ে বড় উদাহরণ! এখানে স্যারদের মনে হয় ঘড়ির কাটার 1 মিনিট = 10 মিনিট, অর্থাৎ সময় প্রসারিত, আবার, পরীক্ষার্থী মানে আমাদের মনে হয় 1 মিনিট= 10 সেকেণ্ড, সময় এখানে সংকুচিত! ৪. যত বিজ্ঞানসম্মত কথা বলবেন তত বেশি সমস্যায় পড়বেন।
উদাহরণ : (1)আমি একবার ডাক্তার দেখাতে গিয়েছিলাম স্যার। ডাক্তার বললেন, ” তোমার ওজন কত?” আমি আমার ভর 30 কেজি কে ৯.৮ দিয়ে গুণ করে বললাম 294 নিউটন। ডাক্তার রেগে অস্থির! বললেন, ” আমার সঙ্গে মশকরা? আমার এত সময় নেই! তাড়াতাড়ি বল, সত্যি করে বল, তোমার মত হ্যাংলা প্যাংলা রোগা ছেলের ওজন কিছুতেই এত বেশি হতে পারে না!”৷ আমি অনেক বোঝাতে চেষ্টা করলাম, কিন্তু ব্যার্থ হয়ে বললাম, আমার ওজন 30 কেজি! কি করব বলুন!
… … … মৃগাঙ্ক, শ্রীমঙ্গল থেকে
উত্তর: হা হা হা! তোমার সূত্রগুলো ভালো- তবে তোমার বেশ সূত্র তোমার আগেই অন্যরা বের করে রেখেছে। তোমার ৪ নম্বর সূত্রটা মনে রেখে চল, তাহলেই পাব্লিকের মার না খেয়ে জীবনটা পার করতে পারবে।

প্রশ্ন: আপনার প্রিয় বিজ্ঞানী কে স্যার? আমার প্রিয় 3 জন। আইজ্যাক নিউটন, আলবার্ট আইনস্টাইন ও গ্যালিলিও গ্যালিলি। তিনজনই জীবনে অনেক কষ্ট করেছেন। গ্যালিলিও গ্যালিলিকে যাবজ্জীবন কারাদণ্ড দেয়া হয়েছিল আমাকে একটা অটোগ্রাফ ও দেবেন কিন্তু স্যার, আর আশীর্বাদ করবেন যেন আমি বিজ্ঞানী হতে পারি। গুণাংক বণিক, শ্রীমঙ্গল।
উত্তর: তোমার তিনজন বিজ্ঞানীই অনেক বড় বড় বিজ্ঞানী। তবে আইজ্যাক নিউটন তো তেমন কষ্ট করেননি। আইনস্টাইনের ব্যক্তিগত জীবন নিয়ে সমস্যা ছিল কিন্তু গ্যালেলিওর মত নির্যাতন তাকে কখনও সহ্য করতে হয়নি।

প্রশ্ন: স্যার, আপনার স্ত্রী কি বাংলায় কথা বলেন? যদি বলে থাকেন, তবে সাস্ট নিয়ে যে বইটা উনি লিখেছেন যেটা ইংরেজিতে কেন লিখেছেন? অন্বেষা, সেকেন্ড ইয়ার, জার্নালিজম, ডিইউ।
উত্তর: খুবই ইন্টারেস্টিং প্রশ্ন। কেউ ইংরেজিতে বই লিখলে ধরে নিতে হবে সে বাংলায় কথা বলে না?

প্রশ্ন: আচ্ছা স্যার ,আমি পড়েছি ট্রান্সফরমারের সেকেন্ডারি কয়েলের ভোল্টেজ বাড়ানো হলে তড়িৎ প্রবাহ কমে ( কারণ ক্ষমতা ধ্রুব থাকে) কিন্তু ওহমের সূত্র অনুযায়ী ভোল্টেজ বাড়ালে তো কারেন্ট বাড়ার কথা। এই কনফিউশনটা সমাধান করার আমি অনেক চেষ্টা করেছি স্যার, দয়া করে একটু সাহায্য করুন! (ইন্টারনেটে খোঁজাখুঁজি করাও শেষ) – জিসান গাজীপুর
উত্তর: ট্রান্সফরমারের সেকেন্ডারি কয়েলের দুই মাথাকে একটা ব্যাটারির দুই মাথা হিসেবে কল্পনা কর, যেটা সর্বোচ্চ কতোটুকু বিদ্যুৎ প্রবাহ করতে পারবে সেটা ক্ষমতা দিয়ে নির্দিষ্ট করা আছে। (আসল ব্যাটারি সবসময় ডিসি, এটা এসি) এখন চাইলে সেকেন্ডারি কয়েলের দুই মাথায় একটা রেজিস্টর লাগিয়ে ও’মের সূত্র ব্যবহার করতে পার।

প্রশ্ন: স্যার,কেমন আছেন.আরেকটা খুশির খবর আছে৷নেট ঘেটে একটা লাইব্রেরির ওয়েবসাইট পেয়েছি৷যেখানে অনেক লেখকের অনেক বই পড়া যায়৷এখন কেউ আমাকে বই পড়তে বাঁধা দিতে পারবেনা,টাকার অজুহাত দেখিয়ে৷তবে প্র্রযুক্তিতে বই পড়ে কোনো মজা নেই,আসল বই পড়ার স্বাদ পাওয়া যায় না।তবু কি করবো, বাড়ি থেকে বই কিনেও দিবে না,লাইব্রেরিতেও ভর্তি হতে দিচ্ছে না।মহা যন্ত্রণা।তাই এই ব্যবস্থা।যাইহোক আপনাকে জানালাম,নয়তো মনে শান্তি পাচ্ছিলাম না৷কারণ,আপনিইতো আমার অনুপ্রেরণা৷অন্যরা মনে করে বই পড়লে নাকি মাথা নষ্ট হয়ে যায়৷কি আজব৷তবে আমরা ক্ষুদে জাফর ইকবালরা আপনার অনুপ্রেরণাকে সঙ্গী করে সুন্দর আগামীর স্বপ্ন দেখি৷ জিয়াদ,খুলনা
উত্তর: খুবই খুশি হলাম শুনে। সবাইকে এই মূল্যবান তথ্যটা জানিয়ে দাও, অন্যান্য বই পাগলরাও বই পড়ুক।

প্রশ্ন: স্যার, আগামী ১৪ নভেম্বর থেকে আমার এস এস সি পরীক্ষা শুরু হচ্ছে। আমার জন্য দোয়া করবেন,আমি যেন ভালো রেজাল্ট করতে পারি। আমার এবং আমার বাবা মায়ের স্বপ্ন গুলো পুরণ করতে পারি। মিশকাতুল জান্নাত হৃদিতা, লালমনিরহাট।
উত্তর: তোমাকে দোয়া করার আগেই মনে হয় পরীক্ষা শেষ হয়ে গেছে! কিন্তু আমার দোয়া ছাড়াই নিশ্চয়ই তোমার খুব ভালো পরীক্ষা হয়েছে! এখন দোয়া করছি তোমার নিজের স্বপ্নগুলো যেন পূরণ হয়।

প্রশ্ন: আসসালামু আলাইকুম স্যার, “টুনটুনি ও ছোটাচ্চু” বইয়ে আপনি লিখেছেন বানরওয়ালা যখন ধরা পড়ে, তখন তার বানরটা মান্দার গাছের মগডালে উঠে যায়।কিন্তু আমার জানামতে মান্দার গাছে কাঁটা থাকে।তাহলে বানরটা গাছে উঠলো কীভাবে?বানরদের কি কাঁটাযুক্ত গাছে উঠার কৌশল জানা থাকে?না কি আপনি এখানে কাঁটা ছাড়া অন্য কোনো গাছের কথা বলেছেন? লতিফা দাপা-ইদ্রাকপুর, ফতুল্লা, নারায়ণগঞ্জ।
উত্তর: হায় হায়! মান্দার গাছে কাটা থাকে নাকী? নামটা ইন্টারেস্টিং ছিল বলে এই নামটা দিয়েছিলাম। যাই হোক যা হবার হয়ে গেছে এখন চুপচাপ থাকো- বেশি মানুষজনকে ব্যাপারটা জানিয়ে লাভ নেই!

প্রশ্ন: পৃথিবীর সবচেয়ে কঠিন কাজ কি? সুনেহরা পারিজাত হাসান লালমাটিয়া, মোহাম্মদপুর ঢাকা।
উত্তর: তোমার এই প্রশ্নের উত্তর দেওয়াটাই মনে হয় পৃথিবীর সবচেয়ে কঠিন কাজ! একজনের কাছে যেটা কঠিন অন্যের কাছে কী সেটা কঠিন না হয়ে সোজা হতে পারে না? তুমি বরং আমাকে জিজ্ঞেস করতে পার, আমার কাছে কোনটা সবচেয়ে কঠিন মনে হয়, সেটা বলতে পারি, “পড়া মুখস্ত করা”।

প্রশ্ন: স্যার, আসসালামু আলাইকুম। কেমন আছেন? স্যার আপনি কি Anne Frank এর মিউজিয়ামে গিয়েছেন? স্যার আপনি কি ওয়েবসাইটটিতে মিউজিয়াম এর কিছু ছবি দিবেন? Please? মুনিরা, ফুলপুর, ময়মনসিংহ
উত্তর: হ্যাঁ গিয়েছিলাম। দেখি ছবিগুলো খুঁজে পাই কী না। এন ফ্রাংক যে কনসেনস্ট্রেসন ক্যাম্পে মারা গিয়েছিল সেখানেও গিয়েছিলাম, তার গণকবরটিও দেখেছি। বাচ্চা এই মেয়েটার জন্য খুব মায়া হয়।

প্রশ্ন: স্যার শুভেচ্ছা নিবেন। আমার একটি প্রশ্ন ছিল। সেটা হচ্ছে- আমরা তো জানি যে ব্লাক হোল থেকে আলো বের হয়ে যেতে পারে না বলে সেটাকে দেখায় কালো( কারণ কোনো কিছুই বের হয় না)। বলের প্রভাবে আরকি। এটা একটা মৃত নক্ষত্র। তাহলে এটি যদি কোনো কিছু তার দিকে টেনে নেয় তাহলে সেটি আরও বড়ো হয়ে যায় কেন?( প্রশ্নটা ঠিক এমন না। আসল প্রশ্ন ব্লাক হোলের পরিধি কীভাবে বাড়ে?) রাইয়ান তাওসীফ।ঢাকা
উত্তর: এই সাইটেই আমার ব্ল্যাকহোলের উপর একটা বই আছে সেটা পড়ে দেখো- এই ধরনের প্রশ্নের উত্তর গুলো সেখানে দেওয়া আছে। বইটা পড়ার পরও যদি প্রশ্ন থাকে তখন জিজ্ঞেস কর।

প্রশ্ন: স্যার এ বছর কি ক্লাস ৫ এর জন্য ভর্তি পরীক্ষা হবে?আপনার কী মনেহয়?আমার প্রশ্নের উত্তর দিবেন প্লিজ প্লিজ প্লিইইইজ!ইপসিতা নাথ, চট্টগ্রাম।
উত্তর: আমি তো জানি না! ছোট বাচ্চাদের পরীক্ষার আমি ঘোর বিরোধী।

প্রশ্ন: আসসালামু আলাইকুম স্যার। আশা করি ভালো আছেন। আজকে কিছু সিরিয়াস কথা বলব। এমন পাবলিক প্লেসে বলতে খানিকটা সংকোচবোধ করছি। আবার ই-মেইল করার সাহসও হচ্ছে না। যদি অন্য কারও কাছে চলে যায় তাহলে কী করব! (আমি একটু বোকা টাইপের। এর আগে কোনোদিন কাউকে মেইল করেনি তাই আর কি…) এজন্য ঠিক করেছি এখানেই আপনাকে লিখি। যদি আমার কথাবার্তা ঠিক না থাকে তাহলে উত্তর না দিলেও চলবে। আমি কোনো একদিন সাহস করে আপনাকে ই-মেইল করে দিব! আজ পর্যন্ত ছোট-বড় অনেকজনকে কথাগুলো বলেছি। কিন্তু কেউই আমার পক্ষে না, সবাই একই ধরনের যুক্তি দিয়ে আমাকে ভুল প্রমাণিত করে। তবু আমি মানতে পারি না। আমার প্রশ্ন হলো, যখন ক্রিকেট খেলা হয় তখন পাকিস্তানকে বাঙালিরা কেন সাপোর্ট করে?! আমার কথা হাস্যকর শোনাচ্ছে কিনা জানি না। তবে ব্যাপারটা আমার কাছে অনেক বেশি অদ্ভুত লাগে। হয়তো সবাই সাপোর্ট করেনা, কিন্তু আমি যতজনকে দেখেছি- আম্মু, আব্বু, খালামণি, মামা, মামি, আমার কাজিন, ফ্রেন্ড সবাই পাকিস্তান খেলায় জেতার পর বেশ হইচই করে খুশিতে! এমনটা কেন হবে স্যার? আমি ওদের সবাইকে বলেছি যে একাত্তর সালে পাকিস্তানিরা যে এদেশের লক্ষ লক্ষ মানুষ মেরে ফেলেছিল তা কি ভুলে গেছ? সবার একটাই উত্তর,” এখনকার পাকিস্তানিরা তো আর তখন আমাদের দেশে নির্যাতন চালায়নি। তাহলে অতীতের জন্য এই প্রজন্মের মানুষকে কেন ঘৃণা করব?” আমি কোনো উত্তর দিতে পারিনি! যদি কিছু মানুষ আমার বাবাকে গুলি করে তাঁর লাশ নদীতে ফেলে দেয়, কিংবা আমার ভাইকে ধরে নিয়ে গিয়ে অমানুষিক নির্যাতন করে, কিংবা আমার মার সম্মান নষ্ট করে তাহলে পঞ্চাশ বছর পর সেই মানুষদের পরবর্তী প্রজন্মের কাউকে দেখলে অন্যদের কথা জানি না, তবে আমি ভয়ানক কিছু একটা করে ফেলতাম। আমি জানি, পাকিস্তানিরা এখন আমাদের সাথে বন্ধুত্বপূর্ণ আচরণ করে, আমাদের স্বাধীনতার সুবর্ণজয়ন্তীতে শুভেচ্ছা জানায়। আমি তাদের সারা জীবন ঘৃণা করতে বলছি না, কিন্তু তাই বলে ত্রিশ লক্ষ মানুষকে যেই দেশের মানুষ মেরেছিল সেই দেশের খেলা দেখে হাততালি দেওয়া বা উল্লাস করাটা কি খুব জরুরি? আমার এক ফ্রেন্ডের সাথে সেদিন সিরিয়াস টাইপের ঝগড়া করেছি! সে বলল পাকিস্তানের জাতীয় সঙ্গীতটা খুব সুন্দর। আমার তখন ভয়ানক রাগ উঠে গেছে। সেখান থেকেই ঝগড়ার সূত্রপাত! আমার নাকি সবাইকে শ্রদ্ধা করা উচিত। আমি মানুষকে কখনো অপমান করতে চাই না, যদি কখনো করে থাকি তাহলে সেটা ভুলবশত। আর পাকিস্তানিদের যেহেতু দেখিনি তাই তাদের প্রতি মনে হয় ঘৃণাবোধ জন্মায়নি। তবে একেবারে শ্রদ্ধার বিষয়টাও আমার মধ্যে আসে না। আমি কী করব এখন??? আচ্ছা স্যার, আরেকটা প্রশ্ন করি। আমি অনেক ভীতু, যখন কোনো কারণে ব্লাড টেস্ট করতে যাই- এক/দুই ফোঁটা রক্ত দেখেই আমার শরীর কেমন জানি কাঁপে! আর যখন সূচ ঢুকানো হয় তখন তো… থাক না বলি! (হাস্যকর আর সম্পূর্ণ অপ্রাসঙ্গিক উদাহরণের জন্য সরি!!) একাত্তর সালে পাকিস্তানিরা কত্ত বেশি সাহসী (এটাকে সাহস বলা ঠিক না, কিন্তু উপযুক্ত কোনো শব্দ আমার জানা নেই) ছিল যে তারা মানুষকে লাইন করে দাঁড় করিয়ে একসাথে মেরে ফেলত! তাদের কি হাত কাঁপত না? কখনো কি মনে হতো না যে বৃদ্ধ লোককে সে মারছে ঠিক তার মতো একজন বৃদ্ধ বাবা তার ঘরেও আছে? কেউ যদি তার বাবাকে মেরে ফেলে তখন সে নিজে কী করত?? আমি কাউকে অসম্মান করার উদ্দেশে লিখিনি। শুধু জানতে চাই যে পাকিস্তান ক্রিকেট খেলায় ছয় বা চার মারলে আমি আমার পরিবারের সাথে হাসিমুখে হাততালি দিতে পারি না এতে কি আমার কোনো দোষ আছে? প্রশ্নটা মাথায় এসেছে কারণ সবাই বলে আমার চিন্তাভাবনা ভুল। কেন ভুল তা বুঝতে পারিনা। সবাই এক না আমি জানি। কিন্তু আমি খেয়াল করেছি আমার ফ্রেন্ডরা বা আমার ফ্যামিলি এই ব্যাপারটায় সবাই একমত। আমি নিজেই শুধু ভিন্ন রকম ভাবছি। কেমন জানি অস্বস্তি লাগছে আমার!! ~ ***, ঢাকা ( আমার নামটা প্রকাশ করবেন না প্লিজ। পূর্ব অভিজ্ঞতা থেকে বলছি আমার উপর অনেকে ক্ষেপে যেতে পারে।)
উত্তর: তোমার বক্তব্যটা আমি পুরোটুকু দিয়ে দিচ্ছি, অন্যরা যদি চায় দেখুক। এই দেশের নূতন রাজাকাররাও পড়ে দেখুক! শোন, আমরা বলি দেশ হচ্ছে মায়ের মতন। আসলে কথাটা আরো সিরিয়াস। দেশ শুধু মায়ের মত না, দেশটাই আসলে মা। ডক্টর শহীদুল্লা খুব সুন্দর করে বলেছেন, প্রত্যেক মানুষের মা তিনটি, যে মা তোমাকে জন্ম দিয়েহে, তোমার মাতৃভাষা এবং তোমার মাতৃভুমি। দেশ বা মাতৃভুমি যেহেতু মা কাজেই সেটার অবমাননা করা হচ্ছে নিজের মা’কে অবমাননা করা। যে তার নিজের মা’কে অবমাননা করে তার জন্য আমার ভেতর বিন্দুমাত্র সম্মান বোধ নেই। শুধু তাই না তার জন্য আমার ভেতর গা ঘুলিয়ে ওঠা বিতৃষ্ণা ছাড়া আর কিছু নেই। এখন যার যা খুশি, যত রকম ব্যাখ্যা আছে দিতে পারে আমার বিতৃষ্ণা তাতে এতোটুকু কমবে না। কাজেই বাংলাদেশের সাথে পাকিস্তানের খেলার সময় যদি কেউ পাকিস্তানকে সাপোর্ট করে তাহলে বুঝতে হবে সে তার মা’কে অবমাননা করছে। আমি না চাইলেও তার জন্য আমারভেতরে শুধু বিতৃষ্ণা এবং বিতৃষ্ণা  পাক খেতে থাকে। আমার কিছু করার নেই।
ডিসেম্বের মাসে মুক্তিযুদ্ধ যাদুঘর বাচ্চাদের জন্য ঢাকা বিশ্ববিদ্যালয় মাঠে একটা বড় অনুষ্ঠান করে- প্রতি বছর অন্যদের সাথে আমাকেও সেখানে ডাকে। আমি সেখানে বাচ্চাদের সাথে কথা বলি। গতবার কথাবার্তা বলে এক জায়গায় পাকিস্তান সম্পর্কে এক জায়গায় বলেছি, “আমি এই দেশটার নাম মুখে উচ্চারণ পর্যন্ত করি না- যদি কখনো করতে হয় বাসায় গিয়ে সাবান দিয়ে মুখ ধুয়ে ফেলি-’’
পাকিস্তান নামক এই দেশটি বাংলাদেশে যে হত্যাকাণ্ড চালিয়েছিল, আমি সেটা নিজের চোখে দেখেছি। অর্ধ শতাব্দী পরেও সেই হত্যাকান্ড নিয়ে পাকিস্তানের কোন অপরাধবোধ নেই, কোনো দুঃখবোধ নেই তাই সেই দেশ এবং সেই দেশের সাপোর্টারদের জন্য আমার এবং আমার প্রজন্মের ভেতরে প্রবল বিতৃষ্ণা ছাড়া আর কিছু নেই।

প্রশ্ন: একটা প্রশ্ন স্যারের কাছে-জাহানারা ইমামের লেখা ʼʼএকাত্তরের দিনগুলিʼʼ-এই বইটির কোন ইংরেজি অনুবাদ আছে কি?এবং থাকলে কোথায় পাবো।আমার ১০ বছর বয়সী ছেলের জন্য খুঁজছি। ২ বছর বয়সে ও দেশের বাইরে চলে আসে। তখন ইংরেজীতে কথা বলতে শেখার সংগ্রাম, আর গত বছর থেকে জার্মান ভাষা শেখার সংগ্রাম-তাই বাংলা পড়া শেখানোতে মনোেযাগ দিতে পারিনি-তবে ও বাংলা ভালোভাবে বলতে পারে।বই পড়তে খুব ভালোবাসে। এই বইটা ইংরেজিতে পড়তে পারলে,ওর জন্য বুঝতে সুবিধা হতো। বা অন্য কোন ইংরেজি বইয়ের নাম পেলেও হবে-যা ওর জন্য বোঝা সহজ। ভলো থাকবেন,সুস্হ থাকবেন।ধন্যবাদ স্যার। মাকসুদা,জার্মানী থেকে।
উত্তর: হ্যাঁ আছে। বইটির নাম ‘Of blood and fire: The untold story of Bangladesh war of independence’

প্রশ্ন: আপনি একজনের প্রশ্নের উত্তরে বলেছেন “পর্যায় সারণি তো বইয়েই আছে। মুখস্থ কেন করতে হবে?” সেটাই তো সমস্যা!! রসায়নের Mcq solve করতে হলে তো এগুলো লাগবেই! তাইনা? আর সৃজনশীল প্রশ্ন গুলো তো নামেই ‘সৃজনশীল’!!!! কারণ এসব সৃজনশীল প্রশ্নের উত্তর করতে হলেও সম্পূর্ণ পর্যায় সারণি মুখস্থ রাখতে হয়!! ব্রেকেটে তো আর দেয়া থাকে না olympiad এর মতন!!! এই জন‍্যে পুরো পর্যায় সারণিটা বইএ থাকা সত্বেও মাথায় গুজে রাখা লাগে!!! আচ্ছা আপনার SSC বা HSC পরীক্ষার সময় আপনি এটা মুখস্থ করেননি?!!! বাঁধন ( Class 8, টাঙ্গাইল )
উত্তর: পড়তে পড়তে অনেক কিছু মনে থাকে। কিন্তু মুখস্ত করার জন্য কখনো কিছু মুখস্ত করিনি। মাথা খারাপ নাকি?

প্রশ্ন: স্যার,আমি মাহদি সিলেট থেকে। দশম শ্রেনীতে পড়ি৷ স্যার বিজ্ঞানি আলবার্ট আইনস্টাইন এর একটা ভালো জীবনীগ্রন্থ এর নাম বলতে পারবেন? ইন্টারনেটে অনেক বইয়ের নাম দেখি, কিন্তু কোনটা ভালো জানি না ৷ আর আমার ওত টাকা নেই যে একটা একটা করে কিনে দেখব। বাংলা বই হলে ভালো। অনুবাদ হলেও চলবে। ইংরেজিতে হলেও চলবে৷ আসসালামু আলাইকুম
উত্তর: একটা ভালো বাংলা বই ছাপা হচ্ছে। তুমি কয়দিন পর আবার জিজ্ঞেস করো- তখন বলব।

প্রশ্ন: স্যার ভালো আছেন? আপনাকে আমার একটা প্রশ্ন করার ছিল এখন টি টোয়েন্টি বিশ্বকাপ চলছে অনান্য দেশের মতো বাংলাদেশও এটাতে অংশগ্রহণ করেছে আর সাথে পাকিস্তানও অংশ নিয়েছে। এখন আমি ও আমার একজন বান্ধবীর মধে্য কথা চলছিল এই বিষয়ে সেখানে আমার বান্ধবী মন্তব্য করে যে পাকিস্তানিরা অনেক ভালো খেলে। ওর এই কথায় আমার প্রচন্ড রাগ হয় তখন আমি তাকে বলি ওরা আমাদের সাথে যা করেছে তারপরও তুই এটা কীভাবে বলতে পারলি। প্রতুত্তরে ও বলে পাকিস্তানিরা নাকি যা করেছে তা অতীতে করেছে এখন ওরা বাংলাদেশের ভালো চায়। ওর এই কথায় আমি বলি যে ওরা যা করেছে আমাদের অতীতের সাথে করলেও ওরা অন্যায় করেছে এবং প্রজন্মের পর প্রজন্ম ওরা বাংলাদেশিদের ঘৃণা পাবে এবং ওরা কখনও বাংলাদেশের ভালো চাইবে না। আচ্ছা স্যার সবাই কেনো পাকিস্তানের সাপোর্ট করে। (অধিকাংশ) কিন্তু আমি ওদের ঘৃণা করি আমি জানি আপনি করেন কিন্তু যখন এইভাবে পাকিস্তানের সাপোর্ট ওরা কেনো করে। পাকিস্তান ভালো খেলুক খারাপ খেলুক তাতে আমাদের কী এভাবে কেনো এখন ওদের সাপোর্ট কারে আমি অনেক কিছু বলে ফেললাম আমি জানিনা আপনি কী করবেন তবুও আমার এই প্রশ্নটার উত্তরটা প্লিজ দেবেন। সুস্মিতা, নাটোর
উত্তর: তোমার অনেক রাগ হয় জেনে আমি অবাক হচ্ছি না! যারা এই দেশের ইতিহাসটি শুধু জানে না অনুভবও করে তাদের রাগ হতেই পারে। আমি অন্য একজনের প্রশ্নের উত্তরে বিষয়টা নিয়ে আরো বিস্তারিত লিখেছি- তাই তোমাকে সেগুলো আবার লিখছি না।
আমি মুক্তিযুদ্ধের ইতিহাস নামে ছোট একটা পুস্তিকা লিখেছিলাম যেটা এই ওয়েব সাইটে আছে, সেই পুস্তিকার শুরুতে বলেছিলাম পৃথিবীতে যত অনুভুতি আছে তার মাঝে সবচেয়ে মধুর অনুভুতি হচ্ছে দেশের জন্য ভালোবাসা। যাদের নিজের দেশের জন্য ভালোবাসা নেই, ভালোবাসাটুকু আছে পাকিস্তানের মত একটা হতভাগা দেশের জন্য (এখন যেরকম অবস্থা যদি সেই অবস্থাই থাকে তাহলে যে দেশের কোনো ভবিষ্যত নেই) তারা হচ্ছে পৃথিবীর মাঝে সবচেয়ে বড় হতভাগা। পৃথিবীর সবচেয়ে মধুর অনুভুতি পাওয়ার ক্ষমতা পর্যন্ত তাদের নেই। তাহলে তুমি তাদের উপর রাগ কেন হবে? এই নির্বোধ প্রাণীদের জন্য বড়জোর তোমার করুণা হতে পারে।

প্রশ্ন: আসসালামু আলাইকুম স্যার। প্রথমেই বলে নেই আমার মনমেজাজ ভালো নেই। আমি আজকে অনেক ভুলভাল, উল্টাপাল্টা কথা বলব। জেনেশুনে অনেক বেয়াদবিও করব। এর জন্য আগেই আপনার কাছে ক্ষমা চেয়ে নিচ্ছি, সরি সরি সরি। আমার পনেরো বছর জীবনের এই কথাগুলো মা বাবাকে বলার খুব ইচ্ছা। কিন্তু সাহস নেই। ইদানীং দম বন্ধ হয়ে আসে। আপনি একবার বলেছিলেন দুঃখকষ্ট আপনজনদের সাথে ভাগ করে নিতে। আমি আপনাকে আপন ভেবে কথাগুলো বললে কি আপনি রাগ করবেন??? পড়ালেখা, হোমওয়ার্ক, এসাইনমেন্ট… (আরও যা যা আছে) সবকিছু নিয়ে আমি মোটামুটি ব্যস্ত জীবন কাটাই। মাঝে মাঝে যখন একটু রেস্ট নেওয়ার জন্য চোখ বন্ধ করি, তখন প্রায়ই যে দৃশ্যটা চোখের সামনে ভেসে উঠে সেটা হলো- গ্রামের বাড়িতে অনেক মেহমান এসেছে। আমি(তখন বয়স সাত) আর আমার চাচাতো বোন (বয়স দশ) বাড়ির সামান্য দূরের একটা দোকানে গিয়েছি কিছু জিনিস কিনতে। কেনা শেষে দুজনে গল্প করতে করতে ফিরছি, রাস্তা মোটামুটি ফাঁকা। এমন সময় চার/পাঁচজন বড় বড় ছেলে সাইকেলে করে এসে আমাদের চারদিকে দাঁড়িয়ে অযথা অযথা হাসছে। একজন আবার তার সাইকেলের চাকা এনে আমার স্যান্ডেলের উপর তুলে দিয়েছে। আমি সেটা কোনোপ্রকারে ছুটিয়ে নিচ্ছি। তা দেখে তারা আবার অট্টহাসিতে ফেটে পড়ছে! একসময় আমার বোন আমার হাত ধরে আচমকা দৌড় দিল। আমি ছোট তখন তেমন কিছুই বুঝলাম না। ছেলেগুলো হাসছিল আমরাও হাসি সমস্যা কি! পাশের গলি অবধি আসার পর শুনি ওরা জোরে জোরে শিস বাজাচ্ছে। তখন কিছুটা ভয় অবশ্য লেগেছে। আমি মনেপ্রাণে বিশ্বাস করতে চাই যে এমন অদ্ভুত আর অবাস্তব(?) ঘটনা কখনোই ঘটেনি। কিন্তু সাথে সাথেই ঐদিনের পরের ঘটনাগুলো মনে পড়ে। আমার বোন নিজের ভাগের চকলেটটা আমাকে ঘুষ হিসেবে দিয়ে হাতজোড় করে বলেছিল “প্লিজ কাউকে বলিস না।” আমি একটা চকলেট বেশি পেয়েছি তাতেই খুশি! কিছু বলিনি কাউকে। তারপর আটবছর কেটে গেছে। আমার স্মৃতিশক্তি ভয়ানক খারাপ, কোনোকিছুই মনে থাকে না। তবে ছোটবেলার ঐদিনটার কথা পুঙ্খানুপুঙ্খভাবে মনে অছে, কিচ্ছু ভুলিনি। গতকাল মামার বাসায় গিয়ে … … … …
উত্তর: মেয়েদের দুঃসহ জীবন নিয়ে যে মেয়েটি দুই হাজার থেকে বেশি শব্দের এই বিশাল লম্বা লেখাটি লিখেছে (আমি ছোট একটা অংশ দেখিয়েছি) তাকে বলছি: তোমার লেখাটি অত্যন্ত মর্মস্পর্শী। তুমি কী কোন একটা পত্র পত্রিকায় প্রকাশ করার জন্য এটা পাঠিয়ে দেবে- সবারই এগুলো জানা উচিৎ। তুমি শুধু যে খুব সুন্দর করে লিখতে পার তা নয়, খুব চমৎকার ভাবে মনের কথাটা প্রকাশও করতে পার। তোমার লেখালেখি করা উচিত।
তোমাকে বলছি তুমি একা নও। টিকে থাকো- একদিন তোমরা সবকিছু পরিবর্তন করবে আমি জানি।

নভেম্বর ৩,  ২০২১

যারা অটোগ্রাফ চেয়েছ, প্রশ্ন-উত্তর শেষে সেগুলো দিয়ে রেখেছি, খুঁজে নাও প্লিজ।

প্রশ্ন: Md zafar ikbal sir i am your very big fan please give me a Autograph. Name ইমতিহান ইসলাম Class 2
উত্তর: এই যে অটোগ্রাফ। আমার ‘’বিগ ফ্যান’’ না হয়েও অটোগ্রাফ নিতে পারো। কোন সমস্যা নেই। (তুমি মাত্র ক্লাশ টু’তে পড়, তুমি আর কত big হবে?)

প্রশ্ন: আসসালামুআলাইকুম, দাদাভাই , আমি সেই কাশফি সাভার থেকে, মনে আছে? আচ্ছা, দাদাভাই আমি মাসখানেক ( সম্বভত ১/২ মাস) আগে আপনাকে একটা ই-মেইল দিয়েছিলাম। এখন ওবদি জবাব এলো না! আমি আপনার ই-মেইল অ্যাড্রেস টাও খুব সাবধানে দেখে, লিখেছি। সেখানে অনেক কিছু বলেছি আপনাকে। একটু ভালো করে দেখো না গো, দাদাভাই ! প্লিজজ ! আমার নামের ইংরেজি বানান —- ( Ashia Kashfi )
উত্তর: তোমরা যদি ভাবো আমি খুব দায়িত্বশীল মানুষ, কোন ই-মেইল পেলেই সাথে সাথে উত্তর দিয়ে দেই তাহলে খুব ভুল করবে। যদি কোন উত্তর না পাও তাহলে আবার পাঠাবে— কয়দিন পর আবার—তারপর আবার— যতক্ষণ পর্যন্ত উত্তর না পাও পাঠাতেই থাকবে।

প্রশ্ন: আমার সবচেয়ে বড় দুঃখের সংখ্যা প্রায় শখানেক হবে।তার মধ্যে অন্যতম দুঃখটা হলো আমার কোনো গল্পের বইপড়ুয়া বন্ধু নেই। তারা সবাই শুধু ‘পাঠ্যপুস্তক’ পড়ে। স্কুলে পুরস্কার পাওয়া বই আর জন্মদিনে পাওয়া বই ছাড়া আর কোনো গল্পের বই পড়েনি। আমি যখন ওদেরকে কোনো বইয়ের কথা বলতে যাই তখন ওদের মুখগুলো দেখে মনে হয় বিজ্ঞান স্যারের লেকচার শুনছে! গল্পের বইয়ের প্রতি আগ্রহী করার জন্য আমি তাদেরকে আমার হীরার চেয়েও মূল্যবান বইগুলো ধার পর্যন্ত দিতে চেয়েছিলাম কিন্তু তারা নেয়নি! (আমার হাড়কিপটে বান্ধবীগুলো ফ্রীতে পাওয়া জিনিস এভাবে রিফিউজ করতে পারে আমার এখনও বিশ্বাস হচ্ছে না) এদের গল্পের বইয়ের প্রতি কীভাবে আগ্রহী করা যায়?(আপনি হয়তো বলবেন দরকার কী আগ্রহী করার, কিন্তু দরকার আছে! আমি গল্পের বইয়ের গল্প করবো কার সঙ্গে?) আফরিদাহ আহসান, ঠাকুরগাঁও
উত্তর: শুনে খুবই দুঃখ পেলাম। যারা এই বয়সেও বই পড়া শিখেনি তাদের বইয়ে আগ্রহী করার কাজটা মোটেও সহজ না! তোমার কাছে যদি এমন কোন মজার বই থাকে যেটা তুমি মনে করো একজনকে জোর করে পড়ালে বইয়ে আগ্রহী হবে তাহলে সেটা পড়িয়ে দেখতে পার!

প্রশ্ন: আচ্ছা আপনার কাছে যদি তিনটা অপশন থাকে যে : ১) Class 7 ২) class 10 ৩) college কোনটা নেবেন? কেন? রিমি (ঢাকা)
উত্তর: হুমম, ব্যাপারটা এরকম হল যে আমার ফেবারিট হচ্ছে আলু আর তুমি জিজ্ঞেস করেছ আপনাকে করলা, ঢেঁড়স কিংবা চিচিংগা দেয়া হল, আপনি কোনটা খাবেন?

প্রশ্ন: আচ্ছা যেগুলা আম্মুর সাথে শেয়ার করা যায় না সেই সবই কি খারাপ??? আমি জানি বেশিরভাগই খারাপ। কিন্তু সব!!!?? এই যেমন আম্মু সব সময় বলে ভালো স্টুডেন্টরা কখনো ক্লাসে চিল্লাপাল্লা করে না, কেউ কখনো ফ্রেন্ডদের “পাগল”” শয়তান ” বলেনা। সব সময় ক্লাসে চুপচাপ বসে খালি পড়ে। কিন্তু আমি তো সুযোগ পেলেই ফ্রেন্ডসদের ব্যাগের চেন খুলে দেই, ওদের যত ইচ্ছা মার দেই আবার মার খাইও। এগুলা তো সব মজা। ওরা কিছ্ছু মনে করে না,আমিও না। আবার বলে যে ভালো স্টুডেন্টরা সবার সাথে বসে না। ঠিক আছে খারাপদের সাথে আমি বসিনা। কিন্তু যারা পড়ায় ভালো না তাদের সাথে একটু কথা বললে কি এমন হয়? আবার ধরেন হঠাৎ মনে হলো আজকে এই রাস্তা দিয়ে না গিয়ে ঐ রাস্তা দিয়ে যাই। কোনো কারণ ছাড়াই এটা চরি। বিশ্বাস করেন এমনিই। ব্যস মনে হলো তাই। এটা আম্মু জানলে একশ বার জেরা করবে। তাই বলে কি লাভ? আমি নিজে তো জানি তাইনা? তাহলে? এসব আম্মুকে বললেই পরেরদিন দেখতে যাবে আমি কি করি না করি। আচ্ছা আমি নিজে তো জানি আমি এসব শুধু মজা করেই করি। এগুলা শেয়ার করলেই বাসায় ঝামেলা বাজবে। তাহলে এগুলা কি খারাপ??আমি চিন্তায় আছি। ( **** নামটা আবার লেইখেন না কিন্তু। তখন আবার বাসায় সত্যিই ঝড় শুরু হবে। প্লিজ…… seriously বললাম )
উত্তর: তুমি যে কাজগুলার কথা বলেছ–তোমার আম্মু সেগুলো জানলে বড়জোর তোমার কান ধরে একটা টান দিতে পারেন। কিন্তু অনেক কাজ আছে সেগুলো করলে তোমার আম্মুর মনটা খারাপ হয়ে যাবে, রীতিমত হতাশ হয়ে যাবেন, আমি সেগুলো বুঝিয়েছিলাম। কমন সেন্স দিয়ে বুঝতে হয় কোনটা দুষ্টুমি আর কোনটা অন্যায়। বাচ্চারা তো দুষ্টুমি করবেই, তা না হলে বাচ্চা হবে কেন?

প্রশ্ন: স্যার,আমাদের স্কুলে একজন স্যার আছেন। স্কুলের সবাই তার যন্ত্রণায় অতিষ্ঠ। তিনি অষ্টম শ্রেণির বিজ্ঞান ক্লাসে এসএস সির পড়া পড়ান, দশম শ্রেণির ক্লাসে এইচ এস সির পড়া পড়ান।ক্লাস নেওয়ার সময় পাঠ্যবই পড়ান না, এমনকি পরিক্ষার আগে বলে দেন, পাঠ্যবই পড়ে লাভ নেই,ওখান থেকে কিচ্ছু আসবে না,উপরের ক্লাসের বই পড়। পরিক্ষার সময়ও অধিকাংশ প্রশ্ন বই এর বাহির থেকেই করেন। এখন আমরা কয়েকজন চাচ্ছি এমন কিছু করতে যাতে তিনি উপরের ক্লাসের পড়া না পড়িয়ে পাঠ্য বইয়ের পড়া পড়ান।কিন্তু কি করবো বুঝতে পারছি না। আপনার বই পড়ে যা যা উপায় জেনেছি তার কোনোটাই করতে সাহস পাচ্ছি না আমরা।সহজ কোনো উপায় বলে দিন না,প্লিজ। স্যারের বিরুদ্ধে বিদ্রহ ঘোষনাকারী দলের একজন সদস্যা,———(নাম বললাম না, যদি স্যার কোনোভাবে বুঝতে পারেন তাহলে বিদ্রোহ শুরুর আগেই শহিদ হতে হবে,হাহাহা)
উত্তর: প্রকাশ্যে তোমাদের এই ধরনের প্রজেক্টের বুদ্ধি দেওয়াটা বুদ্ধিমানের কাজ হবে বলে মনে হয় না। তাছাড়া এডভান্স পড়ালে ক্ষতি কী? ভালোই তো! আমি তো সারা জীবন বেশি পড়িয়ে এসেছি, যেটা পড়াইনি সেটা পরীক্ষায় দিয়ে এসেছি। তাইতো করার কথা।

প্রশ্ন: স্যার এটা কী সত্যি যে ডিপ্রেশনের মত মানসিক রোগ একবার হয়্র গেলে সেরে উঠতে অনেক বেশি সময় লাগে? কেন এত সময় লাগে? আমি নিজে ডিপ্রেশনে ভুগি, ডক্টরদের কাছে গিয়েও ডিপ্রেশন কমে না, ৮ বছর ধরে একি অবস্থা। আমার জন্যে আপনার উপদেশ কি, স্যার? সুমাইয়া, বরিশাল
উত্তর: আমি কমন সেন্স দিয়ে যেটুকু বুঝি সেটার উপর নির্ভর করে কিছু বলা কী ঠিক হবে? আমি তো বিশেষজ্ঞ নই তাই আসলে আমার মতামতের কোন গুরুত্ব নেই। সবচেয়ে বড় কথা সত্যিকারের ডিপ্রেশনে ব্যাপারটা কি রকম আমি সেটা বুঝতেও পারি না। আমাদের সবার জীবনেই তো আনন্দ থাকে, দুঃখ থাকে— সবকিছু নিয়েই তো আমরা বেঁচে থাকি! এটাই তো জীবন। আমার জীবনেও তো অনেক বড় বড় ধাক্কা এসেছে, মন খারাপ হয়েছে— তখন কি ভবিষ্যতের কথা ভেবে গা ঝাড়া দিয়ে উঠিনি? এটা কি খুব কঠিন?

প্রশ্ন: স্যার আপনি তো এখন অনেক ব্যস্ত মানুষ। ছোটবেলার মত বই নিশ্চয়ই পড়তে পারেন না। এখন ব্যস্ততার মাঝে কীভাবে বই পড়েন? ছোটবেলার সময়টা কি মিস করেন? রাবেয়া, মিরপুর
উত্তর: এখন খুব কষ্ট করে সময় বের করে বই পড়তে হয়। এক সাথে তিন চারটা বই পড়ি। মজার মজার বইগুলোর দিকে লোভ নিয়ে তাকিয়ে থাকি—কখন পড়ব—কখন পড়ব! হ্যাঁ, ছোটবেলা খুব মিস করি। কে না করে?

প্রশ্ন: আপনি বলেছিলেন আমরা বাঘ ভালুক দেখে যে জন্য ভয় পাই আপনি একই কারণে…..। আচ্ছা আমরা ভয় পাই কারণ বাঘ ভালুক ধরলে জীবন নিয়ে বাচার উপায় নাই অথবা খুব কম। কিন্তু নিরিহ মাকড়শা আপনার কী করবে? কিছু মাকড়শা হয়তো ভয়ংকর… কিন্তু তাই বলে ওদের সবাইকে ভয় পান কেন? দেখবেন যেন আপনার বার্থডেটে সব পরিবার নিয়ে হাজির না হয় আপনার বাসায়… হা হা হা ( বাঁধন, class 8 টাঙ্গাইল )
উত্তর: ঠিক আছে, ঠিক আছে মাকড়শাকে ভয় পাওয়া খুবই গাধামোর মত একটা কাজ! কিন্তু কে বলেছে আমি বুদ্ধিমান? সারাজীবনই আমি এরকম নির্বুদ্ধিতা করেই এসেছি, এখনও না হয় করলাম! তোমাদের ঘুম কেন নষ্ট হচ্ছে? আমাকে শান্তিমত একটু ভয় পেতে দাও!

প্রশ্ন: স্যার,সবাইক অটোগ্রাফ দিবার লাগছেন, মোক ও একনা দেন ক্যা।মুই ম্যালা বছর থাকি তোমার একনা অটোগ্রাফের আশাত আছং।না হইলেও ৫বছর থাকি অপেক্ষা করবার লাগছং। দেও ক্যা মোক একনা অটোগ্রাফ ? (স্যার … … … … … দিন না।) লালমনিরহাটের ভাষায় লিখলাম,বোঝার সুবিধার জন্য পরেরটা লিখেছি।।মিশকাতুল জান্নাত হৃদিতা, লালমনিরহাট। পুনশ্চঃঅটোগ্রাফে হৃদিতা নামটা দিবেন প্লিজ।
উত্তর: দিলাম অটোগ্রাফ! লালমনিরহাটের ভাষাটি কি সুইট! তুমি প্রমিত বাংলায় অনুবাদ না করলেও আমি বুঝতে পারতাম। আমরা বেশ কয়েক বছর বগুড়া ছিলাম। এত সুন্দর ভাষাটা ভুলে যেও না যেন। আমাদের আঞ্চলিক ভাষাগুলো যে আমাদের সম্পদ সেটা কি তোমরা জান?

প্রশ্ন: স্যার আপনি ২০২২ সালের বইমেলার জন্য কি কি বই লিখেছেন জানতে পারি? অনন্ত আহমেদ, ষষ্ঠ শ্রেণি,ঢাকা ।
উত্তর: আমি এখন আর বই মেলার জন্য লিখি না— আগেই লেখি। আমার “ডেঞ্জেরাস মামী’’ বইটা বের হয়ে গেছে। ‘‘আমি পরমানব’’ নামে সায়েন্স ফিকশন ছাপার কাজ হচ্ছে। “সহজ ইলেকট্রনিক্স’’ লেখা শেষ এখন ফাইন টিউনিং হচ্ছে। অন্যগুলো এখনো মগজে কুট কুট করছে।

প্রশ্ন: আচ্ছা আমি কি সত্যিই আপনার সাথে কথা বলছি? যেই মানুষটার সাথে কথা বলা ছিল শুধুই একটা স্বপ্ন, তার সাথে কথা বলছি? ব্যাপারটা যতই ভাবছি ততোই অবিশ্বাস্য মনে হচ্ছে। আসলে আমি অনেক বেশি এক্সাইটেড হয়ে পড়েছিলাম।(দেখছেন না দেড়-দুমাসে কতো প্রশ্ন করে ফেলেছি) আর আপনি যে আমার এক কথায় অটোগ্রাফ দিয়েছেন(দুবার!), সেটা তো আমার কল্পনারও বাইরে ছিল। আমি কখনো এতোটা আশা করিনি, সত্যি। আসলে আমি আপনাকে যে কতোখানি শ্রদ্ধা করি, বিশ্বাস করি, মানি, ভালোবাসি, সেটা কখনোই বলে বোঝাতে পারবোনা। আমি গুছিয়ে কিছু বলতে পারিনা, কী উল্টাপাল্টা লিখছি নিজেই জানিনা। তবে একটা কথা বলতে চাই, থ্যাংক ইউ সো মাচ আমি যতটা আশা করিনি তার চেয়েও বেশি দেওয়ার জন্য। আপনি হয়তো ভাবছেন কীইবা দিয়েছেন? কিন্তু আপনি জানেননা এই বারো বছরের মেয়েটার কাছে এটা কতো বড়। আমি হয়তো বাড়াবাড়ি করছি, কিন্তু আমি মনের কথাটাই বলছি। আফরিদাহ আহসান, ঠাকুরগাঁও
উত্তর: হা হা হা! তোমাদের খুশি করা কতো সোজা! দেখেছ? সেজন্যই তো আমি বড়দের ধারে কাছে যেতে চাই না— তোমাদের কাছে ঘুরঘুর করি!

প্রশ্ন: আমার নাম বন্যা বড়ুয়া। আমি চট্টগ্রামে থাকি। স্যার,,,বন্ধুদের নিয়ে ঘুরতে যাওয়া কি খারাপ যখন কেউ কলেজে বা ইউনিভার্সিটিতে পড়ে এই সময়?? আর আপনার তিন্নি ও বন্যা নামে যে বইটি আছে সেটির নাম আপনার মাথায় এলো কিভাবে?? আসলে আমার নামও বন্যা তো তাই বললাম।ভালো থাকবেন।
উত্তর: না বন্ধুদের নিয়ে ঘুরতে যাওয়া মোটেই খারাপ না, তবে কোন বন্ধুদের সাথে কোথায় যাচ্ছ, কোথায় থাকবে কী করবে সেটা একটুখানি জানতে চাই। বিশেষ করে তোমার গ্রুপে কিশোর গ্যাংয়ের কোনো মেম্বার আছে কিনা, কিংবা পকেটে গাঁজার পুরিয়া থাকে এরকম কেউ আছ কিনা সেটা বিশেষ করে জানা দরকার! বড়রা ছোটদের সেফটি নিয়ে একটু বেশি দুশ্চিন্তা করতেই পারেন সেজন্য অনেক সময় নিজেরা নিজেরা ঘুরতে যেতে দেন না! (বন্যা নামটা আমার পছন্দের নাম সে জন্য লিখেছিলাম। তুমি কি জান, শুধু আমদের দেশে বন্যা নামটি মানুষ ভালবেসে দেয়— পৃথিবীর অন্য সব দেশে বন্যা মানেই দুর্যোগ— ভয়াবহ ব্যাপার!)

প্রশ্ন: স্যার, আপনি বলেছিলেন penguin এ আপনার একটি বই প্রকাশ হয়েছে। কিন্তু penguin দের ওয়েবসাইটে গিয়ে দেখলাম ওরা সব উলটা পালটা কথা লিখে বসে আছে। বলেছে rasha বইটার লেখক মুহম্মদ জাফর ইকবাল কিন্তু তিনি ভিন্ন মানুষ। স্যার ওরা তো আপনার উলটা পালটা জীবনী লিখেছে! কিছু করুন স্যার! আমি আপনাকে লিংকটা দিলাম। আপনি পড়ে নিশ্চয় ব্যথিত হবেন( কারণটা আপনি বের করে নিন) https://penguin.co.in/book_author/muhammed-zafar-iqbal/ রাইয়ান তাওসীফ,অষ্টম, ঢাকা
উত্তর: হা হা হা! তোমাদের কারণে আমি এতো মজার একটা বিষয়টা দেখতে পারলাম! থ্যাংকু থ্যাংকু থ্যাংকু! আমার বইয়ের অনুবাদককে লিংকটা পাঠিয়েছিলাম, সে দেখে তো রেগেমেগে ফায়ার! Penguin কে বলে তক্ষুনি ঠিক করতে চাইছিল, আমি অনেক কষ্টে তাকে থামিয়েছি। আমি এখন সবাইকে এটা দেখাচ্ছি, সবাই খুবই মজা পাচ্ছে! কতো বড় বড় প্রতিষ্ঠান আহাম্মকের মত কতো মারাত্মক ভুল করে বসে থাকে দেখেছ?

প্রশ্ন: স্যার আমার ভাগ্য এত খারাপ কেন? উঁহু, ভাগ্যের দোষ দিয়ে লাভ নেই। দোষ হলো আমার সাহসের। তাহলে প্রশ্নটা সঠিকভাবে করলে দাঁড়ায় এমন: আমি এত ভীতু কেন? একসাথে পুরো শিশিটা ফাঁকা করলেই হয় কিন্তু আমি দুইটা পর্যন্ত খেয়ে থেমে যাই।
… … … … …
জানেন স্যার, মৃত্যুকে আমি খুব কাছে থেকে দেখেছি। তখন আমার বয়স মনেহয় বারো, ডেঙ্গু হয়ে হসপিটালে ভর্তি ছিলাম বহুদিন। আমাকে লাইফসাপোর্ট দিতে হয়েছিল! কীভাবে বেঁচে ফিরেছি জানি না, দাদা বলেছে আমি নাকি আমার পাপের ফল ভোগ করছি! অসুস্থ ছিলাম বলে পড়ালেখা ঠিকমতো করতে পারেনি। তবু সেই বছর পিইসিতে বৃত্তি পাই নি দেখে বাবার বকা আমাকে ঠিকই শুনতে হয়েছে। 600 মার্কসের মধ্যে 591 পেয়েছিলাম কিন্তু বাবা তাতে বিন্দুমাত্র খুশি হয়নি! আমি আমার মা, বাবা কিংবা দাদা কাউকেই অপরাধী বানাচ্ছি না। আমি কারোর বিরুদ্ধে নালিশও জানাচ্ছি না। শুধু বলতে চাইছি আমি তাদেরকে অনেক অনেক অনেক অনেক অনেক বেশি ভালোবাসি। কিন্তু তাদের ব্যবহার, আমার বিরুদ্ধে সমালোচনা আমি কিছুতেই সহ্য করতে পারি না। আজ সকালবেলা ঘুম ভেঙেছে বাবার ধমক শুনে। মনেহয় রাতে ঘুমানোর সময়ও ধমক শুনতে হবে!
… … … … … … … …
***, ঢাকা (নামটা ভুলেও প্রকাশ করবেন না!)
উত্তর: শোন— তোমার লেখাটা পড়ে আমার খুব মন খারাপ হয়েছে। দেখতেই পাচ্ছ আমি তোমার লেখার প্রায় পুরোটা কেটে ফেলেছি, অল্প একটু রেখেছি, পুরোটা দেখলে অন্যদের মন খারাপ হবে সেজন্য। তুমি যেহেতু আমাকে এটা লিখেছ আমি ধরে নিচ্ছি আমাকে তুমি একটুখানি হলেও বিশ্বাস করো। কাজেই আমি তোমাকে বলছি, সবকিছু ঠিক হয়ে যাবে। তুমি একা নও এই দেশের অসংখ্য অসংখ্য বাচ্চা তোমার মত, কিন্তু সবাইকে টিকে থাকতে হবে। এইখানে আমি কিছু লিখতে চাই না, তুমি কি m.z.iqbal@mzi.rocks এ আমাকে একটা ইমেইল পাঠাবে প্লিজ? আমি যেটা বলতে চাই সেখানে বলব। (কিন্তু তুমি কী জান, তুমি কী সুন্দর করে লিখতে পার? কী ভয়ানক কথা কী সহজে হালকা করে লিখে ফেল? বড় হয়ে তোমাকে লেখক হতে হব, মনে থাকে যেন।)

প্রশ্ন: স্যার, ছোটবেলায় কি আপনার গোঁফ রাখার শখ ছিল অনেক….?? মানে স্যার আপনার কি গোঁফ অনেক ভালো লাগত??? নুহা, মগবাজার ঢাকা
উত্তর: না—না—না— গোঁফের মাঝে ভালো লাগার কি আছে? নাকের নিচে ঝাড়ুর মত কী একটা ঝুলে আছে, ছিঃ!! তার উপর যদি সর্দি হয়— ইয়াক থু!

প্রশ্ন: স্যার, আপনি ‘বনবালিকা’ বইটার উপরে বড় করে লিখে দিয়েছেন, ‘বৈ জ্ঞা নি ক ক ল্প কা হি নী’। কিন্তু আমিতো বইটার মধ্যে এই সম্পর্কিত কিছুই পাইনি।আমার কাছে এটি কিশোর উপন্যাস এর মত লেগেছে।প্লিয উত্তর দিবেন। আলিফ,রিদিতা
উত্তর: উড়তে পারে মানুষ, পানির নিচে থাকতে পারে মানুষ এরকম বইগুলোকে যেহেতু আগে সায়েন্স ফিকশন বলেছি তাই পশুর কথা বুঝতে পারে মানুষের বইকেও সায়েন্স ফিকশন বলেছি! তুমি যদি এটাকে কিশোর উপন্যাস বলতে চাও বল, তাতে আমার কোন আপত্তি নেই! রূপকথা বললেও আপত্তি নাই!

প্রশ্ন: স্যার আমি শুনেছি যে আপনি হুমায়ুন আহমেদ এর ভাই, সেটা কি সত্যি? আর আপনি কি হুমায়ূন আহমেদের বিভিন্ন ধারাবাহিক নাটক যেমন: ‘বহুব্রীহি’, ‘কোথাও কেউ নেই’ এ গুলো দেখেছেন? দেখে থাকলে এ নাটকগুলোর মধ্যে আপনার প্রিয় চরিত্র কোনগুলো? ওয়াসিফ, ঢাকা।
উত্তর: হ্যাঁ, আমি হুমায়ূন আহমেদের ভাই! হ্যাঁ আমি তার নাটকগুলো দেখছি। বাকের ভাই আমার একটা প্রিয় চরিত্র।

প্রশ্ন: স্যার আপনি একটা প্রশ্নের উত্তরে বলেছেন যে আপনার “হিসাবে” এখন করোনা বাড়ার কথা না। এর মানে আপনার কি কোনো আলাদা হিসাব-নিকাশ বা থিওরি আছে করোনা সংক্রমণ সংক্রান্ত? ? হিসাবটা কী রকম?? আমাদের জানাবেন প্লিজ?? লাইবা তাফান্নুম, ঢাকা।
উত্তর: আমি গত প্রায় ৫০০ দিন করোনার ডাটা আমার নিজস্ব সফটওয়ারে প্লট করে আসছি, কাজেই বলতে পার আমার নিজস্ব একটা  হিসাব আছে! আমি যেহেতু বিশেষজ্ঞ না, তাই দশজনকে বলে বেড়াই না।

প্রশ্ন: স্যার, যদি কিছু মনে না করেন তাহলে একটা প্রশ্ন করি। “লাবু এলো শহরে” বইটিতে লেখা আছে যে বাংলাদেশে দুইশ পঞ্চাশটা চা-বাগানের মধ্যে মাত্র চার/পাঁচটা বাগানে স্কুল আছে। কথাটি কি সত্যি? আমার তো একদমই বিশ্বাস হচ্ছে না! (সাবা, ঢাকা)
উত্তর: বইটি যখন লিখেছি তখন অবশ্যই সত্যি ছিল। এত দিন পরে অবস্থার উন্নতি হয়েছে কি না জানি না। চা বাগানের শ্রমিকদের জীবনটি খুবই কষ্টের। অবিশ্বাস্য।

প্রশ্ন: প্রিয় স্যার, আমি সাদিক। বাগেরহাট থেকে বলছি। … … … … পরিক্ষায় যা লিখি তা সরাসরি বইয়ের মত হয় না, আমি যেমনটা বুঝছি ঠিক তেমনটা লিখি। আমার শিক্ষকেরা বলেছেন যে আমার লেখায় কোনো ভুল নাই, কিন্তু তারপরেও তারা বলেন যে বইয়ে যেমন আছে ঠিক তেমনটাই লিখতে হবে, না হলে ঠিকমতো মার্ক পাওয়া যাবে না।
… … … … … …
আমি বেশিরভাগ বায়োলজিক্যাল টার্মের ইংরেজি নামগুলো জানি, বাংলা টার্ম তেমন জানি না। বাংলা টার্মগুলা বেশ উদ্ভটও বটে, যেমনঃ উদ্ভিদের Gynoecium (স্ত্রীস্তবক) এর প্রধান অংশ Ovary/ডিম্বাশয় যার মধ্যে থাকে Ovule/ডিম্বক… … … বাংলায় ডিম্বাশয়, ডিম্বক, ডিম্বকরন্ধ্র, ডিম্বকনাভি ইত্যাদি টার্মগুলা আমি ঘোলায়ে ফেলি অথচ ইংরেজি টার্মগুলা কতই না চমৎকার। আর সবচেয়ে বড় কথা হল আমরা যখন আর দুই বছর পরে কলেজে পড়ব কিংবা মেডিকেলে কিংবা কোনো ভার্সিটিতে হায়ার স্টাডিজ করব তখন তো আমাদের ঠিকই ইংরেজি টার্মগুলা শিখতে হবে, বাংলা টার্মগুলার কোনো প্রয়োজনও পড়বে না, তাই খালি খালি এই দুইটা বছরের জন্য কয়েকটা উদ্ভট বাংলা শব্দ শেখার প্রয়োজনীয়তা আমি বুঝতে পারলাম না। … … … … তো আমার মুল কথাটা হচ্ছে, আপনি একবার বলেছিলেন, “আমার স্টুডেন্টরা A+ পাবে ঠিকই। কিন্তু তারা বুঝে A+ পাবে, মুখস্ত করে নয়।” আমি তখন থেকেই সবকিছু বুঝে পড়ার চেষ্টা করি এবং করে যাচ্ছি। A+ পাওয়া আমার উদ্দেশ্য না, তবে আমার বড্ড ভয় করে। যদি আমার এই নিজের মত লেখার কারণে কিংবা কিছু ইংরেজি টার্ম ব্যাবহার করার কারণে আমি A+ মিস করি, তখন কী হবে? … … … … আসলে আমি জানি না দিন শেষে আমি কী পাব, কিন্তু আমার শিক্ষকেরা আমার মধ্যে একটা ভীতি ঢুকায়ে দিছে যে আমি আমার মত বুঝে লিখলে কিংবা বায়োলজিতে কিছু ইংরেজি টার্ম ব্যাবহার করলে আমি ভালো মার্কস পাবো না। তাই ভয়ে ভয়ে জর্জরিত হয়ে আপনার শরণাপন্ন হইছি, আপনি আমাকে অন্তত এইখানে কিছু লিখে হেল্প করুন। প্লিজ।
উত্তর: বৈজ্ঞানিক ইংরেজি শব্দের প্রতিশব্দ বলে একটা কথা আছে, সেটা খুব অস্বাভাবিক কিছু নয়। তুমি যেহেতু বিশ্ববিদ্যালয় পর্যন্ত যাবে তাই ইংরেজি প্রতিশব্দ না শিখে ইংরেজি শব্দগুলো শিখলে ক্ষতি নেই, কিন্তু সেটা সবার জন্য সত্যি নয়। যারা এসএসসি পর্যন্তই পড়বে তাদের জন্য বাংলা প্রতিশব্দ দিয়ে পড়া সহজ, তোমার কাছে সেটা যত হাস্যকরই মনে হোক। যতদিন তোমাদের পাঠ্য বইয়ে ইংরেজি শব্দগুলো ব্যবহার না করছে ততদিন তোমার শিক্ষকেরা বাংলা প্রতিশব্দে প্রশ্নোত্তর চাইতেই পারেন। তবে যদি কোন শিক্ষক বলেণ যে শুদ্ধ লেখা যথেষ্ট নয় অবশ্যই বইয়ের ভাষায় লিখতে হবে তাহলে সেটি মোটেও ঠিক নয়, রীতিমত অন্যায় একটি কথা। উল্টোটাই হওয়ার কথা, হুবহু বইয়ের ভাষায় লিখতে নিরুৎসাহিত করা উচিৎ। না বুঝে পড়াশুনাকে আমি পড়াশুনা হিসাবে ধরি না, কাজেই আমাকে জিজ্ঞাসা করলে এই উত্তর পাবে।
তবে লক্ষ করছি তোমরা প্রায় সময়েই লেখাপড়া পরীক্ষা, পরীক্ষার নম্বর এগুলো নিয়ে কথা বল, আমি এগুলো জানি না, জানতেও চাই না। আমি শুধু “শেখা এবং জানা” ব্যাপারটা বুঝি, অন্য কিছু বুঝি না কাজেই পরীক্ষা, পরীক্ষার নম্বর নিয়ে আমার সাথে কথা বলে লাভ নেই। ( প্লিজ, লেখালেখির সময় “ঢুকায়ে দিছে” “হইছি” এভাবে না লিখে শুদ্ধ ভাষায় লিখো! আমি তোমাদের কাছে সুন্দর ভদ্র ভাষা আশা করতেই পারি।)

প্রশ্ন: হাসিন, ঢাকা। স্যার আপনার কি কখনো রাইটার্স ব্লক হয়েছে? হয়ে থাকলে তখন কেমন লাগছিল?
উত্তর: আসল লেখকদের রাইটার্স ব্লক হয়। আমি মনে হয় ভূয়া লেখক। আমার এইসব fancy বিষয়গুলো হয় না। অনেক কিছু লিখে পছন্দ হয় নাই বলে পুরোটা ফেলে দিই— এরকম মাঝেই মাঝেই হয়। জীবনে যত লিখেছি তার অর্ধেক মনে হয় ফেলে দিয়েছি।

প্রশ্ন: স্যার, আপনি কি স্ট্রিং থিওরি বিষয়ে কোনো বই লিখেছেন? না লিখে থাকলে, আপনাকে অনুরোধ করব, এই বিষয়ে একটি বই লেখার জন্য। ইন্টারনেট থেকে কয়েকটা আর্টিকেল পড়ে, এ বিষয়ে আমার আরও জানার ইচ্ছে হয়েছে, তাই যদি কোনো বই/রিসোর্স বলেন, তবে খুবই খুশি হতাম আকিফ সামী মালিথা কুষ্টিয়া জিলা স্কুল
উত্তর: না লিখিনি। ‘একটুখানি বিজ্ঞান’ বইয়ে হয়তো একটু লিখেছিলাম। বড় বড় বিষয় লেখার আগে আমি কিছু জরুরী ছোট বিষয় লিখতে চাই।

প্রশ্ন: স্যার, আপনার বইয়ে সব (প্রায়) খারাপ স্যারদের নাম রাজ্জাক হয় কেন??? (ইলা,নরসিংদী )
উত্তর: হয় নাকি? বইগুলোর নামগুলো বলবে প্লিজ?

প্রশ্ন: (***, ঢাকা) আসসালামু আলাইকুম স্যার। আশা করি ভালো আছেন। আজ মহাসপ্তমী, আমার বাসার পাশের গলিতেই ইয়া বড় মন্দির! একটু পরপরই ঢাকের আওয়াজ শোনা যায়, কী যে খুশি খুশি লাগে না! কত সুন্দর করে সবকিছু সাজায়, কত লাইটিং! মানুষ কী সুন্দর করে সাজগোজ করে হাসিহাসি মুখে মন্দিরে আসে! আর মেলার কথা না-ই বললাম! প্রতিবছরই তাই দুর্গাপূজা আসলে আমি ভীষণ আনন্দে থাকি। তবে এবছর ব্যতিক্রম, এবার আমি মন্দিরে যাব না বা যাওয়ার অনুমতি পাব না। আচ্ছা স্যার, বড়রা এমন অদ্ভুত কেন? আমি মুসলমান হয়ে দুর্গাপূজা দেখতে যাই- এই সহজ স্বাভাবিক কথাটা কোনো ‘বড়’ মানুষ (আম্মু ছাড়া) মানতেই পারে না! কেউ কেউ সরাসরি বলে, “পূজা দেখা হারাম, গুনাহ্ হয়।” আবার কেউ বলে, “হিন্দুরা তো মসজিদে আসে না, তাহলে তুমি কেন মন্দিরে যাবে?” অনেকে আবার নানারকম ইশারা-ইঙ্গিতে আমায় বুঝায় যে আমার মোটেও মন্দিরে যাওয়া উচিত না। আবার কেউ কেউ অদ্ভুত দৃষ্টিতে আমার দিকে তাকিয়ে থাকে। এমনকি যদি দুর্গাপূজার সময় মন্দিরের সামনে দিয়ে কোনো কারণে যেতে হয়, তাহলে আব্বু কড়াভাবে নিষেধ করে দেয় যেন আমি ভিতরে না তাকাই! এত ঝামেলা সত্ত্বেও আম্মু আমাকে প্রতিবছর ঠিকই পূজা দেখিয়ে নিয়ে এসেছে, কিন্তু এবছর মনে হয় আর সম্ভব হবে না। তাই মনটা অ-নে-ক বেশি খারাপ। স্যার বড়রা এমন কেন? কী হয় আমি মন্দিরে গেলে? শুধু মন্দিরে গিয়ে পূজা দেখা না, বিকেলবেলা যদি পাশের বাসার হিন্দু প্রতিবেশী ধূপ জ্বালিয়ে মিষ্টি টুংটাং শব্দ করে পূজা করে তাতেও দেখি বড়রা (আবার বললাম আম্মু ছাড়া) কেমন জানি বিরক্তবোধ করেন! আমার বেস্ট ফ্রেন্ড হিন্দু ধর্মের, ঈদের সময় তো ও আমাকে খুব ভালোমতো শুভেচ্ছা জানায়। এমনকি আযান দিলে তো ইসলাম ধর্মের মতো মাথায় ঘোমটাও দেয়! (আমি নিজেই দিতে ভুলে যাই!) আর রোজার সময় রোজা না রাখলেও ইফতারি ঠিকই খায়!!! বছরের শুরুতে নতুন বই পেলে অনেক ইনিয়ে-বিনিয়ে ইসলাম ধর্মের বই নিয়ে উল্টেপাল্টে দেখে! উদাহরণগুলো দিলাম বলার জন্য যে আমার ফ্রেন্ড তো হিন্দু হওয়া সত্ত্বেও আমাদের ধর্মকে যথেষ্ট সম্মান করে। তবে আমার আশেপাশের বড় মানুষরা কেন হিন্দু ধর্মীয় উৎসবে এত বিরক্তি প্রকাশ করে? আমি মন্দিরে গিয়ে মানুষের আনন্দ দেখে খুশি হচ্ছি, নিজের ধর্মকে তাতে নিশ্চয়ই অসম্মান করা হয় না! তাহলে কেন আমাকে বড়রা (আম্মু বাদে!) দুর্গাপূজা দেখতে দিবে না? এর মাধ্যমে কি হিন্দু সম্প্রদায়কে অশ্রদ্ধা করা হচ্ছে না? (পুনশ্চ: আমার নাম গোপন রাখবেন প্লিজ প্লিজ প্লিজ!)
উত্তর: তোমার লেখাটা পড়ে একদিকে আমার খুব মন খারাপ হল অন্যদিকে তোমার নিজের ভাবনা গুলো জানতে পেরে মনটা ভালো হয়ে গেল। কবে যে সবাই তোমার মত করে বুঝতে পারবে ‘ভিন্ন’ মানে খারাপ নয়, ‘ভিন্ন’ মানে বৈচিত্র। আর পৃথিবীর সৌন্দর্য হচ্ছে বৈচিত্রে। যারা এই বৈচিত্রের সৌন্দর্য উপভোগ করতে পারে না তারা পৃথিবীর সৌন্দর্যটাই উপভোগ করতে পারে না, তাদের কী নিরানন্দ একটি জীবন। তুমি সেরকম একজন নও দেখা খুব ভালো লাগল। শুধু এরকম থেকে যেও না, অন্যদেরও তোমার মতন করে তুলো। (এবারে বিজয়া দশমীর দিন সারা দেশে কী হল দেখেছ নিশ্চয়ই। দুবৃত্তরা যখন অপরাধ করে তার একটা সমাধান হয়তো খুঁজে পাওয়া যাবে, কিন্তু এই দেশের শিশুদের বাবা মায়েরা এরকম হলে তার সমাধান কী?)

প্রশ্ন: আমি শুধু আপনাকে কৃতজ্ঞতা ও ধন্যবাদ জানাতে চাই আপনার অসাধারণ সব বইগুলোর জন্য। বিজয় কুমার দাস বাসাবো,ঢাকা
উত্তর: থ্যাংকু বিজয়, আমার ছেলেমানুষি বইগুলোকে অসাধারণ বলার জন্য।

প্রশ্ন: আংকেল, আপনি কি খেয়াল করেছেন, এই ওয়েবসাইটের বয়স ৫ বছরের বেশি হয়ে গেছে! ইশতিয়াকের তো বিশ্বাসই হচ্ছে না। মনে হচ্ছে, এই তো সেদিনের কথা! তবে ভালোও লাগছে প্রথম থেকে এই ওয়েবসাইটের সঙ্গে থাকতে পেরে। আপনি হয়তো বলবেন, এই ওয়েবসাইটে এসে বাচ্চাকাচ্চারা সময় নষ্ট করে। তবে আমি কিন্তু মোটেও সেটা বলবো না। বরং এই ওয়েবসাইট (আরো ভালো করে বললে, এই প্রশ্ন উত্তরের ব্যাপারটা) যে কত হেল্পফুল সেটা বুঝিয়ে কিংবা গুছিয়ে বলতে পারছি না। আমি অবশ্য কখনোই কথা গুছিয়ে বা সুন্দর করে বলতে পারি না। জানেন, মাঝেমধ্যে এমনও হয়, আমি হয়তো প্রশংসা করলাম, অথচ ভাবলো বদনাম করছি! হিহিহি। ঢাকা থেকে ইশতিয়াক
উত্তর: সত্যি? পাঁচ বছর থেকে আমি এই পাগলামো করছি? সবাই এটা সহ্য করছে এখনও? কী সর্বনাশ!

প্রশ্ন: স্যার, আমি জ্যামিতি একদমই পছন্দ করি না। কীভাবে আমি জ্যামিতিতে ভালো করতে পারি?স্যার আমাকে একটা অটোগ্রাফ দিবেন, Please? আফিফা, ফুলপুর, ময়মনসিংহ।
উত্তর: হায়! হায়! জ্যামিতি পছন্দ করো না? এটা কীভাবে সম্ভব? বুঝে বুঝে জ্যামিতি করলে এর থেকে মজা আর কোথায় আছে? যখনই একটা সম্পাদ্য কিংবা উপপাদ্য পাবে সেটার সমাধান না দেখে প্রথমে নিজে নিজে চিন্তা করে দেখ তার সমাধান বের করতে পার কিনা। দেখবে অনেক সময় পেয়ে যাবে, তখন তোমার কত মজা হবে দেখ। আর যদি সম্পাদ্য উপপাদ্য মুখস্ত করার চেষ্টা কর তাহলে এর থেকে নীরস জঘন্য জিনিস আর কী হতে পারে? (তুমি নিশ্চয়ই কম্পাস আর রুলার ব্যবহার করে যেকোনো কোণকে দুই ভাগ করতে পার। যেকোনো কোণকে তিনভাগ করতে পারবে? চেষ্টা করো দেখি! দেখবে জ্যামিতি কত মজা। এই নাও অটোগ্রাফ।)

প্রশ্ন: ১৫/১০/২০২১ রূপসা,খুলনা স্যার, আশা করি ভালো আছেন৷আজ কয়েকটা বিষয়ে সংক্ষেপে লিখব৷ ১৷আপনাকে নিয়ে অনেকে খারাপ মন্তব্য করে৷আপনার খারাপ না লাগলেও আমাদের খুব খারাপ লাগে৷তবে তারা আপনাকে ভুল বুঝে৷আমার দুলাভাইকে আমি তপু বইটা পড়ে বলেছে কি শুনবেন৷বলেছে,আপনি সেখানে রাজাকার স্যারের মাধ্যমে বুঝাতে চেয়েছেন ,মুসলমানরা হিন্দুদের পছন্দ করে না৷কিন্ত রাজাকার স্যার তো নিজেই হিন্দু ছিল তাই না৷যইহোক,বড়রা যেমন ছোটদের মন বুঝেনা তেমনি ছোটদের বইও বুঝে না৷বইটা পড়ে আমার রবীন্দ্রনাথের প্রতি যে ক্ষোভ ছিল তা ঘুচে গেছে৷… … … … ৷ ৫৷আমি আমার ডাইরির ২৩ ডিসেম্বর পৃষ্ঠা আপনার অটোগ্রাফের জন্য ফাঁকা রেখেছি৷যদি কোনোদিন সরাসরি দেখা হয় ,সেই আশাই আছি৷ওয়েবসাইটে অটোগ্রাফ নেয়া আমার কাছে মজার কিছু না৷ আজ আর না৷মনে এখনো অনেক কথা জমে আছে৷যাইহোক,ভালো থাকবেন,স্বুস্থ্য থাকবেন,নিজের খেয়াল রাখবেন৷ধন্যবাদ৷ ইতি জিয়াদ হোসেন
উত্তর: আমাকে নিয়ে খারাপ মন্তব্য করলে তোমার কেন খারাপ লাগে? এমনি এমনি তো করে না, একটা কারণ আছে বলে তো করে-তাহলে সমস্যা কী? তোমার রবীন্দ্রনাথের উপর ক্ষোভ ছিল? কী সর্বনাশ! কেন? তোমার বয়সী বাচ্চাদের কেন রবীন্দ্রনাথের উপর ক্ষোভ? কারণটা একটু লিখবে? আমি জানতে চাই, বুঝতে চাই। যদি তোমার রবীন্দ্রনাথের উপর ক্ষোভ থাকে তাহলে তোমার দুলাভাইয়ের সাথে তোমার পার্থক্যটা কোথায়? (তোমার সাথে দেখা হলে অবশ্যই তোমার ডাইরিতে অটোগ্রাফ দিয়ে দেব)

প্রশ্ন: স্যার, আমার সালাম নিবেন। আশা করি ভালো আছেন। আপনি ব্ল্যাকহোলের বাচ্চা বইটিতে লিখেছিলেন মিঠুন এসেছে অক্সব্রীজ ইন্টারন্যাশনাল স্কুল থেকে। আমার খুব জানতে ইচ্ছে করছিল এই নামে আদোউ কোন স্কুল আছে নাকি। তবে, কালকে চট্টগ্রামের একটি রাস্তা দিয়ে আসার সময় দেখি বিশাল বড় সাইনবোর্ড লাগানো ” অক্সব্রীজ স্কুল, ইংলিশ মিডিয়াম, ভর্তি চলছে” আমি এত অবাক হয়েছি যে বলার মত না। আপনি কি এই স্কুলের কথাই লিখেছিলেন? আর, আমাকে অটোগ্রাফ দেয়ার জন্য 1 googolplex ধন্যবাদ! ঐন্দ্রী চক্রবর্তী, চট্টগ্রাম,
উত্তর: আমি অনেক চিন্তা ভাবনা করে ‘অক্সব্রীজ’ নামটা বের করেছিলাম! ভেবেছিলাম এই নামে নিশ্চয়ই কোনো নাম থাকবে না— এখন তুমি বলছ সত্যি আছে? কী সর্বনাশ! স্কুল কতৃপক্ষ আমার নামে মানহানির মামলা না করে দেয়!

প্রশ্ন: Assalamualaikum sir. Amar proshno holo apni ki gaan gete paren? Maisha, address: Uposhohor, Rajshahi.
উত্তর: না—না—না— আমি গান গাইতে পারি না। আমার গলায় কোন সুর নাই! চেষ্টা করলে গানের মত না শুনিয়ে মেশিনগানের মত শোনাবে! (তবে হ্যাঁ, গান শুনতে খুব ভালো লাগে!)

প্রশ্ন: গতকাল আমি প্রথমবার একটা জোঁক দেখলাম। গল্পে যেরকম ভয়ংকরভাবে জোঁকের কার্যকলাপের বর্ণনা দেওয়া থাকে তাতে আমি ভেবেছিলাম এটার সাইজ নিশ্চয়ই অনেক বড়সড় হবে, কিন্তু আমি যেটা দেখলাম সেটা এইটুকুন।(মনে হয় বাচ্চা) অনায়াসে সুতা বলে ভুল করা যায়।(যখন কিলবিল করে উঠলো তখন আমি ঝেড়ে একটা দৌড় দিয়েছি) স্থানীয় ভাষায় এটাকে টিনা বলে। টিনাই যে জোঁক সেটাও আমি জানতাম না। আফরিদাহ আহসান, ইসলাম নগর, ঠাকুরগাঁও সদর, ঠাকুরগাঁও
উত্তর: ইন্টারেস্টিং! জোঁককে টিনা বলে? একটা নূতন জিনিষ শিখলাম। সূতার মত জোঁক দেখে ভয় পাওয়ার কিছু নাই— কিন্তু বাঘা বাঘা পানির জোঁক খুবই মারাত্মক। পানিতে কিলবিল করে সাঁতরায় তারপর কামড়ে ধরলে টেনে ছোটানো যায় না— টানলে রবারের মত লম্বা হতে থাকে। রক্ত খেয়ে ঢোল হয়ে যায়।

প্রশ্ন: Sir, i am humayra..i read in class 5..i almost read your all books…i like to read your books..i am your big fan.i wish i can meet with you one day..it’s a dream of my life…i hope you always be well like this inshallah…how can i meet with you sir…?Humayra Anjum, Dhaka
স্যার আমার একটা সমস্যা আছে সেটা হলো আমি যতো ভালো করেই পড়ি না কেনো আমি যখন ম্যাম/স্যার এর কাছে পড়া দেই তখন আমার অনেক ভয় করে এবং আমি সবকিছু ভুলে যাই এর কী কোনো সমাধান আছে?থাকলে আমাকে প্লিজ বলবেন,হুমায়রা আনজুম, ঢাকা
উত্তর: এই তো করোনা কমে যাচ্ছে আবার আমরা আগের জীবনে ফিরে যাব, তখন আবার সবার সাথে সবার দেখা হবে! আমি এর মাঝে নানা মিটিংয়ে যেতে শুরু করেছি। তোমার সাথেও নিশ্চয়ই একদিন দেখা হবে। ম্যাম/ স্যারের কাছে পড়া দিতে ভয় করে? আমাকে দিতে হলে আমারও নিশ্চয়ই ভয় করতো— কিন্তু পড়া দিতে হবে কেন? আমি পড়ব আমার নিজের জন্য সেটা নিয়ে স্যার ম্যাডামের মাথা ব্যথা কেন?

প্রশ্ন: আপনি আমার প্রশ্নের জবাবে জানতে চেয়েছিলেন, আমি কী করে রাজশেখর বসুর চমৎকার ছোটগল্পের সন্ধান পেলাম।প্রথমে বিশ্ব সাহিত্য কেন্দ্র থেকে প্রকাশিত ‘কিশোর আনন্দ’ সিরিজে ওনার লেখা দুটো গল্প পড়েছিলাম।এরপর এতো মুগ্ধ হয়েছিলাম যে,নেট থেকে খুঁজে খুঁজে pdf আকারে রাজশেখর বসুর ‘পরশুরাম গল্পসমগ্র’ নামিয়ে পড়ে ফেলেছি।মাঝে মাঝেই গল্পগুলোয় চোখ বোলাই।এত্তো ভালো লাগে! ইতি, পিনাকী গুপ্ত,খুলনা।
উত্তর: চমৎকার! তুমি কী রাজশেখর বসুর অনুবাদ রামায়ণটি দেখেছ? পড়েছ? রাম সম্পর্কে যে বিশেষণগুলো আছে সেটা পড়ে হতবাক হয়ে গেছি। কোনো একদিন কোনো একটা বইয়ে আমি চেষ্টা করে দেখব আমি কারো সম্পর্কে এরকম বিশেষণ বানাতে পারি কিনা!

প্রশ্ন: স্যার, আয়না আর কাচের মধ্যে পার্থক্য কি?? আমি আমার বোনকে জিজ্ঞেস করলাম আয়না আর কাচের মধ্যে পার্থক কি… সে আমায় গম্ভীর গলায় বলল, ” তুই শুধু জেনে রাখ আয়না আর কাঁচ এক না ” । তারপর আমি বললাম, “আয়না আর কাচের পার্থক্য কি আমি সেটা জিজ্ঞেস করেছি আমি মোটেও বলি নাই আয়না আর কাঁচ এক নাকি এক না।” তারপর আমার বড় বোন আমায় হুঙ্কার দিয়ে বলল, “আমি জানি না তুই এখন ভাগ এখান থেকে। নিজের কাজ নিজে কর “। কলেজে পড়া একজন ছাত্রীর কাছে আমি মোটেও এরকম উত্তর আশা করি নি..। আচ্ছা স্যার বড়রা এত আজব হয় কেন? তাই বাধ্য হয়ে ই আপনার কাছে আসা। আপনি একটু প্লিজ বলবেন আয়না আর কাচের মধ্যে পার্থক্য টা কি??? নাকি আপনি আবার আমার আপুর মতো হুঙ্কার দিবেন?? নুহা, মগবাজার, ঢাকা।
উত্তর: আয়না হচ্ছে আয়না, কাচ হচ্ছে কাচ! (তুমি নিশ্চয়ই আয়নায় নিজের মুখ দেখেছ, কাচের জানালা দিয়ে বাইরের গাছপালা, পাখি দেখেছ— কাজেই দুটোর পার্থক্য জান না সেটা হতে পারে না!) প্রতিফলন হলে আমরা আয়না বলি অর্থাৎ চকচকে ধাতু দিয়েও আয়না হতে পারে। কাচ হচ্ছে SiO2 এর একটা রূপ। বালু আর সোডা গরম করে কাচ তৈরি করা যায়। (বড়রা আজব হয় দেখে কেন অবাক হচ্ছ? সবাই জানে বড়রা আজব, তুমি এখনও টের পাও নাই? তোমার কপালে দুঃখ আছে!)

প্রশ্ন: আমি আসলে প্ল্যানচ্যাটের ব্যাপারটা ঠিক বিশ্বাস করে উঠতে পারিনা। আচ্ছা সত্যি করে বলুন তো রঙিন চশমা বইয়ের ঘটনা দুইটা কি আসলেই সত্যি?? তাহলে তো বিশ্বাস করতেই হয়। ( বাঁধন, class 8 টাঙ্গাইল )
উত্তর: ‘রঙিন চশমা’ বইয়ের ঘটনাগুলো সত্যি— কিন্তু সেটা মোটেই অলৌকিক কোন ব্যাপার প্রমান করে না— self hypnotism এর মত কোন একটা ব্যাপার ঘটে!

প্রশ্ন: স্যার আশা করি আপনি ভালো আছেন। আমি আপনার একজন অনেক বড় ভক্ত। গতকাল আমি আপনার নতুন বই “আমার ডেঞ্জারাস মামী” পড়া শেষ করলাম। কিন্তু ১টা প্রশ্ন আমার মাথায় ঘুরছে টুলুর মতো। নিরা মামী টুলুকে “গণিতের মজার অংক”নামে একটি বই কিনে দেন যা দিয়ে টুলু গণিত ধ্বংস আন্দোলন করতে চেয়েছিলো। বইটিতে একটি অংক ছিল যে : দুটি সংখ্যা বের করো যে দুটি সংখ্যা যোগ করলেও ২ হবে, গুন্ করলেও ২ হবে। আমার প্রশ্ন হলো: এই ২ টি সংখ্যা কি কি ? নাম :- মেধা, ঠিকানা-মিরপুর,ঢাকা।
উত্তর: আমি এটা দিয়েছি যেন সমস্যাটা মাথায় ঘুরঘর করে। তাহলে তোমার মাথায় ঘুর ঘুর করতে শুরু করেছে? ভেরি গুড! চিন্তা করো দেখি বের করতে পার কী না!

প্রশ্ন: আসসালামুয়ালাইকুম আমার নাম ফাহিম ফয়সাল কানন। আমার বিদ্যালয়ের নাম হাতীমারা হাই স্কুল। আশা করি স্যার আপনি অনেক ভালো আছেন। নিম্নে আমার প্রশ্নটি দেয়া আছে-
বলই ত্বরণ সৃষ্টি করে! ব্যাখ্যা করো।
উত্তর: সর্বনাশ! এখন তুমি আমার পরীক্ষা নেবে? কেন? আমি কী দোষ করেছি? এই প্রশ্নের উত্তরের জন্য কত মার্কস? উত্তর লেখার জন্য কতক্ষন সময়?

প্রশ্ন: স্যার, অনেক অনেক শ্রদ্ধা আর ভালোবাসা। আমি মিহিরিমা আলম, চট্টগ্রামে থাকি। এবারের এসএসসি পরীক্ষার্থী। আমার একটা সমস্যার কথা আপনাকে বলতে চাই। সমস্যাটা যদিও একটু বড় আর জটিল, তা-ও যদি আপনি কষ্ট করে পড়ে সমাধান দেওয়ার চেষ্টা করেন, আমি খুব খুশি হব। এই করোনাকালীন সময়ে প্রায় দেড় বছর ঘরে বন্দী ছিলাম। প্রায় একাই। আমার কোনো ভাইবোন নেই, আর মা ব্যাংকার, বাবা ডাক্তার। তাই পুরো করোনাকালেই তারা অফিস করেছেন, বাড়িতে কেউ ছিল না। শুধু রাতে আর সকালটুকু তাদের দেখা পেতাম। স্কুল খোলার পরে অনেক আশা নিয়ে স্কুলে গেছি, বন্ধুদের সাথে দেখা হবে, আবার আমরা পুরোনো জীবনে ফেরত যাবো, ঠিক যেখানটায় সবকিছু থেমে গেছিল, সেখান থেকে সব আবার নতুন উদ্যমে শুরু হবে। কিন্তু যেরকমটা ভাবলাম সেরকমটা হলো না।
আমি আগে প্রত্যেকদিন স্কুলে যেতাম। ঝড়-বাদল কোনো কিছুই আমাকে থামাতে পারত না। স্কুলে যেতে ভালবাসতাম খুউব। এমনও হয়েছে, খুব ঝড় চলছে, দেশজুড়ে ছয় কি সাত নম্বর সংকেত, তার মধ্যে ছাতা মাথায় আধাভিজে হয়ে আমি স্কুলে চলে গেছি। গোটা স্কুল শুনশান, শিক্ষার্থীরাও কেউ আসেনি, শিক্ষকেরাও না। অবশ্য পরে হেডমাস্টার স্যারের ঝাড়ি খেয়ে বাড়ি ফিরে এসেছি আবার! পুরো স্কুলে প্রথম-দ্বিতীয়-তৃতীয় কক্ষনো হইনি, কিন্তু সর্বোচ্চ উপস্থিতির পুরস্কারটা সবসময়েই বগলদাবা করে নিয়ে এসেছি।
স্যার, দুঃখিত, ভূমিকাটা সামান্য বড় করে ফেলেছি। এই কথাগুলো না বললে হয়তো আমার সমস্যাটা বুঝবেন না, তা-ই বলা। ক্ষমাসুন্দর দৃটিতে দেখবেন প্লিজ। এবার মূল সমস্যাটা বলি। আগে খুব ছটফটে আর চঞ্চল ছিলাম। হাসতাম, দৌঁড়ে বেড়াতাম, দুষ্টুমি করতাম, সবার সাথে কথা বলতাম, খোঁজ নিতাম, কেউ দুইতিনদিন স্কুলে না আসলে তার বাড়ি চলে যেতাম খবর নিতে। ক্লাসে সব্বার খুব ভালো বন্ধুত্ব ছিলো আমার সাথে। এখন কেন জানি সেরকম নেই আমি। লকডাউনের ভেতরে কী কী করেছি, কী কী ঘটেছে- কিচ্ছু মনে করতে পারি না। মানে এই দেড় বছর পুরোটা আমার মাথায় কেমন জানি ধোঁয়ায় ঢেকে গেছে। এই পুরো সময়টার কোনো কিছুই মনে থাকে না। স্কুলে যেতে ভালো লাগে না, ঘর থেকে বের হতেই ইচ্ছে করে না। টানা তিন সপ্তাহ স্কুলে যাইনি আমি। কারও সাথে কথা বলতে খুব পরিশ্রম হয়, কথা বলতে ইচ্ছে করে না, কিছু পড়লে মনে থাকে না, গুছিয়ে লিখতে পারি না, ভাবতে পারি না। সারাক্ষণ ঝিম মেরে বসে থাকি। কেমন যেন চুপচাপ আর বিষন্ন হয়ে গেছি। নিজেকে নিজেই চিনতে পারি না।
আমি এরকম কেন হয়ে যাচ্ছি স্যার?
উত্তর: তোমার প্রশ্নটা অনেক বড়, তারপরেও আমি পুরোটা দিয়ে দিয়েছি তার দুটি কারণ। এক: তুমি যেটা বলতে চেয়েছ সেটা এত সুন্দর করে প্রকাশ করেছ যে তার তুলনা নেই! অন্যরা দেখুক কেমন করে কিছু একটা লিখতে হয়! দুই: তুমি যে সমস্যার কথা লিখেছ সেই সমস্যা হয়তো আরও অনেকেরই আছে।
যাই হোক, তোমাদের এই বয়সটা একটু অন্যরকম। বয়ঃসন্ধির সময় শরীরে নূতন নূতন হরমোন তৈরি হয়— মানসিক পরিবর্তন হতেই পারে। আমি মোটামুটি নিশ্চিত এটা সাময়িক। তুমি এটা নিয়ে দুর্ভাবনা করো না, আবার আগের মত ছটফট আর চঞ্চল হয়ে যাও। করোনার এই দীর্ঘ সময় অনেকের জীবনকে এলোমেলো করে দিয়েছে, আবার গুছিয়ে নাও। দরকার হলে একটু চেষ্টা করেই, আমি বলছি তুমি পারবে। (আবার বলছি, তুমি কী সুন্দর করে পুরোটা লিখেছ দেখেছ? আমি পড়ে মুগ্ধ হয়ে গেছি। যে এত সুন্দর করে গুছিয়ে কোন একটা কিছু ব্যাখ্যা করতে পারে তাকে নিয়ে আমার বিন্দুমাত্র দুর্ভাবনা নেই। দেখবে সব ঠিক হয়ে যাবে। আরেকটা কথা বলতে ভুলে গেছি, তোমার নামটা খুব সুন্দর, মিহিরিমা!)

প্রশ্ন: … … … … (এই দাঁড়ি চিহ্ন পর্যন্ত আমার নাম-শ্রেণি সব গোপন রাখবেন প্লিজ) আমার বড় হয়ে গবেষণার পাশাপাশি শিক্ষকতা করার ইচ্ছা। আমি শিক্ষকদের পর্যবেক্ষণ করে করে শিক্ষার্থীদের সাথে কী কী জিনিস করা উচিৎ আর কী কী করা উচিৎ নয় সেটা বের করার চেষ্টা করি। আপনি আপনার শিক্ষকজীবনের আলোকে আমাকে কিছু উপদেশ দিবেন প্লিজ ? আর হ্যাঁ, অনেকদিন আগে আমার করা প্রশ্নের উত্তর দিয়েছিলেন, সেটার ধন্যবাদ জানানো হয়ে ওঠেনি। জানি আপনার মনে নেই, তবুও সেদিন আমার মন ভালো করে দেওয়ার জন্য আসলেই অনেক ধন্যবাদ!— আজ এটুকুই।।
উত্তর: তুমি শিক্ষক হতে চাও জেনে খুব খুশি হয়েছি। আমার ধারনা শিক্ষকতার মত আনন্দের কোনো কাজ নেই। ভালো শিক্ষক হওয়া খুব সহজ, তোমার ছাত্রছাত্রীদের ভালবাসতে হবে, এবং তার চেয়ে বড় কথা, তাদের সম্মান করতে হবে। তা ছাড়া তুমি যে তাদের পর্যবেক্ষণ করে যাচ্ছ, সেটা খুবই ভালো বুদ্ধি, চমৎকার! (তুমি তো এমন কিছু সিরিয়াস বিষয় লিখনি, তাহলে নিজের নাম পরিচয় গোপন রাখছ কেন? নিজের পরিচয় দিয়ে কথা বলা কী ভালো নয়?)

প্রশ্ন: স্যার, ভুলভাল বিজ্ঞানে কি সাইন্স ফিকশন লেখা যায়?! উত্তরটা জানা আমার জন্য অ—-নে—–ক বেশি জরুরি!! প্লিজ বলুন!!!! – সাবা, ঢাকা (নামটা কেটে দিয়েন প্লিজ)
উত্তর: যেহেতু ফিকশন তাই ভুলভাল লেখা যায়। যত ইচ্ছা ভুলভাল লেখ সমস্যা নাই। কেউ তোমার বিরুদ্ধে মামলা করবে না!

প্রশ্ন: আমার নাম বন্যা বড়ুয়া। আমি চট্টগ্রামে থাকি। স্যার,,আমি storyteller হতে চাই তাই আমাকে কি করতে হবে বা কি করলে আমি storyteller হতে পারবো???
উত্তর: ছোট বাচ্চাদের গল্প শুনিয়ে মুগ্ধ করে রাখো—তাহলেই ভালো story teller হয়ে যাবে!

প্রশ্ন: Sir amar bashay khub jhamela.basha theke palay jaite plan korlam. Shamner mash e jabo dowa koirren. Sakib Al Hasan. 14 green square Green road Dhaka 1205
উত্তর: না, তুমি বাসা থেকে পালাবে আমি সেজন্য দোয়া করব না। দোয়া করি তোমার ঝামেলা যেন মিটে যায়—কিংবা তুমি ঝামেলাটার মুখোমুখি হয়ে সেটা overcome করতে পার।

প্রশ্ন: স্যার, আপনার প্রিয় খাবার কি? (তাসনিম আহমেদ, বগুড়া)
উত্তর: মুড়ি।

প্রশ্ন: স্যার, আপনি এক বছরে সর্বোচ্চ কতগুলো বই পড়েছেন? ইমাম সোহরাওয়ার্দী, বনশ্রী, ঢাকা।
উত্তর: এখন আর সেরকম বেশি বই পড়া হয় না। ছেলেবেলায় অনেক পড়েছি সঠিক সংখ্যা তো বলতে পারব না। কলেজে থাকতে ক্লাস ফাঁকি দিয়ে প্রত্যেকদিন পাবলিক লাইব্রেরি গিয়ে বই পড়তাম!

প্রশ্ন: ইস্টিশন বই নিয়ে আমার কিছু কথা, স্যার আমার পড়া আপনার সেরা গল্পের বই হোল এই ইস্টিশন বইটি। আপনার অসাধারণ শব্দ চয়ন ও মায়া ভরা আবেগ দিয়ে লেখা বইটি খুব সুন্দর। বইটি আমার মত বাচ্চা যারা ইংলিশ মিডিয়াম স্কুলে পড়ে তাদের জন্য একটি নতুন চিন্তার খোরাক দিবে। আমাদের আশেপাশে আমাদের মতই কিছু ছেলে মেয়দের জীবন অনেক কষ্টের দুঃখের, সেখানে আমরা কত আরামে থেকেও আমাদের অনেক দুঃখ অনেক হতাশা, একটু বই কিনে নাহ দিলেই আমরা মন খারাপ করে বসে থাকি। কিন্তু আমাদের মতই বাচ্চারা রাস্তাই নাহ খেয়ে , জামা কাপর নাহ পড়েই দিনের পড় দিন কাটাই।এই কথা গুলো চিন্তা করলে মনে হয় আমাদের হতশা মন খারাপ হয়ার কথায় নাহ , রাদের তুলোনাই আমরা কত আরামে আছি, কতই নাহ ভালো আছি। স্যার আপনাকে অনেক ধন্যবাদ এরকম অসাধারণ বইটি লেখার জন্য। বইয়ের শেষ কথা পড়ে আপনার প্রতি আমার ভালোবাসা ও শ্রদ্ধা আর কয়েকগুন বেরে গেল। স্যার একটা রিকুয়েস্ট ? আপনি ফেসবুকে নিয়মিত লেখেন নাহ। … … … ফেসবুকে আপনি নিয়মিত লেখলে খুব খুশি হব,… … …
উত্তর: ‘ইস্টিশন’ বইটা তোমার ভালো লেগেছে জেনে খুশি হয়েছি। আমি বানিয়ে বানিয়ে ইস্টিশনের বাচ্চাগুলো সম্পর্কে ভালো ভালো কথা লিখেছি কিন্তু আসলে তো ছেলেমেয়েগুলোর জীবন হচ্ছে অসম্ভব কঠিন। নির্দয়। মমতাহীন। আমি আসলে কখনোই ফেসবুক জাতীয় ব্যাপারে মাথা ঘামাইনি, পৃথিবীতে যখন আসল মানুষের সাথে যোগাযোগ করা যায় তাহলে কেন আমি ভারচুয়াল জগতে যোগাযোগ করব? যেসব কারনে এগুলো নিয়ে আমার এলার্জির মত আছে, মাত্র এখন অন্যরা সেটা টের পেতে শুরু করেছে। (আমার কাছে তো আমার বইয়ের পি ডি এফ নেই কোথা থেকে দেব? নেটে নিশ্চয়ই আছে, খুঁজে নাও প্লিজ।)